Está en la página 1de 141

Notas de Clase

Variable
Compleja
enero-abril 2014

J. Juan Rosales Garcı́a, Manuel Guı́a Calderón y J. Francisco


Gómez Aguilar
Universidad de Guanjuato, Capus Irapuato-Salamanca
División de Ingenierı́as, Departamento de Ingenierı́a Eléctrica
rosales@ugto.mx, guia@ugto.mx
ii

.
Índice general

1. Álgebra de los Números Complejos 1


1.1. Introducción . . . . . . . . . . . . . . . . . . . . . . . . . . . . . . 2
1.2. Los Números Complejos y sus Propiedades . . . . . . . . . . . . 2
1.3. Propiedades de los Números Complejos . . . . . . . . . . . . . . 3
1.4. Propiedades de la Conjugación. . . . . . . . . . . . . . . . . . . . 5
1.5. Ejemplos . . . . . . . . . . . . . . . . . . . . . . . . . . . . . . . 6
1.6. Interpretación Geométrica de los Números Complejos . . . . . . 10
1.7. Ejemplos . . . . . . . . . . . . . . . . . . . . . . . . . . . . . . . 12
1.8. Notaciones de un Número Complejo . . . . . . . . . . . . . . . . 14
1.9. Extracción de Raı́ces de los Números Complejos . . . . . . . . . . 17
1.10. Ejemplos . . . . . . . . . . . . . . . . . . . . . . . . . . . . . . . 18
1.11. Conjuntos de Puntos, Cı́rculos y Discos. . . . . . . . . . . . . . . 30
1.12. Ejemplos . . . . . . . . . . . . . . . . . . . . . . . . . . . . . . . 32
1.13. Regiones del Plano Complejo. . . . . . . . . . . . . . . . . . . . . 35
1.14. Ejercicios Propuestos . . . . . . . . . . . . . . . . . . . . . . . . . 35

2. Funciones de una Variable Compleja 37


2.1. Funciones de una Variable Compleja . . . . . . . . . . . . . . . . 37
2.2. Sucesiones y Series Complejas . . . . . . . . . . . . . . . . . . . . 41
2.3. Ejemplos . . . . . . . . . . . . . . . . . . . . . . . . . . . . . . . 43
2.4. Funciones Complejas Elementales . . . . . . . . . . . . . . . . . . 44
2.5. Lı́mite y Continuidad de una Función Compleja . . . . . . . . . . 51
2.6. Derivada de una Función Compleja. . . . . . . . . . . . . . . . . 54
2.7. Funciones Armónicas Conjugadas. . . . . . . . . . . . . . . . . . 59
2.8. Mapeo Conforme . . . . . . . . . . . . . . . . . . . . . . . . . . . 62
2.9. Ejercicios Resueltos. . . . . . . . . . . . . . . . . . . . . . . . . . 63

3. Integrales Complejas 67
3.1. Integral de una Función Compleja. . . . . . . . . . . . . . . . . . 67
3.2. Integrales de Lı́nea. . . . . . . . . . . . . . . . . . . . . . . . . . . 69
3.3. Integral Compleja en Términos de Integrales Reales. . . . . . . . 71
3.4. Teorema de Cauchy-Goursat. . . . . . . . . . . . . . . . . . . . . 77
3.5. Forma Integral de Cauchy. . . . . . . . . . . . . . . . . . . . . . . 82

4. Series en el Dominio Complejo 93


4.1. Series de Potencias . . . . . . . . . . . . . . . . . . . . . . . . . . 93
4.2. Series de Taylor . . . . . . . . . . . . . . . . . . . . . . . . . . . . 96
4.3. Series de Laurent . . . . . . . . . . . . . . . . . . . . . . . . . . . 101
4.4. Ceros y Puntos Singulares Aislados de Funciones Complejas . . . 103

5. Series de Taylor y de Laurent: Singularidades 109


5.1. Series de Potencias . . . . . . . . . . . . . . . . . . . . . . . . . . 109

6. Residuos 119
6.1. Residuos . . . . . . . . . . . . . . . . . . . . . . . . . . . . . . . . 119
6.1.1. Residuo en el Infinito. . . . . . . . . . . . . . . . . . . . . 120
6.1.2. Residuos Logarı́tmicos - Principio del Argumento. . . . . 122
6.2. Integrales de Funciones Reales. . . . . . . . . . . . . . . . . . . . 124
6.2.1. Integrales de Funciones Racionales de cos(θ) y sen(θ). . . 124
6.2.2. Integrales Impropias de Funciones Racionales. . . . . . . . 126
6.3. Ejercicios Resueltos. . . . . . . . . . . . . . . . . . . . . . . . . . 128
Capı́tulo 1

Álgebra de los Números


Complejos

Contents
1.1. Introducción . . . . . . . . . . . . . . . . . . . . . . . 2
1.2. Los Números Complejos y sus Propiedades . . . . . 2
1.3. Propiedades de los Números Complejos . . . . . . . 3
1.4. Propiedades de la Conjugación. . . . . . . . . . . . . 5
1.5. Ejemplos . . . . . . . . . . . . . . . . . . . . . . . . . . 6
1.6. Interpretación Geométrica de los Números Com-
plejos . . . . . . . . . . . . . . . . . . . . . . . . . . . . 10
1.7. Ejemplos . . . . . . . . . . . . . . . . . . . . . . . . . . 12
1.8. Notaciones de un Número Complejo . . . . . . . . . 14
1.9. Extracción de Raı́ces de los Números Complejos . 17
1.10. Ejemplos . . . . . . . . . . . . . . . . . . . . . . . . . . 18
1.11. Conjuntos de Puntos, Cı́rculos y Discos. . . . . . . 30
1.12. Ejemplos . . . . . . . . . . . . . . . . . . . . . . . . . . 32
1.13. Regiones del Plano Complejo. . . . . . . . . . . . . . 35
1.14. Ejercicios Propuestos . . . . . . . . . . . . . . . . . . 35
2 Álgebra de los Números Complejos
1.1. Introducción
Los números complejos son una creación esencialmente algebraica. Cardano
introdujo la unidad imaginaria en 1545 para expresar las soluciones, aunque
fueran imaginarias, de las ecuaciones de segundo grado, y desde este momento
los algebristas encontraron cada vez más evidencias de que los números ima-
ginarios resultantes de admitir al número i como si fuera un número real más
eran suficientes para resolver cualquier ecuación polinomial. Sin embargo, una
prueba de esta conjetura tuvo que esperar hasta el siglo XIX, cuando Gauss
demostró en su tesis doctoral que todo polinomio con coeficientes complejos se
descompone en factores lineales, formulando el teorema fundamental del álgebra.
Otro des-cubrimiento de Gauss mucho más simple, pero no menos importante,
fue que la aritmética de √los números complejos, introducida formalmente a
partir de la relación i = −1, tiene una interpretación geométrica sencilla si
se identifican los factores lineales con los puntos del plano. Esta interpretación
puede considerarse como el punto de partida del estudio analı́tico de los números
complejos. Además de que el análisis complejo es interesante por sı́ mismo,
también hay aplicaciones importantes y algunas veces sorprendentes a problemas
que podrı́amos entender como pertenecientes al dominio real. Esto incluye la
evaluación de integrales reales y sumas de series por técnicas de integración
complejas, ası́ como el uso de cantidades complejas tales como la transformada
de Fourier y la transformada discreta de Fourier para resolver problemas sobre el
movimiento de ondas, propagación de calor y análisis de señales cuyas soluciones
son valores reales. El uso de mapeos conformes entre dominios del plano complejo
es también una técnica significativa para resolver ciertas ecuaciones diferenciales
parciales y problemas relacionados con la ingenierı́a.

1.2. Los Números Complejos y sus Propiedades


Un número complejo se define como un par ordenado de números reales
(x, y) dados por la expresión x + iy y se representa con la letra z. Es decir, todo
número complejo z se escribe como

z = x + iy, x, y ∈ IR, (1.1)


donde i = −1 es la unidad imaginaria y IR representa el campo de los números
reales. A todo número complejo (1.1) le corresponde su complejo conjugado,
1.3 Propiedades de los Números Complejos 3
definido de la siguiente manera

z̄ = x − iy, x, y ∈ IR, (1.2)

Los números x en (1.1) y en (1.2) son la parte real del número complejo z y z̄,
y es su parte imaginaria (y es real, pero como está multiplicada por la unidad
imaginaria i se le conoce como la parte imaginaria). Es decir, una cantidad o un
número complejo se caracterizan por tener una parte real y otra imaginaria. Las
partes real e imaginaria del número complejo z y su conjugado z̄ se representan
de la siguiente manera

Re z = x, Re z̄ = x, Im z = y, e Im z̄ = −y. (1.3)

En ocaciones, el número complejo (1.1) y su correspondiente conjugado (1.2) se


escriben como z = a + ib y z̄ = a − ib, donde a y b son números reales. Esta
notación es muy conveniente para estudiar el álgebra de los números complejos.
Sin embargo, para estudiar el análisis complejo, es más conveniente usar las
notaciones (1.1) y (1.2).
Observación:
Cualquier expresión que contenga una parte real más una parte imaginaria
será una cantidad compleja, independientemente de cómo se escriba. Es decir,
la expresión η + iχ (donde η y χ son reales) es una cantidad compleja y su
conjugado será η − iχ.

1.3. Propiedades de los Números Complejos


Sean z1 = x1 + iy1 y z2 = x2 + iy2 dos números complejos. Entonces, las
operaciones sobre los números complejos se definen por las siguientes reglas:

1. La suma de dos números complejos z1 + z2 es otro número complejo z


dado por

z = z1 + z2 = (x1 + iy1 ) + (x2 + iy2 ) = (x1 + x2 ) + i(y1 + y2 ) (1.4)

donde la parte real del número complejo resultante, z, es la suma (x1 + x2 )


y la parte imaginaria es (y1 + y2 ).
La suma de los números complejos cumple las siguientes leyes:
z1 + z2 = z2 + z1 . Ley conmutativa respecto a la suma
(z1 + z2 ) + z3 = z1 + (z2 + z3 ) = z1 + z2 + z3 . Ley asociativa respecto
a la suma
(z1 + z2 )z3 = z1 z3 + z2 z3 . Ley distributiva respecto a la suma.
4 Álgebra de los Números Complejos
2. La diferencia de dos números complejos z1 y z2 es otro número complejo,
z, dado por

z = z1 − z2 = (x1 + iy1 ) − (x2 + iy2 ) = (x1 − x2 ) + i(y1 − y2 ) (1.5)

donde la parte real del número complejo resultante, z, es la diferencia


(x1 − x2 ) y la parte imaginaria es (y1 − y2 ).
3. El producto de dos números complejos z1 y z2 es otro número complejo, z,
dado por

z = z1 · z2 = (x1 + iy1 )(x2 + iy2 ) = x1 x2 + ix1 y2 + iy1 x2 − y1 y2


= (x1 x2 − y1 y2 ) + i(x1 y2 + y1 x2 ) (1.6)

donde hemos aplicado la propiedad i2 = −1. En este caso la parte real


del número complejo resultante, z, es (x1 x2 − y1 y2 ) y la parte imaginaria
es (x1 y2 + y1 x2 ). La multiplicación de los números complejos satisface las
siguientes leyes:
z1 z2 = z2 z1 . Ley conmutativa respecto a la multiplicación.
(z1 z2 )z3 = z1 (z2 z3 ) = z1 z2 z3 . Ley asociativa respecto a la multiplica-
ción.
z1 (z2 + z3 ) = z1 z2 + z1 z3 . Ley distributiva respecto a la multiplicación.
z1
4. El cociente de dos números complejos z2 , 6 0), es otro número complejo,
(z2 =
z, dado por
z1 z1 z̄2 (x1 + iy1 )(x2 − iy2 )
z = = = =
z2 z2 z̄2 (x2 + iy2 )(x2 − iy2 )
x1 x2 − ix1 y2 + iy1 x2 + y1 y2
=
x22 − ix2 y2 + iy2 x2 + y22
x1 x2 + y1 y2 x2 y1 − x1 y2
= +i (1.7)
x22 + y22 x22 + y22
x1 x2 +y1 y2
En este caso la parte real del número complejo resultante, z, es, x22 +y22
,
x2 y1 −x1 y2
y la parte imaginaria, x22 +y22
.

5. Los números cero y la unidad en el campo de los números complejos


poseen las mismas propiedades que en el campo de los números reales. Para
cualquier número complejo z = x + iy se cumplen las siguientes igualdades:
z+0=z
z·1=z
6. Dos números complejos z1 = x1 + iy1 y z2 = x2 + iy2 , son iguales si y sólo
si sus partes reales e imaginarias son iguales por separado, esto es

z1 = z2 ⇐⇒ x1 = x2 , y1 = y2 (1.8)
1.4 Propiedades de la Conjugación. 5
7. Sea z = x + iy un número complejo y sea z̄ = x − iy su complejo conjugado,
entonces
z + z̄
z + z̄ = x + iy + (x − iy) = 2x → x = Re z = (1.9)
2

8. Sea z = x + iy un número complejo y sea z̄ = x − iy su complejo conjugado,


entonces
z − z̄
z − z̄ = x + iy − (x − iy) = 2iy → y = Im z = (1.10)
2i

9. Sea z = x + iy un número complejo y sea z̄ = x − iy su complejo conjugado,


entonces

z z̄ = (x + iy)(x − iy) = x2 − ixy + iyx + y 2 = x2 + y 2 (1.11)

10. Las potencias naturales de la unidad imaginaria i toman solo cuatro valores
−1, −i, 1, i dadas por las fórmulas

i4n = 1, i4n+1 = i, i4n+2 = −1, i4n+3 = −i (1.12)

donde n = 0, 1, 2, . . . Para elevar a la potencia natural n el número


complejo z = x + iy, se usa la fórmula del binomio de Newton

n(n − 1) n−2
zn = (x + iy)n = xn + nxn−1 (iy) + x (iy)2 +
2!
n(n − 1)(n − 2) n−3
+ x (iy)3 + . . . + (iy)n , (1.13)
3!
en esta expresión se deben tomar en cuenta las propiedades de la unidad
imaginaria i dadas en (1.12).

1.4. Propiedades de la Conjugación.


Sean z y w dos números complejos distintos de cero. Entonces

1. z̄¯ = z

2. z̄ + w̄ = z + w

3. z ± w = z̄ ± w̄

4. zw = z̄ w̄

5. |z| = |z̄|
6 Álgebra de los Números Complejos
1.5. Ejemplos

Ejemplo 1.5.1. Sea z = 3 − i. Evaluar z + z̄, z − z̄ y z z̄

√ el número complejo z = 3 − i su correspondiente complejo
Solución: Dado
conjugado es z̄ = 3 + i. Entonces
√ √ √
z + z̄ = ( 3 − i) + ( 3 + i) = 2 3 (1.14)
√ √
z − z̄ = ( 3 − i) − ( 3 + i) = −2i (1.15)
√ √ √ √ 2
z z̄ = ( 3 − i)( 3 + i) = 3 + i 3 − i 3 − i = 3 + 1 = 4 (1.16)

Ejemplo 1.5.2. Sean z1 = 3+2i y z2 = −6+8i dos números complejos. Evaluar


z1 + z2 , z1 − z2 y z1 · z2 .
Solución: Tenemos

z1 + z2 = (3 + 2i) + (−6 + 8i) = −3 + 10i (1.17)


z1 − z2 = (3 + 2i) − (−6 + 8i) = 9 − 6i (1.18)
z1 · z2 = (3 + 2i) · (−6 + 8i) = −34 + 12i (1.19)
z1
Ejemplo 1.5.3. Sean z1 = 1 + 2i y z2 = 3 − 4i. Evaluar z2 .

Solución: Tenemos
z1 1 + 2i (1 + 2i)(3 + 4i) 3 + 4i + 6i − 8 1 2
= = = =− + i (1.20)
z2 3 − 4i (3 − 4i)(3 + 4i) 9 + 16 5 5
1 1
Ejemplo 1.5.4. Sean z1 = 3 + i, z2 = 3 − i y z3 = 5 + 10 i. Evaluar el producto
z1 · z2 · z3 .
Solución: Tenemos
1 1  1 1 
z1 · z2 · z3 = (3 + i)(3 − i) + i = 10 + i =2+i (1.21)
5 10 5 10
z1 ·z2
Ejemplo 1.5.5. Sean z1 = 3 + 5i, z2 = 2 + 3i y z3 = 1 + 2i. Evaluar z3 + z2 .
Solución: Primero evaluamos la expresión

z1 · z2 = (3 + 5i)(2 + 3i) = 6 + 9i + 10i − 15 = −9 + 19i (1.22)

Luego, tenemos
z1 · z2 −9 + 19i (−9 + 19i)(1 − 2i) −9 + 18i + 19i + 38
= = =
z3 1 + 2i (1 + 2i)(1 − 2i) 1+4
29 37
= + i (1.23)
5 5
Finalmente
z1 · z2  29 37   29   37  39 52
+ z2 = + i + (2 + 3i) = +2 + +3 i = + i (1.24)
z3 5 5 5 5 5 5
1.5 Ejemplos 7
(2−3i)(3−2i)
Ejemplo 1.5.6. Evaluar la expresión (4−3i)(5−4i) .

Solución: Tenemos
(2 − 3i)(3 − 2i) 6 − 4i − 9i − 6 −13i
= = =
(4 − 3i)(5 − 4i) 20 − 16i − 15i − 12 8 − 31i
(−13i)(8 + 31i) 403 104
= = − i (1.25)
(8 − 31i)(8 + 31i) 1025 1025

(1+i)(− 3+i)
Ejemplo 1.5.7. Evaluar la expresión √
(1−i)( 3+i)
.

Solución: Tenemos
√ √ √ √ √
(1 + i)(− 3 + i) − 3 + i − 3i − 1 −( 3 + 1) + (1 − 3)i
√ = √ √ = √ √ =
(1 − i)( 3 + i) 3 + i − 3i + 1 ( 3 + 1) + (1 − 3)i
−a + bi (−a + bi)(a − bi)
= = =
a + bi (a + bi)(a − bi)
−a2 + abi + abi + b2 −a2 + b2 2ab
= = + 2 i (1.26)
a2 + b2 a2 + b2 a + b2
√ √ √
donde definimos
√ a = 3 + 1 y b = 1 − 3. Entonces, tenemos a2 = 4 + 2 3,
b2 = 4 − 2 3, a2 + b2 = 8 y sustituyendo estos valores en (1.26)
√ √ √
−a2 + b2 3 2ab ( 3 + 1)(1 − 3) 4 1
2 2
=− , 2 2
=2 = − = − . (1.27)
a +b 2 a +b 8 8 2
el resultado final es √ √
(1 + i)(− 3 + i) 3 1
√ =− − i (1.28)
(1 − i)( 3 + i) 2 2
Ejemplo 1.5.8. Elevar a la cuarta potencia el número complejo z = 2 + i.
Solución: Dado el número complejo z = 2 + i se necesita calcular z 4 , para
esto usamos la fórmula del binomio de Newton (1.13)

z 4 = (2 + i)4 = 24 + 4 · 23 i + 6 · 4i2 + 4 · 2i3 + i4 = 16 + 32i − 24 − 8i + 1 = −7 + 24i


(1.29)
Ejemplo 1.5.9. Evaluar (17 − 6i)(−4 − 12i).
Solución:
Si definimos z = −4 − 12i, entonces su conjugado z̄ = −4 − 12i = −4 + 12i.
Ahora calculamos el producto

(17 − 6i)(−4 − 12i) = (17 − 6i)(−4 + 12i) = 4 + 228i (1.30)


Ejemplo 1.5.10. Calcular la parte real e imaginaria de la función compleja

f (z) = , donde z = x + iy (1.31)
z2 + 1
8 Álgebra de los Números Complejos
Ejemplo 1.5.11. Evaluar la expresión i3 − 4i2 + 2.
Solución: Usando la propiedad i2 = i · i = −1, se tiene i3 = i · i · i = −i,
entonces

i3 − 4i2 + 2 = −i + 4 + 2 = 6 − i (1.32)
 2
Ejemplo 1.5.12. Evaluar la expresión 2−3i
1+2i .
Solución: Tenemos
2
(2 − 3i)2

2 − 3i 4 − 12i + 9 13 − 12i
= = = = (1.33)
1 + 2i (1 + 2i)2 1 + 4i − 4 −3 + 4i
(13 − 12i)(−3 − 4i) −39 − 52i + 36i − 48 87 16
= = =− − i
(−3 + 4i)(−3 − 4i) 9 + 16 25 25
√ √
Ejemplo 1.5.13. Sean z1 = 1 + i, z2 = 3 + i y z3 = 1 + i 3. Evaluar z2z·z
1
3
.
Solución: Primero evaluamos el producto z2 · z3 , obtenemos
√ √ √ √
z2 · z3 = ( 3 + i)(1 + i 3) = 3 + 3i + i − 3 = 4i (1.34)
Finalmente, se tiene
z1 1+i (1 + i)(−4i) −4i + 4 1 1
= = = = − i (1.35)
z2 · z3 4i (4i)(−4i) 16 4 4
Ejemplo 1.5.14. Hallar las soluciones reales de (2x − iy) + x + 2iy = i.
Solución: La expresión dada la podemos escribir de la siguiente manera
2x − iy + x + 2iy = i → 3x + iy = i (1.36)
Igualando los terminos reales e imaginarios de la izquierda con los términos
reales e imaginarios de la derecha, tenemos el sistema de ecuaciones
3x = 0
y = 1 (1.37)
De donde podemos concluir que las soluciones reales son x = 0 y y = 1.
Ejemplo 1.5.15. Hallar las soluciones reales de la ecuación (3x − i)(2 + i) +
(x − iy)(1 + 2i) = 5 + 6i.
Solución: Realizando las multiplicaciones correspondientes, obtenemos
7x + 2y + (5x − y)i = 4 + 8i. (1.38)
Igualando los términos reales e imaginarios de la izquierda con los términos
reales e imaginarios de la derecha, obtenemos el siguiente sistema de ecuaciones
7x + 2y = 4
5x − y = 8 (1.39)
Resolviendo este sistema de dos ecuaciones con dos incógnitas, resulta x = 20/17
e y = −36/17.
1.5 Ejemplos 9
Ejemplo 1.5.16. Hallar las soluciones reales de la ecuación (x−iy)(a−ib) = i5 ,
donde a y b son ciertos números reales y |a| =
6 |b|.

Solución: Abriendo los paréntesis de la expresión dada, tenemos

ax − ibx − iay − by = i (1.40)

Igualando los términos resulta el sistema de ecuaciones

ax − by = 0 (1.41)
bx + ay = −1 (1.42)

Este sistema tiene las siguientes soluciones reales


a
y = − (1.43)
a 2 + b2
b
x = − 2 (1.44)
a + b2
Ejemplo 1.5.17. Calcular los números complejos z tales que
2z − i
f (z) = (1.45)
2 + iz
a) sea un número real, b) sea un número complejo

Solución: a) Sea z = x + iy, entonces

2z − i 3x + i(−2x2 − 2y 2 + 5y − 2)
f (z) = = (1.46)
2 + iz (2 − y)2 + x2

Entonces, para que la función f (z) sea real, se debe cumplir la relación
 
2 2 2 5 9
−2x − 2y + 5y − 2 = 0, → x + y− = (1.47)
4 16

Es decir, z está en la circunferencia con centro en (0, 5/4) y de radio R = 3/4.


b) f (z) es imaginaria si y sólo si x = 0, es decir, z está en el eje imaginario.

Ejemplo 1.5.18. Calcular los números complejos z tales que


z−1−i
f (z) = (1.48)
z+1+i
a) f (z) sea real, b) f (z) tenga módulo igual a la unidad.

Solución: Sea z = x + iy, entonces

z−1−i x2 + y 2 − 2 + i(2y − 2x)


f (z) = = (1.49)
z+1+i (x + 1)2 + (y + 1)2
10 Álgebra de los Números Complejos
a), f (z) es real si y sólo si y = x. Es decir, z, está en la bisectriz de los cuadrantes
primero y tercero.
b) la función f (z) tendrá módulo igual a la unidad |f (z)| = 1 si y sólo si

|z − 1 − i| = |z + 1 + i| (1.50)

De (1.50), tenemos

(x − 1)2 + (y − 1)2 = (x + 1)2 + (y + 1)2 ⇐⇒ x + y = 0 (1.51)

Es decir, z, está en la bisectriz de los cuadrantes segundo y cuarto.

1.6. Interpretación Geométrica de los Números


Complejos
En esta sección analizaremos la interpretación geométrica de los números
complejos. La idea es simple y natural. Supongamos un plano con un sistema
de coordenadas cartesianas xOy Figura 1.1. Por definición un número complejo
z = x + iy es un par ordenado de números reales (x, y). Por otro lado, a cada
par ordenado (x, y) de números reales le corresponde un punto en el plano.
Entonces, a cada punto (x, y) del plano se le puede ver como una representación
del número complejo z = x + iy. En tal caso, al plano se le llama plano complejo
y al número complejo z se le llama punto de este plano.
Sobre el eje Ox se escriben los números reales z = x + 0i = x, por esto se
le conoce como eje real . Sobre el eje Oy se escriben los números imaginarios
z = 0 + iy = iy, y el eje se llama eje imaginario. Es muy útil la interpretación del
y y
Plano complejo z Plano complejo z Plano polar complejo

z0 = (x0 , y0 ) z0 = x0 + iy0 z0 = ρ0 eiϕ0


iy0
ρ0
iy0
ϕ0
x x Re
x0 x0
(a) (b) (c)

Figura 1.1: Representación geométrica de los números complejos.

−−→
número complejo z = x + iy como radio–vector OP Figura 1.1. Es claro que a
cada radio–vector del plano con terminación en el punto P (x, y) le corresponde
el número complejo z = x + iy. Al vector nulo le corresponde el número complejo
z = 0 + i0 = 0.
1.6 Interpretación Geométrica de los Números Complejos 11
−−→
La longitud ρ del segmento OP se llama módulo del número complejo y se
representa por |z|, es decir
ρ = |z|. (1.52)
Geométricamente, |z| es la distancia del origen (0, 0) al punto P (x, y). Entonces,
la posición de un punto z = x + iy en el plano, además de sus coordenadas
cartesianas (x, y), se puede representar también en coordenadas polares (ρ, ϕ).
En tal caso se tienen las relaciones

x = ρ cos ϕ = |z| cos ϕ, y = ρ sen ϕ = |z| sen ϕ. (1.53)


−−→
El ángulo ϕ entre el vector OP y la abcisa x se llama argumento de z y se
representa como
ϕ = arg z (1.54)
El argumento de z puede ser positivo o negativo, dependiendo de cómo se tome
la dirección. El argumento se considera positivo si la dirección de ϕ es en contra
de las manecillas del reloj y negativo en caso contrario. Para un punto z dado,
su módulo se define de manera única, y su argumento con exactitud de hasta
el término 2kπ, (k = 0, ±1, ±2, . . .). El valor del argumento ϕ que satisface
la desigualdad −π < ϕ ≤ π se llama valor principal del argumento de z y se
representa por (1.54). De tal manera que para los otros valores del argumento
de z tendremos la igualdad

Arg z = arg z + 2kπ. k = 0, ±1, ±2, . . . , (1.55)

donde arg z es el valor principal del argumento Arg z. El valor principal o el


argumento principal se puede calcular por las siguientes fórmulas:

y

arctan x

 si x > 0
y

π + arctan x si x < 0 y y ≥ 0



arg z = −π + arctan xy

si x < 0 y y < 0, (1.56)
π

si x = 0 y y > 0,



 2
− π

si x = 0 y y < 0
2

El punto z = 0 es el único punto del plano complejo, para el cual el argumento


no está definido. Las siguientes fórmulas también son válidas:
y y
tan(Arg z) = , sen(Arg z) = p ,
x x2 + y2
x
cos(Arg z) = p (1.57)
x2 + y 2
12 Álgebra de los Números Complejos
Dos números complejos z1 y z2 son iguales si y sólo si sus módulos son iguales y
sus argumentos son iguales o difieren por un múltiplo de 2π.

|z1 | = |z2 |, Arg z1 = Arg z2 + 2πk, (k = 0, ±1, ±2, . . .) (1.58)

Elevando al cuadrado las expresiones (1.53) y sumándolas, se tiene

x2 + y 2 = ρ2 cos2 ϕ + ρ2 sen2 ϕ = ρ2 (cos2 ϕ + sen2 ϕ) = ρ2 (1.59)

donde hemos usado la identidad trigonométrica cos2 ϕ + sen2 ϕ = 1. Es decir, la


longitud del radio–vector z = x + iy está dada por la expresión

ρ2 = |z|2 = x2 + y 2 (1.60)

la cual refleja el teorema de Pitágoras: el cuadrado de la hipotenusa ρ es igual a


la suma de los cuadrados de los catetos x e y.
Sea z un número complejo diferente de cero y sea z̄ su complejo conjugado.
Entonces el producto de un número complejo por su complejo conjugado es

z z̄ = (x + iy)(x − iy) = x2 − ixy + ixy + y 2 = x2 + y 2 (1.61)

Por consiguiente, igualando las expresiones (1.60) y (1.61) obtenemos la relación

ρ2 = |z|2 = z z̄ = x2 + y 2 . (1.62)

Entonces, la longitud de un número complejo z = x + iy puede calcularse por


las expresiones dadas en (1.62).

1.7. Ejemplos
Ejemplo 1.7.1. Hallar el módulo (longitud), el argumento y las partes real e
imaginaria del número complejo z = (1 + i)3
Solución: Por definición, el módulo ó longitud de un número complejo z es
√ 3
3
|z| = (1 + i)3 = |(1 + i)| = 2 = 23/2

(1.63)

También podemos calcular el módulo de la siguiente manera



|z| = (1 + i)3 = |1 + 3i − 3 − i| = |−2 + 2i| = 2 2 = 23/2

(1.64)

El resultado es el mismo. El argumento de z es


π 3π
ϕ = arg z = arg(1 + i)3 = arg(−2 + 2i) = π + arctan(−1) = π − = (1.65)
4 4
Donde, hemos identificado x = −2 < 0, y = 2 > 0 y usamos la fórmula (1.8).
1.7 Ejemplos 13
 
1−i
Ejemplo 1.7.2. Hallar el módulo y el argumento del número complejo 1+i .

Solución: El módulo es

1 − i (1 − i)(1 − i)
1 + i (1 + i)(1 − i) = | − i| = 1
= (1.66)

Su argumento  
1−i π
ϕ = arg = arg(−i) = − (1.67)
1+i 2
Ejemplo 1.7.3. Calcular el módulo
(2 + i√5)(1 + i√3)3

√ √ (1.68)


5+i 3

Solución: Escribiendo la expresión (1.68) como

(2 + i√5)(1 + i√3)3 2 + i√5 1 + i√3 3 √



4 + 5(1 + 3)3/2
√ √ = √ √ = √ =


5+i 3 5 + i 3 5+3
√ √
9 · 64 √ √ √
r
9 64
= √ = = 72 = 36 · 2 = 6 2 (1.69)
8 8

Ejemplo 1.7.4. Verificar la igualdad



a + bi
b + ai = 1 (1.70)

Solución: Simplificando, se tiene

a + bi (a + bi)(b − ai) 2ab b2 − a2


= = 2 2
+ 2 i (1.71)
b + ai (b + ai)(b − ai) a +b a + b2

Por consiguiente
s 2 b2 4 − 2a2 b2 + a4
s
a + bi 4a b a4 + 2a2 b2 + b2
b + ai = (a2 + b2 )2 + = =1 (1.72)

2 2
(a + b ) 2 (a2 + b2 )2

La relación (1.70) queda demostrada.

Ejemplo 1.7.5. Hallar la curva que describe la ecuación

|z − z0 | = R con R real (1.73)

Solución: Sean z = x + iy y z0 = x0 + iy0 , entonces

z − z0 = (x − x0 ) + i(y − y0 ) (1.74)
14 Álgebra de los Números Complejos
p
El módulo ó longitud esta dado por |z − z0 | = (x − x0 )2 + (y − y0 )2 . Entonces,
la expresión (1.73) se escribe de la siguiente manera
p
(x − x0 )2 + (y − y0 )2 = R (1.75)
Elevando al cuadrado ambas partes de (1.75) obtenemos la siguiente ecuación
(x − x0 )2 + (y − y0 )2 = R2 (1.76)
la cual representa un cı́rculo de radio R centrado en el punto (x0 , y0 ). Por
consiguiente, la expresión (1.73) representa la ecuación de un cı́rculo de radio
R centrado en el punto z0 del plano complejo. Si x0 = 0 y y0 = 0, entonces, de
(1.76) resulta la ecuación
x2 + y 2 = R 2 (1.77)
la cual representa un cı́rculo de radio R centrado en el origen. En el plano
complejo este cı́rculo corresponde a la ecuación
|z| = R (1.78)
ya que x0 = 0, y0 = 0, entonces z0 = 0.

1.8. Notaciones de un Número Complejo


Para resolver algunos problemas de aplicación, es conveniente conocer las
diferentes formas en que se pueden denotar los números; mientras una notación
es más adecuada para realizar ciertas operaciones aritmética otra lo será para
otro tipo de operaciones, como se verá más adelante. Las diferentes formas de
denotar un número complejo tienen su origen en los sistemas de coordenadas
cartesianas rectangulares y el sistema de coordenadas polares aun cuando el fin
primario de estos sistemas de coordenas fue otro.
Notación algebraica: Sabemos que un número complejo z se escribe, por
definición, de la siguiente manera
z = x + iy (1.79)
A esta representación se le conoce como representación algebraica .
Representación Trigonométrica: Si hacemos uso de las expresiones (1.53),
podemos escribir el número complejo z como

z = ρ cos ϕ + iρ sen ϕ = ρ(cos ϕ + i sen ϕ), (1.80)

p
donde ρ = |z| = x2 + y 2 y ϕ = Arg (z) = arg z + 2πk. A esta representación
se le conoce como representación trigonométrica ó forma polar .
1.8 Notaciones de un Número Complejo 15
Sean z1 = ρ1 (cos ϕ1 + i sen ϕ1 ) y z2 = ρ2 (cos ϕ2 + i sen ϕ2 ), entonces la
multiplicación esta dada por

z1 · z2 =
= ρ1 ρ2 [(cos ϕ1 cos ϕ2 − i sen ϕ1 sen ϕ2 ) + i(sen ϕ1 cos ϕ2 + cos ϕ1 sen ϕ2 )]
= ρ1 ρ2 [cos(ϕ1 + ϕ2 ) + i sen (ϕ1 + ϕ2 )] (1.81)

Es decir, los argumentos de los números complejos son aditivos respecto a la


mutiplicación. Entonces, si z1 = z2 = z y ϕ1 = ϕ2 = ϕ, de (1.81), resulta

z 2 = ρ2 (cos 2ϕ + i sen 2ϕ) (1.82)

En general, se tiene
n
z n = [ρ(cos ϕ + i sen ϕ)] = ρn (cos nϕ + i sen nϕ) (1.83)

La expresión (1.83) se conoce como fórmula de Moivre. Esta fórmula nos dice
que; al elevar un número complejo a una cierta potencia entera y positiva n, el
módulo de este número se eleva a la misma potencia y el argumeto se multiplica
por el exponente de la potencia.
Si ρ = 1, de la expresión (1.83) se tiene un caso particular
n
(cos ϕ + i sen ϕ) = cos nϕ + i sen nϕ (1.84)

Una aplicación inmediata de esta expresión es para obtener relaciones trigo-


nométricas. Por ejemplo, suponiendo que n = 2, entonces
2
(cos ϕ + i sen ϕ) = cos 2ϕ + i sen 2ϕ (1.85)

Esta expresión se puede escribir como

cos2 ϕ + 2i cos ϕ sen ϕ − sen2 ϕ = cos 2ϕ + i sen 2ϕ (1.86)

Igualando la parte real e imaginaria resultan las expresiones

cos 2ϕ = cos2 ϕ − sen2 ϕ


sen 2ϕ = 2 cos ϕ sen ϕ (1.87)

Si n = 3, tendremos
3
(cos ϕ + i sen ϕ) = cos 3ϕ + i sen 3ϕ (1.88)
Luego, desarrollando el trinomio, se tiene

cos3 ϕ + 3i cos2 ϕ sen ϕ + 3 cos ϕ(i sen ϕ)2 + (i sen ϕ)3 = cos 3ϕ + i sen 3ϕ (1.89)

Igualando la parte real e imaginaria, obtenemos las fórmulas

cos 3ϕ = cos3 ϕ − 3 cos ϕ sen2 ϕ


sen 3ϕ = 3 cos2 ϕ sen ϕ − sen3 ϕ. (1.90)
16 Álgebra de los Números Complejos
Observación:
Si un número complejo está dado en su forma algebraica z = x + iy, entonces
para elevarlo a una cierta potencia entera y positiva n usando la fórmula
de Moivre (1.83), debemos, primero escribirlo en su correspondiente forma
trigonométrica.

Teorema 1.8.1. El módulo del producto de números complejos es igual al


producto de sus módulos y el argumento del producto de números complejos es
igual a la suma sus argumentos.
De la expresión (1.81), tenemos

|z1 · z2 | = |ρ1 · ρ2 cos(ϕ1 + ϕ2 ) + iρ1 · ρ2 sen (ϕ1 + ϕ2 )| =


q
= ρ21 · ρ22 [cos2 (ϕ1 + ϕ2 ) + sen2 (ϕ1 + ϕ2 )] =
= ρ1 · ρ2 = |z1 | · |z2 | (1.91)

y
Arg (z1 · z2 ) = ϕ1 + ϕ2 = Arg z1 + Arg z2 (1.92)
Observación: Como consecuencia, tenemos que el módulo de una potencia
entera positiva de un número complejo es igual al módulo elevado a la misma
potencia y el argumento de un número complejo elevado a una potencia entera
positiva es igual al argumento del número dado multiplicado por el valor de la
potencia.
|z n | = |z|n , Arg (z n ) = nArg z, n≥1 (1.93)
La división de dos números complejos z1 y z2 , z2 6= 0, en su forma trigonométrica
está definida como el número complejo z dado por
z1 z1 z̄2 ρ1 (cos ϕ1 + i sen ϕ1 )ρ2 (cos ϕ2 − i sen ϕ2 )
z = = =
z2 z2 z̄2 ρ2 (cos ϕ2 + i sen ϕ2 )ρ2 (cos ϕ2 − i sen ϕ2 )
ρ1
= [cos(ϕ1 − ϕ2 ) + i sen(ϕ1 − ϕ2 )] (1.94)
ρ2
De donde
z1 ρ1
= |z1 |
z2 ρ2 = |z2 | (1.95)

y los argumentos
 
z1
Arg = ϕ1 − ϕ2 = Arg z1 − Arg z2 (1.96)
z2
Representación Exponencial: Haciendo uso de la fórmula de Euler

eiϕ = cos ϕ + i sen ϕ, 0 ≤ ϕ ≤ 2π (1.97)

y comparando con la expresión (1.80), tenemos que todo número complejo z se


puede escribir como
1.9 Extracción de Raı́ces de los Números Complejos 17

z = ρeiϕ (1.98)

p
donde ρ = |z| = x2 + y 2 y ϕ = Arg (z) = arg z + 2πk. A esta representación
se le conoce como representación exponencial del número complejo z.
Haciendo uso de la fórmula de Moivre (1.83), podemos escribir

z n = (ρeiϕ )n = ρn einϕ (1.99)

Sean z1 y z2 dos números complejos dados en forma exponencial. Entonces,


la multiplicación de z1 = ρ1 eiϕ1 por z2 = ρ2 eiϕ2 será otro número complejo z
representado por

z = z1 z2 = ρ1 ρ2 ei(ϕ1 +ϕ2 ) = ρeiϕ donde ρ = ρ1 ρ2 y ϕ = ϕ1 + ϕ2 (1.100)

El cociente de z1 y z2 será el número complejo dado por


z1 ρ1 eiϕ1 ρ1 ρ1
z= = = ei(ϕ1 −ϕ2 ) = ρeiϕ donde ρ = y ϕ = ϕ1 − ϕ2 (1.101)
z2 ρ2 eiϕ2 ρ2 ρ2
Se puede ver claramente que la representación exponencial de los números
complejos es más elegante que la forma normal e incluso que su representación
trigonométrica.

1.9. Extracción de Raı́ces de los Números


Complejos
En esta sección analizaremos el problema de extracción de raı́ces de una
potencia natural n de un número complejo z arbitrario (en particular, real).
Además, buscaremos todos los posibles valores de la raı́z, reales y complejos.
Para resolver este problema en forma general, utilzaremos la representación
trigonométrica del número complejo z 6= 0

z = ρ [cos(ϕ + 2kπ) + i sen(ϕ + 2kπ)] (1.102)

donde ϕ es uno de los valores del argumento (0 ≤ ϕ < 2π), y ϕ + 2kπ, (donde
k = 0, ±1, ±2, . . .) nos da el conjunto de todos los valores del argumento de z.
El número w se llama raı́z de la potencia n de z, si

wn = z (1.103)

Escribamos en forma trigonométrica el número desconocido w, es decir

w = r(cos θ + i sen θ) (1.104)


18 Álgebra de los Números Complejos
Entonces, aplicando la fórmula de Moivre (1.83) y reescribiendo la igualdad
wn = z, tenemos

rn [cos(nθ) + i sen(nθ)] = ρ [cos(ϕ + 2kπ) + i sen(ϕ + 2kπ)] (1.105)

De las condiciones de igualdad, obtenemos dos relaciones

rn = ρ, nθ = ϕ + 2kπ (1.106)

La primera de las expresiones muestra que

r = ρ1/n (1.107)

Debido a que r > 0 y ρ > 0. La segunda igualdad en (1.106) representa el


hecho de que el argumento nθ del número wn es igual a uno de los valores del
argumento del número z. De esta igualdad encontramos
ϕ + 2kπ
θ= (1.108)
n
la cual nos dice que para diferentes valores de k se obtienen, en general, diferentes
valores de la raı́z w. Representemos el valor de θ que le corresponde a cada
número k elejido por θk ;
ϕ + 2kπ
θk = (1.109)
n
donde k toma los valores 0, 1, 2, . . . n−1. En tal caso, obtendremos θ0 , θ1 , . . . θn−1
y por consiguiente n valores de la raı́z
    
√ ϕ + 2kπ ϕ + 2kπ
wn = ρ cos
n
+ i sen (1.110)
n n

donde k = 0, 1, 2, . . . n−1 y ϕ = arg z. Estos n valores están en una circunferencia



de radio n ρ con centro en el orı́gen de coordenadas y constituyen los vértices
de un polı́gono regular de n lados.

1.10. Ejemplos
Ejemplo 1.10.1. Escribir en la forma trigonométrica los siguientes números:
A) 6 − 6i, B) 3i y C) −10.
Solución: A) Tenemos 6 − 6i. Entonces, x = 6 y y = −6. Por consiguiente,
la longitud o módulo es
p p √
ρ = 62 + (−6)2 = (36)2 = 6 2 (1.111)

Luego, de (1.53), tenemos


√ √
x 6 2 y 6 2
cos ϕ = = √ = , sen ϕ = = − √ = − (1.112)
ρ 6 2 2 ρ 6 2 2
1.10 Ejemplos 19
De donde obtenemos el valor del argumento ϕ = 7π/4. Por lo tanto, la represen-
tación trigonométrica de 6 − 6i es

    
7π 7π
6 − 6i = 6 2 cos + i sen (1.113)
4 4

Solución:
p B) Tenemos
√ 3i. En este caso x = 0 y y = 3. Entonces, el módulo
es ρ = x2 + y 2 = 32 = 3. Usando las fórmulas (1.53), tenemos

0 3
cos ϕ = = 0, sen ϕ = =1 (1.114)
ρ 3

El valor del argumento es ϕ = π/2. Entonces la representación trigonométrica


del número 3i está dada por
h π  π i
3i = 3 cos + i sen (1.115)
2 2
Solución: C) Se tiene −10. En p este caso x = −10 y y = 0. Entonces, el
módulo o longitud de −10 es ρ = (−10)2 + 02 = 10. De las fórmulas (1.53), se
tiene
−10 0
cos ϕ = = −1, sen ϕ = =0 (1.116)
10 10
Entonces el valor del argumento ϕ = π. Por lo tanto el número −10 tiene como
representación trigonométrica

−10 = 10 (cos π + i sen π) (1.117)

Ejemplo 1.10.2. Sea dado el número complejo z = 3 + 2i. Representarlo en su


forma trigonométrica.

Solución:
p Identificamos
p x = 3 y y√
= 2. El módulo del número complejo es
ρ = |z| = x2 + y 2 = (3)2 + (2)2 = 13. Ahora podemos calcular el valor del
ángulo ϕ, o bien el argumento ϕ = arg z
 
y 2 −1 2
tan ϕ = = , → ϕ = tan (1.118)
x 3 3

Entonces, la representación trigonométrica es



      
2 2
z = 13 cos tan−1 + i sen tan−1 (1.119)
3 3

Ejemplo 1.10.3. Sea z = 2 3 − 3i un número complejo. Escribirlo en su forma
trigonométrica.

Solución: Identificamos x = 2 3 y y = 3. Entonces
p √ √
ρ = |z| = x2 + y 2 = 12 + 9 = 21 (1.120)
20 Álgebra de los Números Complejos
Luego
y √ !
3
ϕ = arg z = tan−1 = tan−1 (1.121)
x 2

Finalmente, la representación trigonométrica de z = 2 3 − 3i es
( " √ !# " √ !#)
√ −1 3 −1 3
z = 21 cos tan + i sen tan (1.122)
2 2

Ejemplo 1.10.4. Representar los siguientes números complejos A) z = 2 + 3i,


B) z = 1 − i en forma trigonométrica.

Solución: A) z = 2 + 3i. Identificamos x = 2 y y = 3. Entonces, el módulo


es p p √
ρ = |z| = x2 + y 2 = (2)2 + (3)2 = 13 (1.123)
y el argumento  
−1 3
ϕ = tan (1.124)
2
Usando las fórmulas (1.55) y (1.57), obtenemos la representación polar de
z = 2 + 3i

      
−1 3 −1 3
z = 13 cos tan + sen tan (1.125)
2 2
Solución: B) z = 1 − i. Identificamos x = 1 y y = −1.
p p √
ρ = |z| = x2 + y 2 = (1)2 + (−1)2 = 2 (1.126)

y el argumento  
−1 −1
ϕ = tan = tan−1 (−1) (1.127)
1
Entonces, usando la fórmula (1.55), tenemos la representación polar de z = 1 − i
√ 
2 cos tan−1 (−1) + sen tan−1 (−1)
   
z= (1.128)

Ejemplo 1.10.5. Sean z1 = 21/3 (cos 25o + i sen 25o ) y z2 = 41/3 (cos 35o +
i sen 35o ). Calcular el producto z1 · z2 .

Solución: Usando la fórmula (1.81), el producto esta dado por

z1 · z2 = (23 )1/3 [cos(25o + 35o ) + i sen(25o + 35o )] = 2 (cos 60o + i sen 60o )

= 1+i 3 (1.129)

Ejemplo 1.10.6. Sean z1 = 6[cos (110o ) + i sen (110o )] y z2 = 2[cos (80o ) +


i sen (80o )]. Calcular zz21 .
1.10 Ejemplos 21
Solución: De la fórmula (1.94), tenemos
z1 6
= [cos (110o − 80o ) + i sen (110o − 80o )] = 3 [cos (30o ) + i sen (30o )]
z2 2
3 √
= ( 3 + i) (1.130)
2
Ejemplo 1.10.7. Sea z = 3(cos 43o + i sen 43o ). Se pide elevar el número dado
a la quinta potencia, es decir, calcular z 5 .
Solución: El número está en forma trigonométrica. Podemos usar la fórmula
de Moivre (1.83)
5
z 5 = [3(cos 43o + i sen 43o ] = 243 (cos 215o + i sen 215o ) (1.131)

Ejemplo 1.10.8. Hallar el valor principal


√ ϕ = arg(z) de los siguientes números
complejos; A) z = 1+−2
√ , B) z = ( 3 − i)6 .
3i

Solución: A) El numerador −2 lo podemos escribir como z1 =√ −2 + 0i,


donde identificamos
√ x = −2 y y = 0 y al denominador como z2 = 1 + 3i donde
x = 1 y y = 3. Entonces
p p √
ρ1 = |z1 | = x2 + y 2 = (−2)2 + (0)2 = 4 = 2 (1.132)

Y  
−1 0
ϕ1 = tan =π (1.133)
−2
Entonces, tenemos √
4 [cos π + i sen π] = −2
z1 = (1.134)

Para el Denominador z2 = 1 + 3i, tenemos
p q √ √
ρ2 = |z2 | = x2 + y 2 = (1)2 + ( 3)2 = 4 = 2 (1.135)

Y √  π
ϕ2 = tan−1 3 = (1.136)
3
Entonces h π  π i
z2 = 2 cos + sen (1.137)
3 3
Finalmente, tenemos
−2 z cos π + i sen π
z= √ = 1 = (1.138)
cos π3 + i sen π3
 
1 + 3i z2

Usando la fórmula

arg (z) = arg (z1 ) − arg (z2 )


π 2π
arg (z) = π− = (1.139)
3 3
22 Álgebra de los Números Complejos
√ √
B) Para z = ( 3 − i)6 , sea z1 = 3 − i, entonces
p q√ √
ρ = |z| = x2 + y 2 = ( 3)2 + (−1)2 = 4 = 2 (1.140)

Y 
−1 π
ϕ = tan−1√ =− (1.141)
3 6
    
−π −π
z1 = 2 cos + sen (1.142)
6 6
Escribiendo este resultado en forma exponencial
 iπ
6
z1 = (ρeiφ )6 = 2e− 6 = 64e−πi (1.143)

Entonces, los argumentos

arg z = −π, → Arg z = −π + 2kπ (1.144)

Ejemplo 1.10.9. Usando la representación


√ √ exponencial
√ de los números comple-
jos, verificar la igualdad i(1 − 3i)( 3 + i) = 2(1 + 3i).
√ √ √
Solución: Podemos ver la igualdad i(1 − 3i)( 3 + i) √= 2(1 + 3i), √ como
el producto de tres números complejos z1 = i, z2 = (1 − 3i) y z3 = ( 3 + i).
Escribiendo estos tres números en forma exponencial. Para z1 , tenemos

z1 = i = 0 + i (1.145)

Su magnitud es
p p √
ρ1 = |z| = x2 + y 2 = (0)2 + (1)2 = 1 = 1 (1.146)

y su argumento  
1 π
ϕ1 = tg−1 = (1.147)
0 2
Entonces, de la forma exponencial de un número complejo z = ρeiϕ tenemos
π
z1 = e 2 i (1.148)

Para √
z2 = 1 − i 3 (1.149)
q √  √
p 2 2
2 2
tenemos ρ = |z| = x + y = 3 + (1) = 4 = 2
 
1 π
ϕ2 = tan−1 √ =− (1.150)
− 3 3
En forma exponencial
π
z2 = (ρeiφ ) = 2e−i 3 (1.151)
1.10 Ejemplos 23

Para: z3 = ( 3 + i)

r 
p √ 2 2

ρ3 = |z| = x2 + y 2 = 3 + (1) = 4 = 2,
 
−1 1 π
ϕ3 = tan √ = . (1.152)
3 6
Tenemos
π
z3 = 2e 6 i (1.153)
Entonces
π π π π π π iπ
z1 · z2 · z3 = e 2 i · (2)e− 6 i · (2)e 6 i = 4e( 2 − 3 + 6 )i = 4e 3
h π  π i √
= 4 cos + i sen = 2(1 + i 3) (1.154)
3 3
Ejemplo 1.10.10. Verificar las igualdades; A) |eiϕ | = 1, B) (eiϕ )2 = (e2iϕ )
Solución: A) Representando eiϕ como el número complejo z, es decir z = eiϕ .
Entonces, su conjugado es z̄ = e−iϕ y por consiguiente
z · z̄ = eiϕ · e−iϕ = e0 = 1. (1.155)
B) Usando la fórmula de Euler, tenemos
(eiϕ )2 = [cos(ϕ) + i sen(ϕ)]2 = cos2 (ϕ) + 2 sen(ϕ) cos(ϕ)i − sen2 (ϕ) (1.156)
Usando las fórmulas 2 sen(ϕ) cos(ϕ) ≡ sen(2ϕ), cos2 (ϕ) − sen2 (ϕ) ≡ cos(2ϕ)
Tenemos
(eiϕ )2 = cos(2ϕ) + i sen(2ϕ) = e2iϕ (1.157)
Ejemplo 1.10.11. Verificar la igualdad
1 3
cos3 x = cos 3x + cos x (1.158)
4 4
Solución: Vamos a usar las fórmulas de Euler
eix = cos x + i sen x, e−ix = cos x − i sen x (1.159)
Sumando estas dos expresiones, tenemos
eix + e−ix = cos x + i sen x + cos x − i sen x = 2 cos x,
eix + e−ix
cos x = (1.160)
2
Hemos expresado la función coseno en funciones exponenciales. Elevando al cubo,
resulta
3
e + e−ix e3ix + 3e2ix e−ix + 3eix e−2ix + e−3ix
 ix
cos3 x = = (1.161)
2 8
1 e3ix + e−3ix 3 eix + e−ix
   
1 3
= + = cos 3x + cos x
4 2 4 2 4 4
24 Álgebra de los Números Complejos
De esta forma queda verificada la igualdad. Hemos visto la gran utilidad de los
números complejos en forma exponencial.

Ejemplo 1.10.12. Representar el número complejo z = 3 + i 3 en su forma
trigonométrica y exponencial.
Solución: Tenemos
√ !
p √ √ 3 π
ρ = x2 + y 2 = 9 + 3 = 2 3, y ϕ = tg−1 = (1.162)
3 6
Por consiguiente, la forma trigonométrica se escribe como
√ h π  π i
z = 2 3 cos + i sen (1.163)
6 6
Y su forma exponencial tiene la forma

z = 2 3eiπ/6 (1.164)
√ √
Ejemplo 1.10.13. Escribir z = 2 − i 2 en su forma exponencial.
Solución: Tenemos

p √ 2 π
ρ = x + y = 2 + 2 = 2, tan ϕ = − √ = −1 → ϕ = −
2 2 (1.165)
2 4
Entonces, su forma exponencial es
z = 2e−iπ/4 (1.166)
iπ/2
Ejemplo 1.10.14. Escribir el número complejo z = e en su forma algebraica.
Solución: Tenemos
π π
z = eiπ/2 = cos + i sen =i (1.167)
2 2
Es decir, la forma algebraica de eiπ/2 es el número complejo i.
Ejemplo 1.10.15. Hallar todos los números complejos z que satisfacen la
relación

z n−1 = z̄ (1.168)
iϕ −iϕ
Solución: Sea z = ρe . Entonces, z̄ = ρe . Luego, de acuerdo con la
hipótesis
ρn−1 ei(n−1)ϕ = ρe−iϕ → ρn−2 einϕ = 1 (1.169)
De aquı́, resulta que
ρn−2 = 1, ρ = 1. (1.170)
y
2kπ
inϕ = 2kπi → ϕ = , (k = 0, 1, 2, .., (n − 1)) (1.171)
n
Entonces, el resultado es
zk = ei(2kπ/n) , k = 0, 1, 2...(n − 1) (1.172)
1.10 Ejemplos 25
Ejemplo 1.10.16.
√ Hallar todos los valores de la raı́z cuarta de −16, es decir,
los valores de 4 −16.
Solución: Antes que nada debemos escribir el número −16 en su forma
trigonométrica

−16 = 16 [cos (π + 2kπ) + i sen (π + 2kπ)] (1.173)

Entonces, de la fórmula (1.110) tenemos


√ √
    
4 4 π + 2kπ π + 2kπ
ωk = −16 = 16 cos + i sen (1.174)
4 4

Para los valores de k = 0, 1, 2, 3, tenemos


 √
2 cos π4 + i sen π4 = 2(1 + i)
  
k =0
2 cos 3π  + i sen 3π  = √2(−1 + i)



4  4  k =1
ωk =  5π 5π
√ (1.175)
 2 cos 4 + i sen 4 = − 2(1 + i) k =2

 

 7π
 √
2 cos 4 + i sen 4 = 2(1 − i) k =3

Ejemplo
√ 1.10.17. Hallar todos los valores de la raı́z cúbica de 27, es decir
3
27.
Solución: La representación trigonométrica de 27 es

27 = 27 [cos (0 + 2kπ) + i sen (0 + 2kπ)] (1.176)

Entonces, aplicando la fórmula (1.110), resulta



    
3 2kπ 2kπ
ωk = 27 = 3 cos + i sen (1.177)
3 3

Para k = 0, 1, 2, tenemos

3  √

  √
k=0
3 i 3

ωk = 3 − 2 + 2 = 2 − 3 + i k=1 (1.178)
3 − 3 − i = − 3 √ 3 + i 

  √ 
2 2 2 k=2
q √
Ejemplo 1.10.18. Calcular todos los valores de 4 −8 − (8 3)i

Solución: Sea z = √ −8 − (8 3)i. Entonces, podemos identificar los valores
de x = −8 y y = q −8 3. Por √ consiguiente el módulo de z expresado como
p
ρ = x + y = (−8) + (−8 3)2 = 16. El argumento ϕ lo podemos hallar
2 2 2

de la siguiente fórmula

y −8 3 √ √
tan ϕ = = = 3, → ϕ = tan−1 ( 3) = 240o (1.179)
x −8
26 Álgebra de los Números Complejos

Por consiguiente la forma trigonométrica de z = −8 − 8 3i es

−8 − 8 3i = 16 [cos (240o + 2πk) + i sen (240o + 2πk)] (1.180)

Entonces, usando la fórmula (1.110), tenemos


√ 240o + 360o k 240o + 360o k
q     
4
−8 − (8 3)i = 2 cos + i sen
4 4
o o o o
= 2 [cos (60 + 90 k) + i sen (60 + 90 )] (1.181)

Para los diferentes valores de k = 0, 1, 2, 3, se tiene


 √
2 [cos (60o ) + i sen (60o )] = 1 + 3i k =0
2 [cos (150o ) + i sen (150o )] = −√3 + i



k =1
ωk = o o
√ (1.182)
 2 [cos (240 ) + i sen (240 )] = −1 − 3i k =2

 √
2 [cos (330o ) + i sen (330o )] = 3 − i k =3


3
Ejemplo 1.10.19. Calcular todos los valores de 27i
Solución: Sea z = 27i. Escribamos este número en su forma trigonométrica.
Para esto, tenemos que el módulo es
p p
ρ = x2 + y 2 = (0)2 + (27)2 = 27,

cos ϕ = 0 y sen ϕ = 1, de donde obtenemos el valor del argumento ϕ = 90o .


Entonces, la forma trigonométrica de z = 27i es

27i = 27 [cos(90o + 360o k) + i sen (90o + 360o k)] (1.183)

Usando la fórmula (1.110), tenemos


√ √
 o
90 + 360o k
 o
90 + 360o k
  
3 3
27i = 27 cos + i sen
3 3
o o o o
= 3 [cos (30 + 120 k) + i sen (30 + 120 k)] (1.184)

Para los valores de k = 0, 1, 2, resulta


 √
o o 3 3 3
3 [cos 30 + i sen 30 ] = 2 +√ 2 i
 k=0
o o 3 3
ωk = 3 [cos 150 + i sen 150 ] = − 2 + 32 i k = 1 (1.185)

3 [cos 270o + i sen 270o ] = −3i k=2

Ejemplo 1.10.20. Hallar todos los valores del número complejo (1 − i)1/4 .
Solución: Sea z = 1 − i el número complejo. El primer paso es escribir
este número en su forma polar. Para esto debemos calcular su magnitud y su
argumento, esto es
p √ π
ρ = 12 + (−1)2 = 2, ϕ = arg (1 − i) = − (1.186)
4
1.10 Ejemplos 27
Entonces, tenemos que la forma trigonométrica de z = 1 − i es
√ h  π  π i
z = 1 − i = 2 cos − + i sen − (1.187)
4 4
Por consiguiente, usando la fórmula (1.107), tenemos
 π  π
− 4 + 2kπ − 4 + 2kπ
  
1
wk = z 1/4 = (1 − i)1/4 = (2) 8 cos + i sen
4 4
(1.188)
Dando a k los valores 0, 1, 2, 3, resulta que las raı́ces son:
π π
 1/8 

(2) cos 16 − i sen 16 k =0
cos 7π 7π
(2)1/8
 
ωk = 16 + i sen 16  k =1
(1.189)


(2)1/8 cos 15π 15π
16 + i sen 16  k =2
23π
cos 16 + i sen 23π
 1/8
(2) 16 k =3

Donde hemos usado las propiedades cos(−ϕ) = cos ϕ y sen (−ϕ) = − sen ϕ.
Estas propiedades son debido a que la función coseno es par y la función seno es
impar. Esta propiedad la usamos en (1.189), para el caso k = 0.

Ejemplo 1.10.21. Hallar las raı́ces quintas de z = 4 + 6i

Solución: En forma polar el número complejo z = 4 + 6i, se escribe como



      
3 3
z = (4 + 6i) = 52 cos tan−1 + i sen tan−1 (1.190)
2 2

Entonces, las raı́ces quintas están dadas por la expresión


( " # " #)
√ tan−1 32 + 2kπ tan−1 32 + 2kπ
 
1 10
w = (4 + 6i) 5 = 52 cos + i sen
5 5
(1.191)
 o
360 (0)+56,3o

 5
 360o (1)+56,3 = 11,26o k=0
o
o
= 83,26 k=1


 o 5

360 (2)+56,3o o
ϕk = 5 = 155,26 k = 2 (1.192)
o o
 360 (3)+56,3 = 227,26o k = 3


 5
 360o (4)+56,3o = 299,26o k = 4


5

En este caso se ha hecho la conversión de radianes a grados.



Ejemplo 1.10.22. Calcular las raı́ces del −1

Solución: Sea z = −1, su módulo y argumento son

ρ = 1, arg (−1) = π (1.193)


28 Álgebra de los Números Complejos
En la representación polar, podemos escribir

z = cos π + i sen π (1.194)

Luego, las raı́ces son


   
1/2 1/2 π + 2kπ π + 2kπ
wk = z = (−1) = cos + i sen , (k = 0, 1) (1.195)
2 2

Las diferentes raı́ces son


(
cos π2 + i sen π2 = i k = 0,
ωk = (1.196)
cos 3π 3π
2 + i sen 2 = −i k=1

La solución la podemos representar, simplemente como {i, −i}.


Ejemplo 1.10.23. Hallar las raı́ces cúbicas del número complejo z = −1 + i
Solución: Escribiendo el número complejo z = −1 + i en forma exponencial
√ 3πi
z = −1 + i = 2e 4 (1.197)

En forma trigonométrica, se tiene



1/6
2 [cos (π/4) + i sen(π/4)]
 k=0
ωk = 21/6 [cos (11π/4) + i sen(11π/4)] k=1 (1.198)

 1/6
2 [cos (19π/4) + i sen(19π/4)] k=2

Ejemplo 1.10.24. Calcular 1
Solución: Sea z = 1. Entonces, el módulo de z es ρ = 1 y la forma trigo-
nométrica de z = 1 es
1 = cos 0 + isen0 (1.199)
Usando la fórmula (1.107), tenemos

   
0 + 2kπ 0 + 2kπ
ωk = 1 = cos + i sen = cos(kπ) + i sen (kπ) (1.200)
2 2

donde k = 0, 1. Explı́citamente se tiene


(
cos(0) + i sen (0) = 1 k=0
ωk = (1.201)
cos (π) + i sen (π) = −1 k=1

3
Ejemplo 1.10.25. Calcular 1
Solución: Sea z = 1. Entonces, el módulo de z es ρ = 1 y la forma trigo-
nométrica de z = 1 es
1 = cos 0 + i sen 0 (1.202)
1.10 Ejemplos 29
Usando la fórmula (1.107), tenemos

   
3 0 + 2kπ 0 + 2kπ
ωk = 1 = cos + i sen (1.203)
3 3

donde k = 0, 1, 2. Explı́citamente se tiene



1


k=0
3
ωk = cos 2π 2π 1
 
3  + i sen 3  = − 2 + i √2
k=1 (1.204)

cos 4π 3 + i sen 3

= − 12 − i 23 k=2

( √ √ )
1 3 1 3
1, − + i, − − i (1.205)
2 2 2 2

4
Ejemplo 1.10.26. Calcular 1

Solución: Sea z = 1. Entonces ρ = 1 y la forma trigonométrica es

1 = cos 0 + i sen 0 (1.206)

Entonces
       
2πk 2πk πk πk
ωk = cos + i sen = cos + i sen (1.207)
4 4 2 2

donde k = 0, 1, 2, 3. Explı́citamente tenemos

Plano complejo ω
v
i


 1 k=0 1
u


i k=1
ωk = (1.208) −1


 −1 k = 2
−i k = 3

−i

Figura 1.2: Gráfica de las raı́ces


cuartas de 1.

GRAFICARLOS

Ejemplo 1.10.27. Calcular −5 − 12i
30 Álgebra de los Números Complejos
Solución:

{2 − 3i, −2 + 3i} (1.209)


Ejemplo 1.10.28. Hallar todos los valores de z que satisfacen la ecuación

z 5 + 32 = 0 (1.210)

y graficar los valores obtenidos.


Solución: Escribiendo el número z en forma exponencial

−32 = 32eiπ = 25 eiπ (1.211)

Entonces
    
π 2πk π 2πk
wk = 2 cos + + i sen + , (k = 0, 1, 2, 3, 4) (1.212)
5 5 5 5
Explı́citamente, los valores que satisfacen la ecuación (1.210) son:
h π  π i
k = 0, w0 = 2 cos + i sen
 5  5 
3π 3π
k = 1, w1 = 2 cos + i sen
5 5
k = 2, w2 = 2 [cos π + i sen π] = −2 (1.213)
    
7π 7π
k = 3, w3 = 2 cos + i sen
5 5
    
9π 9π
k = 4, w4 = 2 cos + i sen
5 5
(1.214)

1.11. Conjuntos de Puntos, Cı́rculos y Discos.


p Sea z1 = x1 + iy1 un número complejo diferente de cero, entonces ρ = |z| =
x21 + y12 es la distancia del orı́gen al punto (x1 , y1 ), en el plano complejo. Si
z2 = x2 + iy2 es también un número complejo, entonces la relación

p
|z1 − z2 | = |(x1 − x2 ) + i(y1 − y2 )| = (x1 − x2 )2 + (y1 − y2 )2 (1.215)

es la distancia entre z1 y z2 en el plano complejo. La expresión anterior es la


forma estándar de geometrı́a analı́tica, la cual representa la distancia entre dos
puntos (x1 , y1 ) y (x2 , y2 ).
1.11 Conjuntos de Puntos, Cı́rculos y Discos. 31
Plano complejo w
v
w1

w0

w2 u
1 2

w4

w3

Figura 1.3: Gráfica de las raı́ces quintas de −32.

Para dos números complejos arbitrarios se tiene la importante desigualdad

|z1 + z2 | ≤ |z1 | + |z2 | (1.216)

En forma general, la desigualdad del triángulo se escribe de la siguiente manera

|z1 + z2 + z3 + . . . + zn | ≤ |z1 | + |z2 | + |z3 | + . . . + |zn | (1.217)

Es decir, el valor absoluto de una suma no puede ser mayor que la suma de los
valores absolutos de los sumandos.
Sea a un número complejo y ρ un número positivo, entonces la ecuación

|z − a| = ρ (1.218)

se cumple para aquéllos puntos z cuya distancia a a es ρ. El lugar geométrico


de los puntos que satisfacen esta condición es el cı́rculo de radio ρ alrededor
de a. Esta es la manera de representar a los cı́rculos en el plano complejo y en
ocasiones decimos el cı́rculo |z − a| = ρ. Si a = 0, entonces cualquier punto en el
cı́rculo |z| = ρ tiene la forma polar

z = ρeiφ . (1.219)

6 0, entonces el centro del cı́rculo |z − a| = ρ es a en lugar del orı́gen. En


Si a =
este caso un punto en el cı́rculo tiene la forma

z = a + ρeiφ , (1.220)
32 Álgebra de los Números Complejos
que es un sistema de coordenadas polares trasladado a veces del orı́gen.
Una desigualdad |z − a| < ρ representa a todos los puntos dentro del cı́rculo
de radio ρ alrededor de a. A este conjunto se le conoce como disco abierto. Esto
significa que los puntos en la circunferencia del cı́rculo que acota a este disco no
pertenecen al conjunto. Un punto en este cı́rculo satisface la igualdad:

|z − a| = ρ (1.221)

Un disco cerrado de radio ρ y centro a consiste de todos los puntos en o dentro


del cı́rculo de radio ρ alrededor de a. Este conjunto está representado por la
desigualdad:

|z − a| ≤ ρ. (1.222)

Al representar gráficamente este conjunto dibujamos un cı́rculo sólido como la


frontera para indicar que esos puntos están incluidos en el disco cerrado.
Sean w1 y w2 dos números complejos distintos. La ecuación

|z − w1 | = |z − w2 |, (1.223)

representa la distancia entre z y w1 , la cual debe ser igual a la distancia entre z


y w2 .

1.12. Ejemplos
Ejemplo 1.12.1. ¿Qué representa la expresión |z − 3 + 7i| = 4.?

Solución: La expresión dada representa un cı́rculo de radio ρ = 4 alrededor


del punto (−3, 7) en el plano complejo. El número complejo 3 − 7i es el centro
del cı́rculo.

Ejemplo 1.12.2. ¿Que representa la desigualdad |z − i| < 8?

Solución: la desigualdad dada representa a los puntos dentro del disco


abierto de radio 8 alrededor de i.

Ejemplo 1.12.3. Obtener la ecuación de la recta si

|z + 6i| = |z − i + 3i| (1.224)


1.12 Ejemplos 33
Solución: La ecuación (1.224) se cumple para todos los puntos en el bisector
perpendicular del segmento entre (−6i) y (1 − 3i). Este es el segmento que
conecta a los puntos (0, 6) y (1, −3). Se Puede obtener la ecuación estándar de
esta recta como sigue, primero se escribe

|z + 6i|2 = |z − 1 + 3i|2 (1.225)

o bien
(z + 6i)(z̄ − 6i) = (z − 1 + 3i)(z̄ − 1 − 3i) (1.226)
esto determina los signos de los valores absolutos

z z̄ + 6i(z̄ − z) + 36 = z z̄ − z − 3iz − z̄ + 1 + 3i + 3iz̄ − 3i + 9 (1.227)

Sea z = x + iy entonces

z̄ − z = (x − iy) − (x + iy) = −2iy (1.228)

Luego
−z̄ − z = −(x − iy) − (x + iy) = −2x (1.229)
Ası́, la última ecuación se convierte en:

6i(2 − iy) + 36 = −2x + 3i(−2iy) + 10 (1.230)

12y = −2x + 6y − 26 (1.231)


de esta última expresión obtenemos la ecuación
1
y = − (x + 13) (1.232)
3
que representa una recta. Entonces, la igualdad |z + 6i| = |z − 1 + 3i|, describe
una recta separando el plano en dos conjuntos teniendo a la recta como frontera.
La desigualdad
|z + 6i| < |z − 1 + 3i|
mantiene a los puntos en uno de estos conjuntos, en un lado u otro de esta recta.
Es claro que z está más cerca de −6i que de 1 − 3i si z está abajo de la recta
frontera. Ası́ la desigualdad especifica todos los puntos z debajo de esta recta.
La recta frontera está punteada porque los puntos en esta recta no pertenecen
a esta región.

Ejemplo 1.12.4. Hallar la ecuación de la recta dada por la expresión

|z|2 + 3Re (z 2 ) = 4 (1.233)

Solución: Sea z = x + iy, entonces la ecuación (1.233) se transforma en

x2 + y 2 + 3(x2 − y 2 ) = 4 (1.234)
34 Álgebra de los Números Complejos
de donde la ecuación de la recta es

2x2 − y 2 = 2 (1.235)

El lugar geométrico de los puntos es una hipérbola. Un número complejo satisface


la ecuación dada si y sólo si su representación como un punto en el plano está en
la hipérbola.

Ejemplo 1.12.5. Escribir la ecuación del cı́rculo

x2 + y 2 + 2x + 2y = 0 (1.236)

en forma compleja.

Solución: Tenemos

x2 + y 2 = |z|2 = z z̄, 2x = z̄ + z, 2y = i(z̄ − z) (1.237)

Sustituyendo estas expresiones en (1.236), se obtiene

z z̄ + z̄ + z + i(z̄ − z) = 0 (1.238)

En forma equivalente, se puede escribir

z z̄ + (1 − i)z + (1 + i)z̄ = 0 (1.239)

Ejemplo 1.12.6. Escribir la ecuación de la recta

Ax + By + C = 0 (1.240)

en su forma compleja.

Solución: Sea z = x + iy y su complejo conjugado z = x − iy. Usando las


fórmulas
z̄ + z i(z̄ − z)
x= , y= (1.241)
2 2
Sustituyendo en (1.240), tenemos

A(z̄ + z) + iB(z̄ − z) + 2C = 0 (1.242)

o en su forma equivalente

(A + iB)z̄ + (A − iB)z + 2C = 0 (1.243)

Esta expresión representa la ecuación de la recta en el plano complejo.


1.13 Regiones del Plano Complejo. 35
1.13. Regiones del Plano Complejo.
En el desarrollo del cálculo de funciones complejas, ciertos tipos de conjuntos
y puntos serán importantes. Sea S un conjunto de números complejos, entonces:
Punto interior: Un número complejo z0 es un punto interior de S si existe
un disco abierto alrededor de z0 que contenga sólo puntos de S. Esto significa,
que para todo número real ρ > 0, todos los puntos que cumplen la relación
|z − z0 | < ρ están en S.
Entorno: Llamaremos entorno abierto de centro z0 y de radio  ∈ < al
conjunto de puntos del plano complejo:

z ∈ C : |z − z0 | <  (1.244)

esto es, el conjunto de puntos interiores a la circunferencia centrada en z0 y de


radio . A veces convendrá utilizar lo que se conoce como entornos reducidos, es
decir, entornos de un punto z0 en el que excluimos el propio punto z0 . Por otro
lado, hablaremos de puntos interiores, exteriores, frontera, de acumulación de
un subconjunto de los números complejos como se hace usualmente en R2 . Por
ejemplo, z es un punto interior de S ⊂ C si existe un entorno de z totalmente
contenido en S.
En cuanto a los conjuntos abiertos, cerrados y acotados: Un subconjunto de
los números complejos se dice abierto si todo punto es interior y se dice cerrado
si su complementario es abierto. Diremos, por otro lado, que un conjunto de
números complejos es acotado si está contenido en algún cı́rculo |z| ≤ R.
Los Conjuntos conexos, dominios y regiones: Un conjunto S ⊂ C se dice
conexo si todo par de puntos del mismo pueden unirse mediante una linea poli-
gonal que esté contenida en S. A los conjuntos abiertos y conexos los llamaremos
dominios. Finalmente, llamaremos regiones a los dominios junto con alguno,
ninguno, o todos sus puntos frontera.

1.14. Ejercicios Propuestos


1. Representar los siguientes números complejos en su forma cartesiana x + iy;

3i 2 + 3i 10 1 + 4i 3 + 5i
a) ; b) + ; c) +
4+i 5 − 2i 4 + 3i 3 − 2i 4i

√ √ i + i2 + i3 + i4 + i6
d) i(1 − i 3)( 3 + i)); e) (1 − 2i)3 ; f)
1+i

2. Sean z1 = 1+3i y z2 = 3−2i dos números complejos. Calcular las siguientes


operaciones

1 4
a) 2z1 + 4z2 ; b) + ; c) (3z1 )(5z2 ); d) z1 · z̄2 ; e) z1 · z̄1 · z2
z1 z2
36 Álgebra de los Números Complejos
3. Hallar los módulos, argumentos y las partes real e imaginaria de los sigientes
números complejos:
4 + 3i 1 1+i
a) (1 + i)3 ; b) ; c) ; d) (1 + i)30 ; e) .
2−i (1 − 2i)2 1−i

4. Representar los siguientes números complejos en su forma polar y expo-


nencial;
√ √
a) − i; b) − 4; c) 3 + 3 2i, d) − 3 + 3i; e) − 1 + 3i

5. Hallar el valor de las siguientes expresiones



1 1 1 1
a) − ; b) |3 − 4i| · |4 + 3i|; c) √ − .
1 + 2i 1 − 2i 2 − 3i 2 + 3i

6. Calcular el exponente de los siguientes números complejos


√ √ √
a) (−1 + 3)4 ; b) (2 + i 2)4 ; c) ( 2 + 4i)10 .

7. Calcular todos los valores de los siguientes números complejos


√ √ √
a) 6 −1; b) 81/6 ; c) (−i)1/3 ; d) 4 1 + i; e) i
3

8. Hallar todas las raı́ces de la ecuación algebraica



a) z 4 + 6iz 2 + 16 = 0, b) z 3 − 4 2(1 + i) = 0

9. Representar las curvas que describen las ecuaciones


 
1
a) Re = 1; b) Im(z 2 − z̄) = 2 − Imz; c) z 2 + z̄ 2 = 1

d) |z − i| + |z + i| = 4; e) |z − 2| = |1 − 2z̄|

10. Determinar el dominio en el plano complejo dado por

a) |z − i| < 1; b) |z| ≥ 2; c) |z − 5i| = 8; d) |z − 1 + 2i| ≤ 4


π 3π
e) < arg z < , |z| < 1; f) 1 ≤ |z| ≤ 2
2 2
Capı́tulo 2

Funciones de una Variable


Compleja

Contents
2.1. Funciones de una Variable Compleja . . . . . . . . . 37
2.2. Sucesiones y Series Complejas . . . . . . . . . . . . . 41
2.3. Ejemplos . . . . . . . . . . . . . . . . . . . . . . . . . . 43
2.4. Funciones Complejas Elementales . . . . . . . . . . . 44
2.5. Lı́mite y Continuidad de una Función Compleja . . 51
2.6. Derivada de una Función Compleja. . . . . . . . . . 54
2.7. Funciones Armónicas Conjugadas. . . . . . . . . . . 59
2.8. Mapeo Conforme . . . . . . . . . . . . . . . . . . . . 62
2.9. Ejercicios Resueltos. . . . . . . . . . . . . . . . . . . . 63

En este capı́tulo daremos las definiciones de función y lı́mite de una función


compleja. Se analizan la continuidad y la derivada de las funciones complejas.
Además, se introduce el concepto de funciones armónicas conjugadas y la ecuación
de Laplace.

2.1. Funciones de una Variable Compleja


Supongamos que la variable compleja z = x + iy toma todos los valores
posibles de un conjunto complejo Z. Si a cada valor de z ∈ Z se le puede poner
38 Funciones de una Variable Compleja
en correspondencia uno o varios valores de otra variable compleja w = u + iv
que pertenece al conjunto complejo W , entonces la variable compleja w se llama
función de z en el dominio Z y se representa como
w = f (z). (2.1)
La función (2.1) es una función univaluada si a cada valor de z, donde z ∈ Z, se
le pone en correspondencia solo un valor de w, donde w ∈ W . Podemos también
decir que la función (2.1) mapea (transforma) los puntos del plano complejo Z a
los puntos en el plano complejo W .
Si existen valores de z a cada uno de los cuales se le pone en correspondencia
varios valores de w, entonces decimos que la función (2.1) es multivaluada.
Si w = u + iv es una función de z = x + iy, entonces cada una de las
variables u y v será una función de x, y, es decir u(x, y) y v(x, y). Inversamente,
si w = u(x, y) + iv(x, y), donde u(x, y) y v(x, y) son funciones reales de x, y,
entonces w se puede ver como función de la variable compleja z = x + iy.
Sean z = x + iy y w = u + iv puntos en los planos complejos Z y W , la
dependencia w = f (z) de la variable dependiente w sobre la variable compleja
independiente z puede ser descrita especificando dos funciones de valores reales
u(x, y) y v(x, y) de las variables x y y. Sean
z = x + iy (2.2)
y
u = u(x, y), v = v(x, y) (2.3)
entonces, comparando las expresiones (2.2) y (2.3), una función compleja se
puede escribir como

f (z) = u(x, y) + iv(x, y) (2.4)

donde
u(x, y) = Re f (z) = Re f (x + iy), v(x, y) = Im f (z) = Im f (x + iy) (2.5)
Por consiguiente, una función compleja, f (z), la podemos considerar como un
par de funciones reales u(x, y) y v(x, y) de dos variables reales x, y.
Ejemplo 2.1.1. Sea f (z) = z 2 , determinar las funciones reales u(x, y) y v(x, y).
Solución: Debido a que z = x + iy, tenemos
f (x + iy) = (x + iy)2 = x2 − y 2 + 2ixy (2.6)
De esta expresión identificamos las funciones reales
u(x, y) = x2 − y 2 , v(x, y) = 2xy (2.7)
Entonces, la función f (z) = z 2 es equivalente a las dos funciones reales en (2.7).
2.1 Funciones de una Variable Compleja 39
Ejemplo 2.1.2. Sea la función compleja f (z) = z1 , z 6= 0. Hallar las funciones
reales u(x, y) y v(x, y).
Solución: Tenemos z = x + iy, entonces
1 (x − iy) x y
f (x + iy) = = = 2 −i 2 (2.8)
x + iy (x + iy)(x − iy) x + y2 x + y2
De esta expresión obtenemos las funciones reales
x y
u(x, y) = , v(x, y) = − (2.9)
x2 + y2 x2 + y2
Ejemplo 2.1.3. Hallar las funciones reales si la función compleja es w = zz̄ .
Solución: Sean w = u(x, y) + iv(x, y), z = x + iy y z̄ = x − iy, sustituyendo
en la función dada, tenemos
x − iy (x − iy)(x − iy) x2 − y 2 2xy
u(x, y) + iv(x, y) = = = 2 2
−i 2 (2.10)
x + iy (x + iy)(x − iy) x +y x + y2
Igualando estas expresiones se obtienen las funciones reales
x2 − y 2 2xy
u(x, y) = , v(x, y) = − (2.11)
x2 + y 2 x2 + y 2
Ejemplo 2.1.4. Sea la función compleja, w = z 3 − iz̄, hallar las funciones
reales.
Solución: Sean z = x + iy, z̄ = x − iy, y w = u(x, y) + iv(x, y). Sustituyendo
en la función dada, se tiene

u(x, y)+iv(x, y) = (x+iy)3 −i(x−iy) = (x3 −3xy 2 −y)+i(3x2 y−y 3 −x) (2.12)

Podemos concluir que las funciones reales vienen dadas por las expresiones

u(x, y) = x3 − 3xy 2 − y, v(x, y) = 3x2 y − y 3 − x (2.13)

Entonces, la función w = z 3 − iz̄ es equivalente a las dos funciones reales en


(2.13).
Ejemplo 2.1.5. Supongamos que deseamos saber en que se transforma el punto
z0 = −i en el plano complejo Z, bajo la función w = z 3 .
Solución: Tenemos la función w = z 3 que llevará al punto z0 = −i a otro
punto del plano complejo W . ¿Cual es este nuevo punto?. Para contestar a esta
pregunta, simplemente sustituimos en la función el punto dado, es decir, hacemos
lo siguiente
w = z 3 = (−i)3 = i (2.14)
Si graficamos el punto z0 = −i, tenemos que este punto se encuentra en el eje
imaginario y abajo del origen. Este punto bajo el mapeo w = z 3 se transformó en
un punto w = i que se encuentra en el eje y arriba del origen.
40 Funciones de una Variable Compleja
Ejemplo 2.1.6. ¿En qué punto se transforma z0 = 3 + 2i bajo la transformación
w = z 2 − 3z̄?
Solución: Sustituyendo el punto dado z0 = 3 + 2i en la transformación,
tomando en cuenta su conjugado, se tiene

w = z 2 − 3z̄ = (3 + 2i)2 − 3(3 − 2i) = −4 + 18i (2.15)

Ejemplo 2.1.7. ¿En qué punto se transforma z0 = 2 + 3i bajo la transformación


w = zz̄ ?
Solución: Tenemos el punto complejo z0 = 2+3i, su conjugado es z̄0 = 2−3i,
sustituyendo en la función
z̄ 2 − 3i (2 − 3i)(2 − 3i) 5 12
w= = = =− − i (2.16)
z 2 + 3i (2 + 3i)(2 − 3i) 13 13
No es difı́cil graficar los puntos z0 = 2 + 3i y el punto al cual se transformó (2.16).
El primer punto está en el primer cuadrante y el punto transformado en el tercer
cuadrante del plano complejo W .
Ejemplo 2.1.8. Dada la función w = z 2 + z hallar los valores de la función si
a) z = 1 + i, b) z = 2 − i, c) z = i.
Solución:
a) sustituyendo el valor de z = 1 + i en la función dada resulta

w = (1 + i)2 + (1 + i) = 1 + 3i (2.17)

b) Sustituyendo el valor z = 2 − i en la función dada, se tiene

w = (2 − i)3 + 2 − i = 5 − 5i (2.18)

c) Sustituyendo z = i en la función dada

w = (i)2 + i = −1 + i (2.19)

Ejemplo 2.1.9. Dada la función f (z) = x2 + iy 2 , donde z = x + iy. Hallar


a) f (1 + 2i), b) f (2 − 3i), c) f (−i).
Solución:
a) para este caso, de f (1 + 2i), tenemos que x = 1 y y = 2, sustituyendo
estos valores en la función dada, resulta

f (z) = f (1 + 2i) = 12 + i(2)2 = 1 + 4i (2.20)

b) en este caso x = 2 y y = −3, sustituyendo en la función dada, se tiene

f (z) = f (2 − 3i) = 22 + i(−3)2 = 4 + 9i (2.21)

c) en este caso x = 0 y y = −1, sustituyendo en la función dada, se tiene

f (z) = f (2 − 3i) = i (2.22)


2.2 Sucesiones y Series Complejas 41
1
Ejemplo 2.1.10. Dada la función f (z) = x−iy , donde z = x + iy. Hallar
a) f (1 + i), b) f (i), c) f (3 − 2i).
Solución: Antes que nada, la función dada la podemos escribir como

1 (x + iy) x y
f (z) = = = 2 2
+ 2 i (2.23)
x − iy (x − iy)(x + iy) x +y x + y2

a) para este caso tenemos x = 1 y y = 1, sustituyendo esto valores en (2.23),


resulta
1 i
f (1 + i) = + (2.24)
2 2
b) en este caso x = 0 y y = 1, sustituyendo en (2.23), resulta

f (i) = i (2.25)

c) tenemos x = 3 y y = −2, entonces, sustituyendo en (2.23), resulta


3 2
f (3 − 2i) = − i (2.26)
13 13

2.2. Sucesiones y Series Complejas


Sea z1 , z2 , z3 , . . . una sucesión de números complejos representada por {zn },
donde zn = xn + iyn (n = 1, 2, 3, . . .). El número constante c = a + ib se llama
lı́mite de la sucesión {zn } si, dado cualquier número  > 0 existe un número
positivo N tal que todos los valores de zn con n ≥ N satisfacen la desigualdad

|zn − c| <  (2.27)

La expresión (2.27) significa que podemos hacer cada término zn tan cerca como
queramos de c eligiendo n al menos tan grande como cierto número N . En otras
palabras, dado cualquier disco abierto D alrededor de z0 , podemos encontrar
algún término de la sucesión de manera que todos los términos de la lista a partir
de él (esto es, para un ı́ndice suficientemente grande) caiga en D figura. Si la
sucesión {zn } converge a c cuando n → ∞, escribimos

lı́m zn = c (2.28)
n→∞

Si una sucesión no converge, entonces decimos que la sucesión diverge.


La condición necesaria y suficiente para la existencia del lı́mite de la sucesión
{zn }, es la siguiente: el número complejo c = a + ib es el lı́mite de la sucesión de
los números complejos x1 + iy1 , x2 + iy2 , x3 + iy3 , . . . si y sólo si

lı́m xn = a, lı́m yn = b (2.29)


n→∞ n→∞

La serie
w1 + w2 + w3 + . . . (2.30)
42 Funciones de una Variable Compleja
con términos complejos, se llama convergente si la n-ésima suma parcial Sn de la
serie (2.30) tiende a un lı́mite finito cuando n → ∞, en caso contrario, decimos
que la serie (2.30) diverge.
La serie (2.30) converge si y sólo si, las series formadas de números reales
convergen, es decir
Re w1 + Re w2 + Re w3 + . . . (2.31)

y
Im w1 + Im w2 + Im w3 + . . . (2.32)

Si la suma de serie (2.31) es el número S 0 y la suma de la serie (2.32) es S 00 ,


entonces la suma de la serie (2.30) será el número complejo

S = S 0 + iS 00 (2.33)

Si la serie
w1 + w2 + w3 + . . . + wn (wn = un + ivn ) (2.34)

converge, entonces
lı́m wn = 0 (2.35)
n→∞

Es decir
lı́m un = 0 lı́m vn = 0 (2.36)
n→∞ n→∞

Si la serie
|w1 | + |w2 | + |w3 | + . . . + |wn | + . . . (2.37)

converge, entonces también converge la serie

w1 + w2 + w3 + . . . + wn + . . . (2.38)

En tal caso se dice que la serie (2.38) converge absolutamente.


Una serie de potencias es una serie de la forma

a0 + a1 (z − z0 ) + a2 (z − z0 )2 + . . . + an−1 (z − z0 )n−1 + . . . (2.39)

donde z0 , a0 , a1 , a2 , a3 , . . . son números complejos, además los coeficientes de


la serie son diferentes de cero, y z es una variable compleja.
La serie (2.39) converge en el cı́rculo |z − z0 | < R, donde

an
R = lı́m (2.40)
n→∞ an+1

y diverge fuera de él, es decir, para los valores de z que satisfacen la desigualdad
|z − z0 | > R. A R se le llama radio de convergencia.
2.3 Ejemplos 43
2.3. Ejemplos
Ejemplo 2.3.1. Analizar la convergencia de la serie
     
1 i 1 i 1 i
(1 + i) + + + + + ... + + + ... (2.41)
2 3 4 9 2n−1 3n−1
Solución: La serie compleja (2.41) la podemos escribir de la siguiente manera
 
1 1 1 1 1 1
1 + + + . . . + n−1 + . . . + i 1 + + + . . . + n−1 . (2.42)
2 4 2 3 9 3
Las series reales
1 1 1
1+ + + . . . + n−1 (2.43)
2 4 2
1 1 1
1 + + + . . . + n−1 (2.44)
3 9 3
convergen, ya que están formadas de términos que decaen en forma de progresión
geométrica infinita. Por consiguiente, la serie compleja (2.41) también converge.
La suma de estas progresiones está dada por
1 1 3
S1 = = 2, S2 = = (2.45)
1 − 1/2 1 − 1/3 2
Por consiguiente, la suma de la serie compleja (2.41) es el número complejo
3
S = S1 + iS2 = 2 + i (2.46)
2
Ejemplo 2.3.2. Analizar la convergencia de la serie compleja
       
1 2 2 3 3 4 n n+1
+ i + + i + + i + ... + + i + . . . (2.47)
2 3 3 4 4 5 n+1 n+2
Solución: Podemos escribir la serie (2.47) de la siguiente manera

 
1 2 3 n 2 3 4 n+1
+ + + ... + + ... + i + + + ... + + ... (2.48)
2 3 4 n+1 3 4 5 n+2
Podemos analizar las series por separado, es decir
1 2 3 n
+ + + ... + + ... (2.49)
2 3 4 n+1
2 3 4 n+1
+ + + ... + + ... (2.50)
3 4 5 n+2
El término general de cada serie lo podemos escribir como
n n+1
S1 = S2 = (2.51)
n+1 n+2
44 Funciones de una Variable Compleja
La suma de estos teérminos es
n n+1
S1 = lı́m = 1, S2 = lı́m =1 (2.52)
n→∞ n+1 n→∞ n+2
Entonces, la serie (2.47) diverge, ya que no se cumplen las condiciones de
convergencia (2.35-2.36)

Ejemplo 2.3.3. Hallar el dominio de convergencia de la serie de potencias


√ ! √ !2 √ !n
3+i 3+i 2 3+i
(z − i) + (z − i) + . . . + (z − i)n + . . . (2.53)
3 3 3

Solución: Tenemos
√ !n √ !n+1
3+i 3+i
an = , an+1 = (2.54)
3 3

Entonces √ n √ n
3+i 3+i
an 3 3 3
=  √ n+1 =  √ n  √  =√ (2.55)
an+1 3+i 3+i 3+i 3+i
3 3 3

El módulo es

an
R = lı́m = √ 3 = √ 3 =
3
(2.56)
n→∞ an+1 3 + i 3+1 2

Entonces, el dominio de convergencia de la serie (2.53) es |z − i| < 3/2.

2.4. Funciones Complejas Elementales


1. Función Potencial: Esta función tiene la forma

w = zn, (n − entero positivo) (2.57)

La función potencia esta definida para todos los números complejos; a cada
número complejo z = x + iy le pone en correspondencia z n , es decir, el
producto de n múltiplos, cada uno de los cuales es igual a z.
Tambien se puede definir la función potencia para n entero negativo y para
n = 0. Por definición
1
z −n = (n = 1, 2, ...) z0 = 1 (2.58)
zn
La función w = z −n (n = 1, 2, . . .) esta definida para todos los valores
de z 6= 0
2.4 Funciones Complejas Elementales 45
2. Función Exponencial: La función exponencial w = ez del número com-
plejo z = x + iy se define por la fórmula

w = ez = ex+iy = ex (cos y + i sen y) (2.59)

Por consiguiente

Re ez = ex cos y Im ez = ex sen y (2.60)

De la definición se deducen las siguientes propiedades para ez :


Para cualesquiera números complejos z1 y z2 tiene lugar la igualdad

ez1 +z2 = ez1 ez2

La función exponencial compleja w = ez es periódica de periodo 2πi,


es decir, cumple la relación

ez+2πi = ez

La función exponencial ez es continua en todo el plano complejo.


Para todo número complejo z = x + iy tienen lugar las igualdades

|ez | = ex arg ez = y

La función ez toma todos los valores, excepto el cero, es decir la


ecuación ez = A tiene solución para cualquier número complejo A 6= 0
diferente de cero. Si α = arg A, entonces todas las soluciones de la
ecuación ez = A están dadas por la fórmula

z = ln |A| + i(α + 2kπ), (k = 0, ±1, ±2, . . .) (2.61)

En particular, si ez = 1, entonces z = 2kπi, k = 0, ±1, ±2, . . .


z
Si e = A, entonces el número complejo z se llama el logaritmo del
número complejo A 6= 0 y se representa por log A, donde log es la
expresión para logaritmo base e o logaritmo natural.
De la fórmula (2.61) se sigue

log A = log |A| + i arg A (2.62)

En particular, log(1) = 2kπi, log(−1) = (2k + 1)πi, log(i) =


(2k + 1/2)πi, (k − entero).
3. Funciones Trigonométricas: Las funciones sen z y cos z para los valores
complejos z se definen por las siguientes fórmulas
1 iz 1 iz
e − e−iz , e + e−iz
 
sen z = cos z = (2.63)
2i 2
De éstas definiciones se siguen las siguientes propiedades para las funciones
sen z y cos z
46 Funciones de una Variable Compleja
Las funciones sen z y cos z son continuas en todo el plano complejo.
Las funciones sen z y cos z toman todos los valores, es decir, las
ecuaciones sen z = A y cos z = A tienen soluciones para cualquier
número complejo A.
Todas las fórmulas trigonométricas elementales, válidas para todos
los valores reales x, son válidas y para los valores complejos z. Por
ejemplo;

sen2 z + cos2 z = 1, sen 2z = 2 sen z cos z


sen(z1 + z2 ) = sen z1 cos z2 + cos z1 sen z2
cos(z1 + z2 ) = cos z1 cos z2 − sen z1 sen z2 (2.64)

En particular

sen(z + 2π) = sen z, cos(z + 2π) = cos z (2.65)

Es decir, las funciones sen z y cos z son periódicas de periodo 2π.

sen (−z) = − sen z, cos(−z) = cos z (2.66)


Es decir, sen z es una función impar, mientras que cos z es una función
par.
4. Para cualquier z = x + iy se tienen las siguientes desigualdades
1 y 1 y
e − e−y ≤ | sen z| ≤ e + e−y

(2.67)
2 2

1 y 1 y
e − e−y ≤ | cos z| ≤ e + e−y

(2.68)
2 2
5. Tienen lugar las siguientes fórmulas

sen(x + iy) = sen x · cosh y + i cos x · senh y (2.69)


cos(x + iy) = cos x · cosh y − i sen x · senh y (2.70)

De estas fórmulas (ó de las fórmulas (2.67), (2.68)) se sigue, en particular,


que las ecuaciones sen z = 0 y cos z = 0 tienen soluciones sólo cuando y = 0,
es decir, sólo en el eje real. Por consiguiente, todas las soluciones de la
ecuación sen z = 0 estan dadas por la fórmula z = kπ, k = 0, ±1, ±2, . . .
y todas las soluciones de la ecuación cos z = 0 por la fórmula z = π/2 + kπ
k = 0, ±1, ±2, . . ..
Las funciones tan z y cot z se definen como
sen z cos z
tan z = , cot z = (2.71)
cos z sen z
6 π/2 + kπ, y la función cot z
La función tgz es continua para los valores z =
es continua para los valores z 6= kπ, donde k = 0, ±1, ±2, . . .
2.4 Funciones Complejas Elementales 47
Todas las fórmulas de la trigonometrı́a (en el campo real) son válidas para
las funciones trigonométricas de una variable compleja.
sen(−z) = − sen z, cos(−z) = cos z.
cos2 z + sen2 z = 1.
sen(z1 ± z2 ) = sen z1 cos z2 ± cos z1 sen z2 .
cos(z1 ± z2 ) = cos z1 cos z2 ∓ sen z1 sen z2 .
sen 2z = 2 sen z cos z, cos(2z) = cos2 z − sen2 z.
sen z̄ = sen z.
sen(z + 2nπ) = sen z, ∀ n entero.
cos(z + 2nπ) = cos z, ∀ n entero.

1. Las funciones hiperbólicas senhz, cosh z, tanh z y coth z están definidas


como:

ez + e−z ez − e−z
cosh z = , senh z = (2.72)
2 2

senh z cosh z
tanh z = , coth z = . (2.73)
cosh z senh z

Las funciones hiperbólicas complejas satisfacen las siguientes identidades

senh(−z) = − senh z, cosh(−z) = cosh z.


2 2
cosh z − senh z = 1.
cosh(z1 + z2 ) = cosh z1 cosh z2 + senh z1 senh z2 .
cosh 2z = cosh2 z + senh2 z.
senh 2z = 2 senh z cosh z.
sen(iz) = i senh z.
cos iz = cosh z.
senh z = senh x cos y + i cosh x sen y.
cosh z = cosh x cos y + i senh x sen y.
48 Funciones de una Variable Compleja
| senh z|2 = senh2 x + sen2 y.
| cosh z|2 = senh2 x + cos2 y.

2. Las funciones trigonométricas e hiperbólicas están relacionas mediante las


siguientes fórmulas:

sen z = −i senh(iz), senh z = −i sen(iz), (2.74)


cos z = cosh(iz), cosh z = cos(iz), (2.75)
tan z = −i tanh(iz), tanh z = −i tan(iz), (2.76)
cot z = i coth(iz), coth z = i cot(iz). (2.77)

6 0, está definida como la inversa de


3. La función logarı́tmica Log z, para z =
la función exponencial, y

Log z = log |z| + iArg z = log |z| + i arg z + 2iπk, (k = 0, ±1, . . .).
(2.78)

Esta es una función multivaluada. El valor principal de Logz es el valor


para k = 0 y es representado como log z

log z = log |z| + i arg z. (2.79)

Obviamente

Log z = log z + 2iπk, (k = 0, ±1, ±2, . . .). (2.80)

Para la función logarı́tmica las siguientes fŕmulas son válidas:

Log z1 · z2 = Log z1 + Log z2 , (2.81)


 
z1
Log = Log z1 − Log z2 (2.82)
z2

4. Las funciones trigonométricas inversas Arcsen z, Arccos z, Arctan z y


Arccot z están definidas como las inversas de sen w, cos w, tan w y cot w,
respectivamente. Por ejemplo, si z = sen w decimos que w es el arcseno de
z y se escribe como w = Arcsen z ó de la siguiente manera w = sen−1 z.
Todas estas funciones son multivaluadas y se pueden representar en términos
de la función logarı́tmica
2.4 Funciones Complejas Elementales 49

 p 
Arcsen z = −iLog iz + 1 − z 2 , (2.83)
 p 
Arccos z = −iLog z + z 2 − 1 , (2.84)
 
i 1 + iz
Arctan z = − Log , (2.85)
2 1 − iz
 
i z+i
Arccot z = − Log . (2.86)
2 z−i

Los valores principales de las funciones trigonométricas inversas pueden


ser obtenidos por los valores principales correspondientes de las funciones
logarı́tmicas.
5. La función potencial general w = z a , donde a = α + iβ es un número
complejo arbitrario, está definida por

z a = eaLog z . (2.87)

Esta función, en general, es también multivaluada, su valor principal es

z a = eaLog z . (2.88)

6. La función exponencial general w = az , (con a 6= 0 es un número complejo


arbitrario) esta definida como

az = ezLog a . (2.89)

El valor principal de esta función multivaluada es (2.89).

Ejemplo 2.4.1. Resolver la ecuación

sen z = 3. (2.90)

Solución: El problema se reduce a encontrar

z = Arcsen 3. (2.91)

Usando la fórmula (2.78)


 p 
Arcsen t = −iLog it + 1 − t2 , (2.92)

obtenemos  p 
z = Arcsen 3 = −iLog 3i + 1 − 32 (2.93)
√ √
y teniendo en cuenta −8 = ±i 8, resulta
h √ i
z = −i Log (3 ± 8)i . (2.94)
50 Funciones de una Variable Compleja

Luego, debemos calcular el Log (3 − i 8), usando la fórmula (2.78), para esto
calculamos primero el argumento
h √  i π
arg 3 ± 8 i = . (2.95)
2
y el módulo
 √  q √ √
3 + 8 i = 02 + (3 + 8)2 = 3 + 8

 √  q √ √
3 − 8 i = 02 + (3 − 8)2 = 3 − 8 (2.96)

p
2 2
Recordemos que el módulo de √ un número complejo z es |z| = x + y . En
nuestro caso x = 0 y y = 3 + 8.
Sustituyendo estos resultados en la fórmula (2.78), tenemos
h √  i  √  π
Log 3 ± 8 i = log 3 ± 8 + i + 2kπi, (k = 0, ±1, ±2, . . .). (2.97)
2
Finalmente, poniendo este resultado en (2.94), resulta
π  √ 
z = + 2kπ − i log 3 ± 8 , (k = 0, ±1, ±2, . . .). (2.98)
2
Ejemplo 2.4.2. Resolver la ecuación compleja

sen z = i. (2.99)

Solución:

Ejemplo 2.4.3. Resolver la ecuación compleja

ez = 1 + 2i. (2.100)

Solución:

Ejemplo 2.4.4. Verificar la fórmula


 p 
Arcsen z = −iLog iz + 1 − z 2 . (2.101)

Solución:
Sea w = Arcsen z, entonces

eiw − e−iw
z = sen w = (2.102)
2i
Multiplicando esta expresión por e−iw , resulta la ecuación cuadrática

e2iw − 2izeiw − 1 = 0. (2.103)


2.5 Lı́mite y Continuidad de una Función Compleja 51
la cual tiene como solución

iw 2iz ± 4 − 4z 2 p p
e = = iz ± 1 − z 2 = iz + 1 − z 2 (2.104)
2
Luego
eiw = ei(w−2kπ) , donde k = 0, ±1, ±2, . . . (2.105)
Ası́ que
p 1 p
ei(w−2kπ) = iz + 1 − z2 w = 2kπ + log(iz + 1 − z 2 ) Si k = 0 (2.106)
i
calculando su rama principal, se obtiene que
1 p 1 p
log(iz + 1 − z 2 ) ≡ log(iz + 1 − z 2 ) (2.107)
i i

2.5. Lı́mite y Continuidad de una Función


Compleja
Lı́mite de una función. Sea a un punto lı́mite del conjunto Z, es decir,
cualquier vecindad del punto a contiene un número infinito de puntos del conjunto
Z. El número A se llama lı́mite de la función f (z) cuando z → a po mnozhestvu
Z, si para cualquier  > 0 existe un δ = δ() > 0, tal que para todos z ∈ Z que
cumplen la condición 0 < |z − a| < δ se cumple la desigualdad |f (z) − A| < .
En tal caso escribimos

lı́m f (z) = A (2.108)


z→a

donde a y A son constantes complejas. En ocaciones la fórmula (2.108) se escribe


de la siguiente manera f (z) → A cuando z → a y se dice, la función f (z) tiende
a A cuando z tiende a a.
Decimos que la función w = f (z) es continua en el punto z0 , si se cumple la
relación

lı́m f (z) = f (z0 ). (2.109)


z→z0

Una función continua en cada punto de un dominio D, se llama continua en el


dominio dado.
52 Funciones de una Variable Compleja
Supongamos que
lı́m f (z) (2.110)
z→z0

donde
f (z) = u(x, y) + iv(x, y) (2.111)
existe, y
z0 = x0 + iy0 (2.112)
entonces el lı́mite de f (z) esta dado por

lı́m f (z) = lı́m u(x, y) + i lı́m v(x, y). (2.113)


z→z0 x→x0 , y→y0 x→x0 , y→y0

Muchos teoremas sobre lı́mites en el cálculo de variable real también son válidos
para funciones de variable compleja. Supongamos que los lı́mites

lı́m f (z) = L, lı́m g(z) = K (2.114)


z→z0 z→z0

existen. Entonces son válidas las expresiones

lı́m [f (z) + g(z)] = L+K


z→z0
lı́m [f (z) − g(z)] = L−K
z→z0
lı́m cf (z) = cL ∀c (2.115)
z→z0
lı́m [f (z)g(z)] = LK
z→z0
f (z) L
lı́m =
z→z0 g(z) K
Una diferencia importante entre los lı́mites de funciones complejas y los lı́mites
de funciones reales, es la manera en que la variable se aproxima al punto. Para
un función real g(x), tenemos que,

lı́m g(x) = A
x→a

involucra el comportamiento de la función g(x) conforme x se aproxima a a por


cualquier lado. En la recta solo hay dos maneras de que x se aproxime a a, por
la izquierda o por la derecha.
En el caso de las funciones complejas la relación

lı́m f (z) = L (2.116)


z→z0

involucra el comportamiento de f (z) conforme z se aproxima a z0 en el plano


complejo y esto puede involucrar que z se aproxime a z0 desde cualquier dirección.
2.5 Lı́mite y Continuidad de una Función Compleja 53
Los números f (z) deben aproximarse a L a lo largo de cualquier trayectoria de
aproximación de z a z0 en D. Si a lo largo de una sola trayectoria de aproximación
de z a z0 , f (z) no se aproxima a L, entonces f (z) no tiene lı́mite en z0 .
La diferencia anterior entre las funciones reales y complejas implica que la
relación (2.116), en el plano complejo, sea un enunciado más fuerte que en el
caso real.
Ejemplo 2.5.1. Calcular los siguientes lı́mites:
4z 2
1. lı́mz→∞ (z−1)2 .
Solución: Desarrollando el binomio, tenemos

4z 2
4z 2 z2 4
lı́m2
= lı́m z2
= lı́m 2z 1 = 4 (2.117)
z→∞ z − 2z + 1 z→∞
z2 − 2z 1
z2 + z2
z→∞ 1− z2 + z2

1
2. lı́mz→1 = (z−1)3
Solución: Escribiendo el lı́mite de la siguiente manera

1 1
lı́m = lı́m
z→1 (z − 1)(z 2 − 2z + 1) z→1 (z 3 − 3z 2 + 3z − 1)
1
z3
= lı́m 3 3z 2
z→1 z 3 − + 3z
− 1
z z3 z3 z3
1
1 1
= 1 3 3 1 = → ∞ (2.118)
1 − 1 + 1 − 1
0
el lı́mite no existe.
3. Si f (z) = z 2 + 2z, calcular el lı́mite cuando z → i.
Solución: Se tiene
lı́m f (z) = z 2 + 2z = (i)2 + 2(i) = i2 + 2i = −1 + 2i (2.119)
z→i

2z+i
4. lı́mz→i z+1 .
Solución: Dividiendo entre z y tomando en cuenta que z → 0, se obtiene

2z i i
z + z 2+ z
lı́m = lı́m =2 (2.120)
z→0 z + 1 z→0 1 + 1
z z z
z−2i
5. lı́mz→2i z 4 −16 .
Solución: Aplicando la Regla de L´Hôpital
d(z−2i)
z − 2i dz 1 1 1 i
lı́m = lı́m d(z 4 −16)
= lı́m = = =
z→2i z 4 − 16 z→2i z→2i 4z 3 4(2i)3 32(−i) 32
dz
(2.121)
54 Funciones de una Variable Compleja
6. lı́mz→2i (iz 4 + 3z 2 − 10i).
Solución: Aplicando el lı́mite, resulta

lı́m (iz 4 + 3z 2 − 10i) = [i(2i)4 + 3(2i)2 − 10i] = −12 + 6i (2.122)


z→2i

z2
7. lı́m πi
z 4 +z+1 .
z→e 4

Solución: Aplicando el lı́mite

πi
z2 e 2
lı́mπi 4
= πi
z→e 4
z +z+1 eπi + e 4 +1
cos( π2 ) + i sen( π2 )
=
cos(π) + i sen(π) + cos( π4 ) + sen( π4 ) + 1
i i
= 1 1 = 1
−1 + 2 + 2 i + 1
√ √ √
2
+ √12 i
√ √
2i 1 − i 2i(1 − i)
= · =
1+i 1−i 1 − i2
√ √
2 2
= (i − i2 ) = (1 + i) (2.123)
2 2

(2z−3)(4z+1)
8. lı́mz→ 2i (iz−1)2

Solución: Aplicando el lı́mite

(2z − 3)(4z + 1) [2( 2i ) − 3][4( 2i ) + i]


lı́mi =
z→ 2 (iz − 1)2 [i( 2i ) − 1]2
(i − 3)(2i + i) (i − 3)(3i) 3i2 − 9i
= 1 2
= 3 2 = 9
[− 2 − 1] (− 2 ) 4
12 36 4
= (−1) − i = − − 4i (2.124)
9 9 3

2.6. Derivada de una Función Compleja.


Sea w = f (z) una función definida en un dominio D del plano complejo z.
Sean z y z + ∆z dos puntos en el dominio dado D. Entonces, el incremento de
la función f (z) está dada por la expresión

∆w f (z + ∆z) − f (z)
= . (2.125)
∆z ∆z
2.6 Derivada de una Función Compleja. 55
Se dice que f 0 (z) es la derivada de la función f (z) en un punto z si existe el
lı́mite de ∆w
∆z , cuando ∆z → 0. Es decir

∆w f (z + ∆z) − f (z)
f 0 (z) = lı́m = lı́m . (2.126)
∆z→0 ∆z ∆z→0 ∆z

Una función que tiene derivada para un valor dado de z, se llama función
diferenciable para el valor de z. Si la función w = f (z) es unı́voca y tiene
derivada finita en cada punto de un dominio D, entonces, la función se llama
analı́tica en el dominio dado.
Si la función w = f (z) = u(x, y) + iv(x, y) es diferenciable en el punto
z = x + iy, entonces en este punto existen derivadas parciales ∂u ∂v ∂u ∂v
∂x , ∂x , ∂y , ∂y ,
además estas derivadas estan relacionadas por las condiciones

∂u ∂v ∂u ∂v
= , =− . (2.127)
∂x ∂y ∂y ∂x

A estas condiciones se les conoce como condiciones de Cauchy-Riemann. Estas


condiciones son necesarias para que la fucnión w = f (z) sea diferenciable en
el punto z = x + iy. Inversamente, si las derivadas parciales ∂u ∂v ∂u ∂v
∂x , ∂x , ∂y , ∂y
son continuas en el punto z = x + iy y las condiciones de Cauchy-Riemann se
cumplen, entonces, la función w = f (z) es diferenciable en el punto z = x + iy.
La derivada de la función f (z) se representa por medio de las derivadas
parciales de las funciones u(x, y) y v(x, y), de la siguiente manera

∂u ∂v ∂v ∂u ∂u ∂u ∂v ∂v
f 0 (z) = +i = −i = −i = +i . (2.128)
∂x ∂x ∂y ∂y ∂x ∂y ∂y ∂x

La derivada de las funciones elementales z n , ez , cos z, sen z, log z, arc sen z,


arc cos z, arctan z, senh z, cosh z se encuentran según las fórmulas del análisis
56 Funciones de una Variable Compleja
real
1
(z n )0 = nz n−1 , (arc sen z)0 = √ .
1 − z2
1
(ez )0 = ez , (arc cos z)0 = − √ .
1 − z2
1
(cos z)0 = − sen z, (arctan z)0 = 2 .
z +1
0 0
(sen z) = cos z, (senh z) = cosh z.
1
(log z)0 = , (cosh z)0 = senh z. (2.129)
z
Ejemplo 2.6.1. Calcular la derivada de la función f (z) = z, ∀z ∈ D.

Solución: Usando la definición (2.126), tenemos

f (z + ∆z) = z + ∆z. (2.130)

Entonces
z + ∆z − z ∆z
f 0 (z) = lı́m = lı́m = 1. ∀z ∈ D. (2.131)
∆z→0 ∆z ∆z→0 ∆z

Ejemplo 2.6.2. Hallar la derivada de la función f (z) = z 2 , ∀z ∈ D.

Solución: Por definición, tenemos

(z + ∆z)2 − z 2 2z∆z + (∆z)2


f 0 (z) = lı́m = lı́m
∆z→0 ∆z ∆z→0 ∆z
= lı́m (2z + ∆z) = 2z. (2.132)
∆z→0

En general, si n > 0 y f (z) = z n , entonces f 0 = nz n−1 . Como podemos ver, esta


fórmula es idéntica a la del cálculo con variables reales.
Otras fórmulas similares son:

[f (z) ± g(z)]0 = f 0 (z) ± g 0 (z).

[cf (z)]0 = cf 0 (z).

[f (z)g(z)]0 = f (z)g 0 (z) + g(z)f 0 (z).


i0 0
(z)g 0 (z)
h
f (z)
g(z) = g(z)f (z)−f
[g(z)]2 .

Si f (w) = g(w), y w = f (z) → rmdf rmdg dw


rmdz = dw dz , regla de la cadena. Donde
estamos suponiendo que las derivadas de las funciones f (z) y g(z) existen.

Ejemplo 2.6.3. Hallar la derivada f 0 (w) como función de z, si f (w) = w5 y


w = 2z 2 + i.
2.6 Derivada de una Función Compleja. 57
Solución: Haciendo uso de la regla de la cadena, tenemos
df df dw
= = 5w4 (4z) = 20zw4 = 20z(2z 2 + i)4 . (2.133)
dz dw dz
Ejemplo 2.6.4. Hallar la derivada de la función compleja f (z) = z 3 , donde
z = x + iy.
Solución: Para usar las condiciones de Cauchy-Riemann, debemos calcular,
primero, las funciones reales u(x, y) y v(x, y), tenemos
f (x+iy) = (x+iy)3 = x3 +3ix2 y−3xy 2 +iy 3 = x3 −3xy 2 +i(3x2 y−y 3 ). (2.134)
De donde podemos identificar las funciones reales
u(x, y) = x3 − 3xy 2 , v(x, y) = 3x2 y − y 3 . (2.135)
Ahora debemos probar que las condiciones de Cauchy-Riemann se cumplen.
Derivando estas funciones reales respecto a x, resulta
∂u ∂v
= 3x2 − 3y, = 6xy. (2.136)
∂x ∂x
Sustituyendo estos resultados en la expresión (2.128), obtenemos
∂u ∂v
f 0 (z) = +i = 3x2 − 3y 2 + 6ixy = 3(x2 − y 2 + 2ixy) = 3z 2 . (2.137)
∂x ∂x
De esta forma hemos obtenido una manera más de calcular las derivadas de un
función compleja.
Las condiciones de Cauchy-Riemann constituyen una condición necesaria, pero
no suficiente, para que f (z) sea diferenciable en un punto. Si estas condiciones
no se satisfacen, entonces f 0 (z) no existe en ese punto.
Ejemplo 2.6.5. Sea la función compleja f (z) = ez , donde z = x + iy. Hallar
su derivada.
Solución: Escribiendo la función f (z) = ez de la siguiente manera
f (z) = ez = ex+iy = ex eiy = ex cos y + iex sen y (2.138)
De donde, tenemos las funciones reales
u(x, y) = ex cos y, v(x, y) = ex sen y (2.139)
Veamos si las ecuaciones de Cauchy-Riemann se cumplen. Para esto, derivamos
las expresiones, obteniendo
∂u ∂v ∂v ∂u
= ex cos y, = ex sen y, = ex cos y, = −ex sen y (2.140)
∂x ∂x ∂y ∂y
Como podemos ver, las condiciones se cumplen para todos los valores de x y y.
Finalmente, obtenemos
∂u ∂v
f 0 (z) = +i = ex cos(y) + iex sen(y) = ex eiy = ex+iy = ez (2.141)
∂x ∂x
58 Funciones de una Variable Compleja
Ejemplo 2.6.6. Sea la función f (z) = z̄ 2 . Hallar los puntos en los cuales la
función dada es diferenciable.
Solución: Sabemos que z̄ = x − iy, entonces

f (z) = (x − iy)2 = x2 − y 2 − 2ixy (2.142)

De donde
u(x, y) = x2 − y 2 , v(x, y) = −2xy (2.143)
Veamos si se cumplen las condiciones de Cauchy-Riemann. Para esto derivamos
estas expresiones respecto a x y a y, obtenemos
∂u ∂u ∂v ∂v
= 2x, = −2y, = −2y, = −2x (2.144)
∂x ∂y ∂x ∂y
Luego, las condiciones de Cauchy-Riemann son
∂u ∂v ∂u ∂v
= , =− (2.145)
∂x ∂y ∂y ∂x
Como se puede ver, las condiciones de Cauchy-Riemann se cumplen solo para el
caso en que x = y = 0, por lo tanto, la función f (z) = z̄ 2 es diferenciable solo en
el punto z = 0.
Ejemplo 2.6.7. Sea la función f (z) = z̄. Hallar los puntos en los cuales esta
función es diferenciable.
Solución: Tenemos que z̄ = x − iy, entonces

f (z) = x − iy (2.146)

De donde
u(x, y) = x, v(x, y) = −y (2.147)
Derivando estas expresiones, tenemos
∂u ∂v
= 1, = −1 (2.148)
∂x ∂y
La condición de Cauchy-Riemann no se cumple para cualquier z, y por consi-
guiente la función f (z) no es diferenciable en ningún punto.
Ahora recordemos que todo número complejo lo podemos escribir en su forma
exponencial como z = ρeiφ , entonces, la función f (z) en coordenadas polares
tendrá la forma:

f (z) = u(ρ, φ) + iv(ρ, φ), (2.149)


2.7 Funciones Armónicas Conjugadas. 59
y las condiciones de Cauchy- Riemann en coordenadas polares, se escriben como

∂u 1 ∂v ∂v 1 ∂u
= , =− . (2.150)
∂ρ ρ ∂φ ∂ρ ρ ∂φ

Por lo tanto, la derivada de la función f (z) en éstas coordenadas viene dada por
la expresión

   
ρ ∂u ∂v 1 ∂v ∂u
f 0 (z) = +i = −i . (2.151)
z ∂ρ ∂ρ z ∂φ ∂φ


Ejemplo 2.6.8. Demostrar que la función f (z) = z es diferenciable.
Solución: Escribiendo z en su forma exponencial z = ρeiφ . Luego
 
√ φ √ φ φ
f (z) = ρei 2 = ρ cos + i sen (2.152)
2 2

Identificamos a las funciones reales


   
√ φ √ φ
u(ρ, φ) = ρ cos , v(ρ, φ) = ρ sen (2.153)
2 2

Ahora derivemos estas expresiones


  √  
∂u 1 φ ∂v ρ φ
= √ cos , = cos (2.154)
∂ρ 2 ρ 2 ∂φ 2 2

Es fácil ver que√las condiciones de Cauchy-Riemann se cumplen. Entonces, la


función f (z) = z es diferenciable. Usando la fórmula (2.151), obtenemos

      
0 ρ ∂u ∂v ρ 1 φ i φ
f (z) = +i = √ cos + √ sen
z ∂ρ ∂ρ z 2 ρ 2 2 ρ 2
√ iφ
ρe 2 1
= = √ (2.155)
2ρeiφ 2 z

2.7. Funciones Armónicas Conjugadas.


Sea f (z) = u(x, y) + iv(x, y) una función diferenciable en una región D y,
además, las funciones u(x, y) y v(x, y) tienen derivadas parciales continuas hasta
60 Funciones de una Variable Compleja
el segundo orden. Entonces, diferenciando la primer ecuación de (2.127) respecto
a x y la segunda respecto a y, obtenemos:
∂2u ∂2v ∂2u ∂2v
2
= , 2
=− (2.156)
∂x ∂x∂y ∂y ∂y∂x
2
∂ v ∂2v
Sumando estas igualdades, y teniendo en cuenta ∂y∂x y ∂x∂y , debido a que estas
derivadas son continuas, éstas son iguales. Entonces

∂2u ∂2u
+ 2 = 0. (2.157)
∂x2 ∂y

De igual manera, obtenemos

∂2v ∂2v
+ = 0. (2.158)
∂x2 ∂y 2

La función real u(x, y), la cual tiene en una región D derivadas parciales continuas
de segundo orden y que satisfacen la ecuación (2.157), se llama función armónica
en D, y la ecuación (2.157) se llama ecuación de Laplace.
En otras palabras, las funciones armónicas son aquéllas que satisfacen la
ecuación de Laplace.
Las partes reales e imaginarias de la función f (z) = u(x, y) + iv(x, y)
diferenciable en D serán funciones armónicas en D.
Las funciones armónicas u(x, y) y v(x, y) se relacionan entre sı́ por las
condiciones de Cauchy-Riemann, se llaman conjugadas.
Si en la región D están dadas dos funciones armónicas conjugadas u(x, y) y
v(x, y), entonces, la función f (z) = u(x, y) + iv(x, y) es diferenciable en D.
Teorema 2.7.1. Para que la función f (z) = u(x, y) + iv(x, y) sea diferenciable
en D es necesario y suficiente que las funciones u(x, y) y v(x, y) sean armónicas
en D.
Teorema 2.7.2. Para cualquier función u(x, y) armónica en D, se puede hallar
una función armónica conjugada a ella, la cual se define con exactitud hasta una
constante arbitraria.
Ejemplo 2.7.1. Verificar que la función f (z) = ey · eix es armónica.
Solución: Escribiendo la función dada en forma trigonométrica para identi-
ficar las funciones u(x, y) y v(x, y), tenemos
f (z) = ey · [cos x + i sen x], u = ey cos x
v(x, y) = ey sen x (2.159)
2.7 Funciones Armónicas Conjugadas. 61
calculamos las segundas derivadas parciales de u(x, y) respecto a x y y, obtenemos
∂u ∂u
= −ey sen x, = ey cos x
∂x ∂y
∂2u ∂2u
= −ey cos x, = ey cos x (2.160)
∂x2 ∂y 2
Entonces, como resultado tenemos que, la suma de las segundas derivadas de
u(x, y) respecto a x y y es cero. Por consiguiente, la función u(x, y) es armónica
ya que cumple la ecuación da Laplace.
De igual manera para v(x, y) se obtiene
∂v ∂v
= ey cos x, = ey sen x,
∂x ∂y
∂2v ∂2v
= −ey sen x, = ey sen x (2.161)
∂x2 ∂y 2
que tambien cumple la ecuación de Laplace. Por consiguiente, v(x, y) es armónica.
Con esto podemos afirmar que la función f (z) = ey · eix es armónica.
Ejemplo 2.7.2. Verificar que la función f (z) = z 2 es armónica.
Solución: Sea z = x + iy sustituyendo en la función dada, tenemos
f (x + iy) = (x + iy)2 = x2 + 2ixy − y 2 (2.162)
Identificamos las funciones reales u(x, y) y v(x, y)
u(x, y) = x2 − y 2 , v(x, y) = 2xy (2.163)
Luego, tomando las segundas derivadas de estas funciones respecto a x y y,
resulta

∂u ∂u
= 2x, = −2y,
∂x ∂y
∂2u ∂2u
= 2 = −2 (2.164)
∂x2 ∂y 2
Sumando las segudas derivadas resulta cero, lo cual implica que la función
u(x, y) = x2 − y 2 satisface la ecuación de Laplace y por consiguiente es armónica.
De manera similar se tiene para la función v = 2xy. Las derivadas parciales
son
∂v ∂v
= 2y = 2x
∂x ∂y
∂2v ∂2v
= 0 =0 (2.165)
∂x2 ∂y 2
Como podemos ver la función v(x, y) = 2xy tambien satisface la ecuación de
Laplace y por lo tanto es una función armónica. Las dos funciones u(x, y) y
v(x, y) son armónicas, por consiguiente, la función dada f (z) = z 2 tambien lo
es.
62 Funciones de una Variable Compleja
2.8. Mapeo Conforme

Supongamos que la función w = f (z) es analı́tica en un dominio dado D


con un cierto valor z = x + iy. A este valor z le corresponde un valor dado
w = u + iv. Ası́, a cada punto (x, y) en el plano xOy le correspoende un punto
definido (u, v) del plano uOv. De esta manera se tiene un mapeo (aplicación o
transformación) de D sobre el rango de valores de f (z) en el plano ω.
Si el punto (x, y) en el plano xOy describe una lı́nea Γ, raspolozhenuyu en el
dominio D, entonces el punto (u, v) en el plano (u, v) describe una lı́nea Γ0 La
lı́nea Γ0 se llama imagen de la lı́nea Γ en el plano uOv con ayuda de la función
analı́tica w = f (z)
En la lı́nea Γ elijamos el punto z0 = x0 + iy0 . A este punto, en la lı́nea Γ0 ,
le corresponde el punto w0 = u0 + iv0 . Trazemos en la lı́nea Γ una tangente L
en el punto (x0 , y0 ), y en la lı́nea Γ0 una tangente L0 en el punto (u0 , v0 ). Sea
α el ángulo al cual nada girar la lı́nea L para que coincida con la dirección de
la lı́nea L0 . En la teorı́a de funciones analı́ticas se demuestra, que si f 0 (z0 ) 6= 0
entonces α = arg f 0 (z0 ).
Consideremos otra lı́nea γ, la cual también pasa por el punto (x0 , y0 ) y su
imágen la lı́nea γ 0 , la cual pasa por el punto (u0 , v0 ). Supongamos que l es
tangente a la lı́nea γ en el punto (x0 , y0 ) y l0 es tangente a la lı́nea γ 0 en el punto
(u0 , v0 )
Para que la dirección de la recta l coincida con la dirección de l0 , es necesario
girar la lı́nea l un ángulo α, ya que el ángulo de giro es igual a f 0 (z0 ).
Si φ y ψ son ángulos, sostevleni kasatielnimi L y l con el eje Ox, a φ0 y
ψ kosotielnimi L0 y l0 con el eje Ou, entonces φ0 − φ = α, ψ 0 − ψ = α y
0

φ0 − φ = ψ 0 − ψ. Por consiguiente, ψ − φ = ψ 0 − φ0 . Pero, ψ − φ es el ángulo


entre tangentes L y l, y ψ 0 − φ0 es el ángulo entre las tangentes L0 y l0 . Por lo
tanto, dos rectas cualesquiera, que se intersectan en el punto (x0 , y0 ) se reflejan
en dos correspondientes lı́neas que se cruzan en el punto (u0 , v0 ), de tal manera
que el ángulo β entre tangentes a las lı́neas dadas y sus imagenes sea el mismo.
Se puede mostrar que el módulo de la derivada en el punto (x0 , y0 ), es
decir |f 0 (z0 )|, representa el lı́mite de la relación de la distancia entre los puntos
reflejados ω0 + ∆ω0 y ω0 y los puntos iniciales z0 + ∆z0 y z0 figura.
Si consideramos otra curva y su imagen, podemos concluir que |f 0 (z0 )|
representa el lı́mite de la relación de la distancia entre los puntos ω0 + ∆0 ω0 y
ω0 y los puntos iniciales z0 + ∆0 z0 y z0 .
Por lo tanto |f 0 (z0 )| es una magnitud de deslizamiento (iscazhenia) de escala
en el punto z0 bajo el mapeo con ayuda de la función ω = f (z).
El mapeo (representación) con una función ω = f (z) se llama mapeo conforme
si preserva la magnitud y la dirección de los ángulos, excepto en puntos crı́ticos
(puntos donde f 0 (z0 ) = 0).
2.9 Ejercicios Resueltos. 63
2.9. Ejercicios Resueltos.
Ejemplo 2.9.1. Para cada una de las siguientes funciones encontrar u(x, y) y
v(x, y).

A) f (z) = e3iz
Solución: Sea z = x + iy, entonces

e3i(x+iy) = e3ix−3y = e3ix e−3y = e−3y [cos 3x + i sen 3x]


= e−3y cos 3x + ie−3y sen 3x (2.166)

de donde, podemos identificar

u(x, y) = e−3y cos 3x, v(x, y) = e−3y sen 3x (2.167)

B) f (z) = cos(z)
Solución: Sea z = x + iy

cos(x + iy) = i cos x cos y − sen x sen y


= cos x cosh y − sen x[i senh y]
= cos x cosh y − i sen x senh y (2.168)

de donde identificamos

u(x, y) = cos x cosh y v(x, y) = − sen x senh y (2.169)

C) f (z) = sen(2z)
Solución: Si z = x + iy

sen(2x + 2iy) = i sen 2x cos 2y + i cos 2x sen 2y


= sen 2x cosh 2y + i cos 2x senh 2y (2.170)

De aquı́ podemos identificar las funciones reales

u(x, y) = sen 2x cosh 2y v(x, y) = cos 2x senh 2y (2.171)

D) f (z) = z 2 e2z
Solución: Si z = x + iy

(x + iy)2 e2(x+iy) = (x2 + 2xyi − y 2 )e2x [cos(2y) + i sen(2y)] (2.172)


= x2 e2x cos(2y) + x2 e2x sen(2y)i + 2xye2x cos(2y)i −
− 2xye2x sen(2y) − y 2 e2x cos(2y) − y 2 e2x senh(y)i
= x2 e2x cos(2y) − 2xye2x sen(2y) − y 2 e2x cos(2y) +
+ [x2 e2x sen(2y) + 2xye2x cos(2y) − y 2 e2x sen(2y)]i
64 Funciones de una Variable Compleja
De donde

u(x, y) = e2x [cos(2y)(x2 − y 2 ) − 2xy sen(2y)]


v(x, y) = e2x [sen(2y)(x2 − y 2 ) + 2xy cos(2y)] (2.173)

Ejemplo 2.9.2. Obtener una expresión para w = cos−1 (z) en términos de


logaritmos.

Solución:

eiw + e−iw
z = cos(w) = = e2wi − 2zeiw + 1 = 0 (2.174)
2
Aplicando la formula general
p
iw1,2 2z ± 4z 2 − 4(1)(1) p
e = = z ± 1 z2 − 1 (2.175)
2
p
w = −i[log(z ± 1 z 2 − 1)] (2.176)

Ejemplo 2.9.3. Encontrar los valores de z tales que ez = 1 + 2i se cumpla.

Solución: Sea z = x + iy, tenemos

ex cos y + iex sen y = 1 + 2i (2.177)

Entonces
ex cos y = 1, ex sen y = 2 (2.178)
Elevando al cuadrado

e2x [cos2 y + sen2 y] = e2x = 5 (2.179)

Obtenemos
1
x= ln 5 (2.180)
2
Ahora dividiendo
ex sen y
= tan y = 2 (2.181)
ex cos y
ası́
y = tan−1 (2) (2.182)
Una solución de la ecuación dada es
1
z= log 5 + i tan−1 (2) (2.183)
2

Ejemplo 2.9.4. Calcular todos los valores de 2i .


2.9 Ejercicios Resueltos. 65
Solución: Tenemos

log(2) = [log 2 + i arg 2] = [log 2 + 2nπi] (2.184)

En donde n varı́a sobre todos los enteros. Los valores de 2i son

ei[log 2] = ei[log 2+2nπi] = e−2npi ei log 2 = e−2nπ {cos[log 2] + i sen[log 2]} (2.185)

en donde n es cualquier entero. Ahora, sabemos que 2i tiene una infinidad de


valores complejos, por tanto

2i = {e−2nπ [cos(log 2) + i sen(log 2)]} (2.186)


66 Funciones de una Variable Compleja
Capı́tulo 3

Integrales Complejas

Contents
3.1. Integral de una Función Compleja. . . . . . . . . . . 67
3.2. Integrales de Lı́nea. . . . . . . . . . . . . . . . . . . . 69
3.3. Integral Compleja en Términos de Integrales Reales. 71
3.4. Teorema de Cauchy-Goursat. . . . . . . . . . . . . . 77
3.5. Forma Integral de Cauchy. . . . . . . . . . . . . . . . 82

En las secciones anteriores hemos estudiado el álgebra de los números com-


plejos, las funciones de una variable compleja, lı́mites y derivadas de funciones
complejas. Por consiguiente, estamos listos para estudiar la integración de fun-
ciones de una variable compleja.

3.1. Integral de una Función Compleja.


Debido a que el plano complejo es bidimensional existe una mayor libertad
en lo que entendemos por integral compleja (in what we mean by a complex
integral). Por ejemplo, nos hacemos la pregunta, si la integral de cierta función
entre los puntos A y B depende de la curva por la que se integra (upon the
curve along which we integrate). Consecuentemente, un ingrediente importante
en una integral compleja es el contorno que seguiremos durante la integración.
El resultado de una integral compleja es una expresión compleja. A diferencia de
su contraparte en variables reales, no existe una interpretación fı́sica para estas
68 Integrales Complejas
cantidades, tales como un área bajo una curva. Por lo general, la integración
en el plano complejo es un proceso intermediario con una cantidad fı́sicamente
realizable la cual ocurre solo después que hallamos tomado la parte real o
imaginaria (with a physically realizable quantity ocurring only after we take its
real or imaginary part).
El caso más simple de integración de una función compleja se tiene cuando
f (z) es una función compleja definida en un intervalo cerrado de números reales
[a, b].
Sea f : [a, b] → C una función compleja. Sea f (x) = u(x) + iv(x) una función
compleja definida en el intervalo cerrado de números reales a ≤ x ≤ b. Entonces
la integral de f (x) de a a b, está definida de la siguiente manera

Z b Z b Z b
f (x)dx = u(x)dx + i v(x)dx. (3.1)
a a a

Las integrales de la derecha son integrales de Riemann de funciones reales en


[a, b].

1
2
Ejemplo 3.1.1. Integrar la función f (x) = x − i , ∀x, 1 ≤ x ≤ 2.

Solución: Por definición tenemos


Z b Z 2   Z 2  
2
1 dx 1 2
f (x)dx = − 1 dx − 2i = − − x − 2i log x

x2 x x

a 1 1 1 1
1
= − − 2i log 2 − 2 + 1 + 2i log 1 + 1 =
2
1
= − − 2i log 2 (3.2)
2

Ejemplo 3.1.2. Integrar la función f (x) = x − ix2 , ∀x, 1 ≤ x ≤ 2.

Solución: Por definición tenemos

b 2 Z 2
x3 2
Z Z
x 2
f (x) dx = x dx − i x2 dx = − i =
a 1 2 1 3 1
 1 
1 8 1 3 7
= 2− −i − = − i (3.3)
2 3 3 2 3

π
Ejemplo 3.1.3. Integrar la función f (x) = ei2x , ∀x, 0 ≤ x ≤ 6.
3.2 Integrales de Lı́nea. 69
Solución: Por definición tenemos
Z b Z π6 π6
1 1 iπ 1
f (x) dx = e2ix dx = e2ix = e 3 − e(2i)(0)

a 0 2i 0 2i 2i
1 iπ 1 1 h π  π i 1
= e3 − = cos + i sen −
2i " 2i 2i !# 3 3 2i
  √ √
1 1 3 1 1 3 1
= +i − = + −
2i 2 2 2i 4i 4 2i

3 1
= + (3.4)
4 4i
Ejemplo 3.1.4. Integrar la función f (x) = cos(2x) + i sen(2x), 0 ≤ x ≤ π/4.
Solución: Por definición, tenemos
Z b Z π/4 Z π/4
1 1
f (x) dx = cos(2x) dx + i sen(2x) dx = + i (3.5)
a 0 0 2 2
Esta misma integral la podemos calcular de la siguiente manera, escribiendo la
función dada como f (x) = e2ix e integrando
Z b Z π/4
e2ix π/4 1  iπ/2 
f (x) dx = e2ix dx = = e −1
2i 0 2i

a 0
1 1
= [cos (π/2) + i sen (π/2) − 1] = (1 + i) (3.6)
2i 2

3.2. Integrales de Lı́nea.


También podemos definir la integral de una función compleja sobre una curva
en el plano.
Sea f : [a, b] → C una función compleja. Sea Γ : [a, b] → C una curva suave
en el plano. Supongamos que f es continua en todos los puntos en Γ. Entonces,
la integral de f sobre Γ se define como:
Z Z b
f (z) dz = f [Γ(t)]Γ0 (t) dt (3.7)
Γ a

Como z = Γ(t) en la curva, la integral (3.7) se escribe como

Z Z b
f (z) dz = f [z(t)]z 0 (t) dt. (3.8)
Γ a

Esta forma de escribir


R la integral compleja tiene la ventaja de sugerir la manera
en que la integral Γ f (z) dz es evaluada, reemplazando a z por z(t) en la curva.
Luego, dz = z 0 (t)dt e integrando sobre el intervalo a ≤ t ≤ b.
70 Integrales Complejas
z̄ dz si Γ(t) = eit , 0 ≤ t ≤ π.
R
Ejemplo 3.2.1. Evaluar la integral Γ

Solución: La gráfica de Γ(t) es la mitad del cı́rculo unitario orientado de 1


a −1. En Γ, z(t) = eit y z 0 (t) = ieit . Luego f [z(t)] = z̄(t) = e−it . Entonces
Z Z π Z π
−it it
f (z) dz = e ie dt = i dt = πi. (3.9)
Γ 0 0

z 2 dz si Γ(t) = t + it, en el intervalo


R
Ejemplo 3.2.2. Evaluar la integral Γ
0 ≤ t ≤ 1.
2
Solución: Tenemos z(t) = t(1 + i) y f (z) = z 2 , entonces f [z(t)] = (z(t)) =
(1 + i)2 t2 = 2it2 de donde z 0 (t) = 1 + i. La integral se transforma en
Z Z 1 Z 1  
2 2 2 2 2i 3 2 3 1 2
z dz = (i + i)2it dt = [2it − 2t ] dt = t − t = (−1 + i)
Γ 0 0 3 3 0 3
(3.10)
Ejemplo 3.2.3. Evaluar la integral Γ z Re(z) dz, si Γ(t) = t − it2 en 0 ≤ t ≤ 2.
R

Solución: Tenemos

f [z(t)] = z Re(z) = (t − it2 )t, z 0 (t) = 1 − 2it (3.11)

Entonces
Z Z 2 Z 2
z Re(z) dz = (t2 − it3 )(1 − 2it) dt = (t2 − 3it3 − 2t4 ) dt
Γ
0 3  0
t 3i 4 2 5 2 152
= − t − t =− − 12i (3.12)
3 4 5 0 15

En ocasiones es necesario integrar sobre una curva dada. En tal caso, debemos
encontrar las funciones que describen a la curva.
Ejemplo 3.2.4. Supongamos que necesitamos integrar la función f (z) = z sobre
el segmento de recta de 3 a 2 + i.
Solución: La ecuación de la recta la podemos hallar usando la fórmula:
y − y1 x − x1
= (3.13)
y2 − y1 x2 − x1
Los puntos que unen a la recta son (3, 0) y (2, 1). Sustituyendo estos valores en
la fórmula, encontramos la ecuación
x−3
y= =3−x (3.14)
2−3
Luego, podemos parametrizar la recta como

z(t) = (5 − t) + (t − 2)i, 2≤t≤3 (3.15)


3.3 Integral Compleja en Términos de Integrales Reales. 71
De tal manera que z(2) = 3 y z(3) = 2 + i, con esta parametrización tenemos
una correcta orientación. Ahora z 0 (t) = −1 + i. La integral se puede escribir
como
Z Z 3 Z 3
z dz = [(5 − t) + (t − 2)i] (−1 + i) dt = (−3 + 7i − 2it) dt = −3 + 2i
Γ 2 2
(3.16)

3.3. Integral Compleja en Términos de


Integrales Reales.
Es lógico pensar en la integral de una función compleja sobre una curva como
una suma de integrales de lı́nea de funciones de valor real de dos variables reales
sobre la curva. Un método muy conocido de calcular integrales complejas de
lı́nea consiste en separar la integralen una parte real y otra parte imaginaria.
Esto reduce la integral compleja a integrales de lı́nea de funciones reales.
Sea
f (z) = u(x, y) + iv(x, y) (3.17)
y
z = x + iy, dz = dx + idy (3.18)
sustituyendo en la integral
Z Z
f (z) dz = [u(x, y) + iv(x, y)] [dx + i dy] (3.19)
C C
Z Z
= u(x, y) dx − v(x, y) dy + i v(x, y) dx + u(x, y) dy
C C

Entonces, la integral de una función compleja, está definida como

Z Z Z
f (z) dz = u(x, y) dx − v(x, y) dy + i v(x, y) dx + u(x, y) dy.
C C C
(3.20)

El método para evaluar este tipo de integrales, en general, depende del contorno
C dado.
De la definición de integral de lı́nea, tenemos las siguientes propiedades
Z Z
f (z) dz = − f (z) dz (3.21)
C C0
0
donde C es el contorno C tomado en la dirección opuesta de C.
Z Z Z
f (z) dz = f (z) dz + f (z) dz (3.22)
C1 +C2 C1 C2
72 Integrales Complejas
En general, tenemos
Z Z Z Z
f (z) dz = f (z) dz + f (z) dz + . . . + f (z) dz (3.23)
C1 +C2 +...+Cn C1 C2 Cn

Ejemplo 3.3.1. Evaluar la integral


Z
z̄ dz (3.24)
C

de z = 0 a z = 4 + 2i, a lo largo de dos contornos diferentes. Si el primer


contorno consiste de la ecuación paramétrica z = t2 + it. El segundo término
consiste de dos ramas, la primera recorre a lo largo del eje imaginario de z = 0
a z = 2i y después a lo largo de la lı́nea paralela al eje x de z = 2i a z = 4 + 2i.
Solución: Los puntos z = 0 a z = 4 + 2i por el contorno C1 corresponden a
t = 0 y t = 2, respectı́vamente. Entonces, la integral de lı́nea
Z Z 2 Z 2
8
z̄ dz = (t2 − it) d(t2 + it) = (2t3 − it2 + t) dt = 10 − i (3.25)
C1 0 0 3

La integral de lı́nea para el segundo contorno se divide en dos partes


Z Z Z
z̄ dz = z̄ dz + z̄ dz (3.26)
C2 C2a C2b

donde C2a representa la integración de z = 0 a z = 2i y C2b la integración de


z = 2i a z = 4 + 2i. Para la primer integral
Z Z Z 2
z̄ dz = (x − iy)(dx + i dy) = y dy = 2 (3.27)
C2a C2a 0

ya que x = 0 y dx = 0 a lo largo de C2a . Por otro lado, a lo largo de C2b , y = 2


y dy = 0 ası́ que para la segunda rama, se tiene
Z Z Z 4 Z 4
z̄ dz = (x − iy)(dx + i dy) = x dx + i −2 dx = 8 − 8i (3.28)
C2b C2b 0 0

Entonces, el valor total de la integral del contorno C2 es igual a la suma de las


dos partes (3.27) y (3.28), es decir
Z
z̄ dz = 2 + 8 − 8i = 10 − 8i (3.29)
C2

Aquı́ el punto es que la integración a lo largo de dos caminos diferentes nos


dan diferentes resultados, (3.25) y (3.29) even though integramos de z = 0 a
z = 4 + 2i los dos caminos. This results foreshadows un resultado general que
es extremadamente importante, los resultados dependen upon the path taken.
Since, complex integrations often involve integrands that have nonanalytic points,
many line integrations depend upon the contour taken.
3.3 Integral Compleja en Términos de Integrales Reales. 73
Ejemplo 3.3.2. Evaluar la integral
Z
dz
(3.30)
z
a lo largo del cı́rculo unitario |z| = 1.
Solución: Elijamos la dirección en contra de las manecillas del reloj. Tenemos
y y
z = x+iy y z1 = x2 +yx x
2 −i x2 +y 2 . Entonces, u = x2 +y 2 y v = − x2 +y 2 . Escribiendo

en forma paramétrica x = cos t y y = sen t, donde 0 ≤ t < 2π. Sustituyendo en


la integral (3.30) se tiene
Z Z Z  
dz x dx + y dy y x
= +i − 2 dx + 2 dy =
|z|=1 z |z|=1 x2 + y 2 |z|=1 x + y2 x + y2
Z 2π Z 2π
− cos t sen t + sen t cos t sen2 t + cos2 t
= 2 2
dt + i dt =
0 cos t + sen t 0 cos2 t + sen2 t
Z 2π 2π
= i dt = it = 2πi (3.31)

0 0

Ejemplo 3.3.3. Evaluar la integral


Z
z n dz, n = 0, ±1, ±2, . . . (3.32)
|z|=1

Solución: Representando el cı́rculo |z| = 1 en forma paramétrica

z(t) = cos t + i sen t, 0 ≤ t < 2π,


z 0 (t) = − sen t + i cos t (3.33)

Entonces, usando la fórmula de Moivre

f [z(t)] = [z(t)]n = (cos t + i sen t)n = [cos (nt) + i sen (nt)] (3.34)

Usando la expresión
I Z 2π
f (z)dz = f [z(t)]z 0 (t)dt =
0
Z 2π
= [cos(nt) + i sen(nt)] [− sen t + i cos t] dt
0
Z 2π
= [− sen(n + 1)t + i cos(n + 1)t] dt (3.35)
0
1 2π
= [cos(n + 1)t + i sen(n + 1)t|0 = 0, si n 6= −1
n+1
R 2π
Si n = −1, entonces 0 dz z = 2πi, resultado obtenido antes (3.31). Entonces,
tenemos (
si n 6= −1
Z
n 0
z dz = (3.36)
|z|=1 2πi si n = −1
74 Integrales Complejas
Observe que para n = −2, −3, . . . la función z −n dentro de la integral no
está definida en el puntoR z = 0 y por lo tanto el teorema de Cauchy no se aplica,
sin embargo la integral |z|=1 z n dz es cero; y para n = −1 la integral es igual a
2πi.
Ejemplo 3.3.4. Determinar la siguiente integral para la lı́nea que une a (0, π)
con (1, 0). Z
ez dz (3.37)
C

Solución: Graficando la recta se obtiene:


Plano complejo z
y
(0, π)

x
(1, 0)

Figura 3.1: Recta de (0, π) a (1, 0).

La ecuación de la recta la podemos hallar usando la fórmula:


y − y1 x − x1
= (3.38)
y2 − y1 x2 − x1

Los puntos que unen a la recta son (0, π) y (1, 0). Sustituyendo estos valores en
la fórmula, encontramos la ecuación

y−π x−0 y−π


= , = x, y − π = −xπ
0−π 1−0 −π
y = π − πx, y = π(1 − x) (3.39)

Resolviendo la Integral
Z Z
z
e dz = ex+iy (dx + i dy) (3.40)
C C

De la ecuación de la recta (
y = π(1 − x)
(3.41)
dy = −πdx
3.3 Integral Compleja en Términos de Integrales Reales. 75
Sustituyendo en la integral
Z Z
x+i(π−πx)
e dx + i(−π dx) = ex+iπ−iπx dx − iπ dx
C C
Z Z
= ex+iπ−iπx dx(1 − iπ) dx = (1 − iπ) ex+iπ−iπx dx
C C
Z Z
x(1−iπ) iπ iπ
= (1 − iπ) e e dx = e (1 − iπ) ex(1−iπ) dx (3.42)
C C
eiπ (1 − iπ) x((1−iπ) 1
= e = e − eiπ = e − [cos π + i sen π] = e + 1
(1 − iπ) 0

Ejemplo 3.3.5. Determinar la siguiente integral para el segmento de recta desde


z =0 a z =1+i Z
[y − x − i(3x2 )] dx + i dy (3.43)
C
Solución: Graficando se obtiene: La ecuación de la recta la podemos hallar
usando la fórmula:
y − y1 x − x1
= (3.44)
y2 − y1 x2 − x1
Los puntos que unen a la recta son (0, 0) y (1, i). Sustituyendo estos valores en
la fórmula, encontramos la ecuación
y−0 x−0 y x
= = , y=x (3.45)
1−0 1−0 1 1
Si z = x + iy, sustituyendo, y = x, se tiene z = y + iy, derivando: dz = (1 + i) dy.
Sustituyendo en la integral
Z 1 Z 1
2
(y − x − i3x )(1 + i) dy = (y − y − i3y 2 )(1 + i) dy (3.46)
0 0
1 1
y 3 1
Z Z
2
= − i3y (1 + i) dy = −3i(1 + i) y 2 dy = −3i(1 + i)
3 0

0 0
= −i(1 + i) = 1 − i
R 2+4i
Ejemplo 3.3.6. Evaluar la integral 1+i z 2 dz para las siguientes condiciones:
a) A lo largo de la parábola x = t, y = t2 donde 1 ≤ t ≤ 2; b) A lo largo de la
recta que une 1 + i y 2 + 4i; c) A lo largo de las rectas 1 + i a 2 + i y hasta 2 + 4i
R 2+4i
Solución: a) Desarrollando la función 1+i z 2 dz se obtiene
Z 2+4i Z (2,4)
2
z dz = (x + iy)2 (dx + i dy) =
1+i (1,1)
Z (2,4)
= (x2 − y 2 + 2ixy)(dx + i dy)
(1,1)
Z (2, 4) Z (2,4)
= (x2 − y 2 ) dx − 2xy dy + i 2xy dx + (x2 − y 2 ) dy
(1, 1) (1,1)
(3.47)
76 Integrales Complejas
Ahora, los puntos (1, 1) y (2, 4) corresponden a t = 1 y t = 2, respectivamente.
Evaluando en estos puntos resulta
Z 2+4i Z 2
z 2 dz =
 2
(t − t4 ) dt − 2(t)(t2 ) dt +

1+i 1
Z 2
2(t)(t2 ) dt + (t2 − t4 )(2t) dt
 
+ i
1
Z 2 Z 2
2 4 3
 3
4t − 2t5 dt
 
= t − t − 2t dt + i (3.48)
1 1
 2  2
t3 t5 t4 t6
 
4 341
= − − +i t − =− − 6i

3 5 2 3 30
1 1

b) La lı́nea recta tiene la ecuación


y − y1 x − x1
= (3.49)
y2 − y1 x2 − x1
Sustituyendo los puntos (1, 1) y (2, 4) en la fórmula, encontramos la ecuación
y = 3x − 2 (3.50)
Sustituyendo (3.50) en (3.47) y tomando en cuenta dy = 3 dx, se tiene
Z 2+4i Z 2
z 2 dz =
 2
(x − (3x − 2)2 dx − 2x(3x − 2)(3) dx +

1+i 1
Z 2
2x(3x − 2)dx + x2 − (3x − 2)2 dx
   
+ i (3.51)
1
 2
x3 x3 x2 x3 x2

= − 9 + 12 − 4x + 18 − 12
+

3 3 2 3 2
1
 3 2 4 3 2
 2
x x x x x
+ i 6 − 4 + 3 − 27 − 36 − 12x

3 2 4 3 2
x=1
86
= − − 6i
3
c) De 1 + i a 2 + i, o de (1, 1) a (2, 1), y = 1, dy = 0 se obtiene
Z 2 Z 2
x3 2  2
x 2 4
(x2 − 1) dx + i 2x dx = − x + 2i = + 3i (3.52)

3 2 3

x=1 x=1 1 x=1

Ahora de 2 + i a 2 + 4i, o de (2, 1) a (2, 4), x = 2, dx = 0 se obtiene


Z 2 Z 2
y 2 2 y 3 2
 
2
−4y dy + i (4 − y ) dy = −4 + i 4y − = −30 − 9i (3.53)
y=1 y=1 2 1 3 1
Sumando ambos resultados se obtiene
Z 2+4i  
4 86
z 2 dz = + 3i + (−30 − 9i) = − − 6i (3.54)
1+i 3 3
3.4 Teorema de Cauchy-Goursat. 77
3.4. Teorema de Cauchy-Goursat.
Hemos aprendido a calcular las integrales complejas de lı́nea reduciéndolas a
integrales reales. En general, esta forma de calcular las integrales es algo difı́cil.
Si embargo, existen propiedades más profundas del análisis complejo que nos
permiten de una forma mas fácil calcular las integrales de funciones complejas.

Teorema 3.4.1. Cauchy-Goursat Sea f (z) una función analı́tica en un dominio


D y sea Γ una curva simple cerrada dentro de D tal que f (z) sea analı́tica sobre
y dentro de Γ. Entonces

I
f (z) dz = 0, (3.55)
Γ

el cı́rculo en la integral indica que ésta se toma en una trayectoria Γ cerrada.


En
H otras palabras, el teorema de Cauchy-Goursat establece que la integral
Γ
f (z) dz = 0 si f (z) es diferenciable en la curva y en todo punto encerrado por
ella.
El teorema de Cauchy-Goursat tiene consecuencias muy importantes y útiles.
Por ejemplo, supongamos que tenemos un cierto dominio donde f (z) es analı́tica.
Dentro de este dominio queremos evaluar una integral de lı́nea del punto A al
punto B a lo largo de dos diferentes trayectorias γ1 y γ2 . Entonces, la integral
por la trayectoria cerrada formada por la integración a lo largo de γ1 y después
a lo largo de γ2 dispuesta en dirección opuesta, es

I Z Z
f (z) dz = f (z) dz − f (z) dz = 0. (3.56)
γ1 γ2

Esto implica Z Z
f (z) dz = f (z) dz (3.57)
γ1 γ2

Debido a que γ1 y γ2 son trayectorias completamente arbitrarias, tenemos el


siguiente resultado general:
Si en un dominio, la función f (z) es analı́tica, entonces la integral entre
cualesquiera dos puntos A y B dentro del dominio es independiente de la
trayectoria. Esto significa que la integración no depende de por cual treayectoria
se realice, sino depende solo de los puntos inicial y final.
78 Integrales Complejas
Teorema 3.4.2. Primer teorema de Deformación de Contornos: El valor
de una integral de lı́nea de una función analı́tica alrededor de cierto contorno
simple cerrado permanece invariable si deformamos el contorno de tal manera
que no pasemos por encima de un punto no analı́tico (singular).
El teorema de deformación nos permite deformar una trayectoria cerrada
de integración Γ, en otra, γ, sin cambiar el valor de la integral de lı́nea de
una función diferenciable f (z). Una condición crucial para este proceso es que
ningún paso de la deformación debe pasar sobre un punto en el cual f (z) no sea
diferenciable. Esto implica que f (z) necesita ser diferenciable en ambas curvas
y en la región entre ellas. Ahora extendemos este resultado al caso en que Γ
encierra un número finito de trayectorias cerradas ajenas, es decir que no se
intersectan.
Teorema 3.4.3. Segundo teorema de Deformación de Contornos: Sea
Γ una trayectoria cerrada. Sean γ1 , γ2 , . . . , γn trayectorias cerradas dentro de Γ.
Supongamos que ningún par de trayectorias se intersectan, y que ningún punto
interior de alguna γj es interior a alguna otra γk . Sea f (z) diferenciable en un
conjunto abierto que contiene a Γ, cada γj , y todos los puntos que son interiores
a Γ y exteriores a cada γj . Entonces:

I I
f (z) dz = Σnj=1 f (z) dz. (3.58)
Γ γj

La integral de f (z) alrededor de Γ es la suma de las integrales de f (z) alrededor


de cada una de las curvas γ1 , γ2 , . . . , γn .
Ejemplo 3.4.1. Integrar la función f (z) = z −1 alrededor del contorno cerra-
do Γ, si éste consiste de un cuadrado, centrado en el orı́gen, con vértices en
(1, 1), (−1, 1), (−1, −1) y (1, −1).
Solución: La integral directa de z −1 dz por el contorno dado es muy
H

tediosa. Sin embargo, debido a que el integrando es una función analı́tica en


todos los puntos, excepto en el origen, podemos deformar el contorno original
en un cı́rculo de radio ρ, centrado en el orı́gen. Entonces, tenemos z = ρeit y
dz = iρeit dt, luego la integral es
Z 2π Z 2π
iρeit
I I
dz dz
= = dt = i dt = 2πi (3.59)
Γ z |z|=ρ z 0 ρeit 0

2
ez dz, donde Γ es una cierta trayectoria
H
Ejemplo 3.4.2. Evaluar la integral Γ
cerrada en el plano.
2
Solución: Debido a que la función f (z) = ez es analı́tica en todo el plano
(no tiene puntos singulares) y por el teorema de Cauchy-Goursat la integral
3.4 Teorema de Cauchy-Goursat. 79
deberá ser cero I
2
ez dz = 0 (3.60)
Γ
Ejemplo 3.4.3. Calcular el valor de la integral
I
2z + 1
2
dz (3.61)
Γ z + 3iz

donde Γ es el cı́rculo |z + 3i| = 2.


Solución: De la expresión (3.1) podemos ver que la función f (z) es analı́tica,
excepto en los puntos donde el denominador se anula z 2 + 3iz = 0. Estos puntos
son; 0 y −3i. Desarrollando en fracciones parciales
2z + 1 A B
f (z) = = +
z(z + 3i) z z + 3i
1 1
zA + 3iA + Bz = 2z + 1 → A= , B =2− (3.62)
3i 3i
Obtenemos  
2z + 1 1 1 1
f (z) = = + 2+ i . (3.63)
z(z + 3i) 3iz 3 z + 3i
Sustituyendo este resultado en la integral (3.1)
I I  I
2z + 1 1 dz i dz
2
dz = + 2 + (3.64)
|z+3i|=2 z + 3iz 3i |z+3i|=2 z 3 |z+3i|=2 z + 3i

Luego, debido a que z1 es diferenciable en Γ y dentro del dominio simplemente


conexo encerrado por ella, ya que z = 0 no pertenece al dominio dado y por el
teorema de Cauchy-Goursat
I
1 dz
= 0. (3.65)
3i |z+3i|=2 z

1
Sin embargo, z+3i es diferenciable en el dominio dado, excepto en el punto
z = −3i el cual pertenece al dominio dentro sel contorno Γ como se muestra
en la Figura 3.2, de tal manera que no podemos aplicar el teorema de Cauchy-
Goursat a la integral de esta función. Sin embargo, podemos calcular la integral
parametrizando, esto es, escribiendo z(t) = −3i + 2eit , entonces z + 3i = 2eit y
dz = 2ieit donde 0 ≤ t ≤ 2π. La integral se reduce a calcular
 I   Z 2π
6+i 1 6+i 1
dz = it
2ieit dt (3.66)
3 |z+3i|=2 z + 3i 3 0 2e
  Z 2π
6+i 6+i
= idt = (2πi)
3 0 3
Finalmente, tenemos el valor de la integral
I  
2z + 1 2
2
dz = − + 4i π (3.67)
|z+3i|=2 z + 3iz 3
80 Integrales Complejas
y
−2.0 −1.0 1.0 2.0
× x
(0, 0)

−i
Γ
−2i

×(0, −3i)

−4i

−5i

Figura 3.2: Contorno de integración, Γ, y ubicación de los polos, ×, de f (z).

Ejemplo 3.4.4. Calcular el valor de la integral


I
z
dz (3.68)
Γ (z + 2)(z − 4i)
donde Γ es una trayectoria que encierra a los puntos −2 y 4i.
Solución: Esta integral se puede evaluar con ayuda del segundo teorema de
deformación. Para esto coloquemos un cı́rculo γ1 alrededor del punto −2 y un
cı́rculo alrededor de 4i con radios suficientemente pequeños para que ningún
cı́rculo intersecte al otro ni a Γ y de tal manera que cada uno esté encerrado por
Γ. Entonces
I I I
z dz z dz z dz
= + (3.69)
Γ (z + 2)(z − 4i) γ1 (z + 2)(z − 4i) γ2 (z + 2)(z − 4i)
Luego, desarrollando en fracciones parciales, tenemos
   
z 1 2 1 4 2 1
= − i + + i (3.70)
(z + 2)(z − 4i) 5 5 z+2 5 5 z − 4i
Poniendo en la integral, resulta
I  I  I
z dz 1 2 dz 4 2 dz
= − i + + i
Γ (z + 2)(z − 4i) 5 5 γ z + 2 5 5 γ1 z − 4i
 I   I1
1 2 dz 4 2 dz
+ − i + + i (3.71)
5 5 γ2 z + 2 5 5 γ2 z − 4i

La segunda y tercer integral de la derecha son cero por el teorema de Cauchy(γ1


no encierra a 4i y γ2 no encierra a −2). La primer y cuarta integral son iguales
a 2iπ.
I    
z dz 1 2 4 2
= 2iπ − i + + i = 2iπ (3.72)
Γ (z + 2)(z − 4i) 5 5 5 5
3.4 Teorema de Cauchy-Goursat. 81
Ejemplo 3.4.5. Calcular el valor de la integral
I
dz
(3.73)
Γ z −a
donde Γ es cualquier trayectoria cerrada, la cual contiene al número complejo a.
Solución: No podemos parametrizar la trayectoria Γ, porque no la conocemos,
solo sabemos que es cualquier trayectoria que encierra a. Para hacer uso del
primer teorema de deformación, suponemos un cı́rculo γ de radio ρ alrededor
de a, con ρ suficientemente pequeño para que γ quede encerrada por Γ. Luego,
1
f (z) = z−a es diferenciable en todos los puntos excepto en a, es decir, en ambas
curvas y la región entre ellas. Por el teorema de la deformación, tenemos
I I
dz dz
= (3.74)
Γ z−a γ z−a

Ahora sı́ podemos parametrizar al cı́rculo representado por γ, esto es γ = a + ρeit


para 0 ≤ t ≤ 2π. Entonces
Z 2π Z 2π
iρeit dt
I
dz
= = i dt = 2πi (3.75)
γ z−a 0 ρeit 0

Finalmente, tenemos el valor de la integral (3.73)


I
dz
= 2πi (3.76)
γ z−a

Por el teorema de deformación (3.74), tenemos que el valor de la integral original


es I I
dz dz
= = 2πi (3.77)
Γ z−a γ z−a

Ejemplo 3.4.6. Evaluar la integral


I
3z + 5
dz (3.78)
|z|=1 z 2 + 2z

Solución: La función es analı́tica en todo punto, excepto en los puntos z = 0


y z = −2. Desarrollando en fracciones parciales la función
3z + 5 A B
f (z) = = +
z(z + 2) z z+2
5 1
Az + 2A + Bz = 3z + 5 → A= , B= (3.79)
2 2
Sustituyendo los valores obtenidos de las constantes A y B resulta
3z + 5 5 1
f (z) = = + . (3.80)
z 2 + 2z 2z 2(z + 2)
82 Integrales Complejas
Por consiguiente, la integral (3.78) se transforma en
I I I
3z + 5 5 dz 1 dz
2 + 2z
dz = + (3.81)
|z|=1 z 2 |z|=1 z 2 |z|=1 z +2
En la primer integral de la derecha el integrando no es una función analı́tica en
z = 0, el cual está dentro del cı́rculo, por consiguiente el teorema de Cauchy-
Goursat no se puede aplicar. No obstante, usando el resultado anterior tenemos
I
5 dz 5
= (2πi) = 5πi (3.82)
2 z 2
La segunda integral es cero, por el teorema de Cauchy-Goursat, ya que la función
es analı́tica en todo punto excepto en z = −2 pero éste esta fuera del cı́rculo.
Finalmente, tenemos I
3z + 5
2 + 2z
dz = 5πi (3.83)
|z|=1 z

3.5. Forma Integral de Cauchy.


Teorema 3.5.1. (Primera Fórmula Integral de Cauchy). Sea f (z) una función
compleja y diferenciable en un dominio D. Sea Γ una trayectoria cerrada en D
la cual encierra únicamente puntos del dominio D. Entonces, para cualquier
punto z0 encerrado por Γ se cumple la relación

I
1 f (z)
f (z0 ) = dz. (3.84)
2πi Γ z − z0

A esta fórmula se le conoce como primera fórmula integral de Cauchy.


Ejemplo 3.5.1. Hallar el valor de la integral de lı́nea dada por
z2
I
dz. (3.85)
z+i
Solución: Comparando la integral (3.85) con (3.84), tenemos que f (z) = z 2
y z0 = −i. La función dada f (z) = z 2 es una función analı́tica en todo el plano
complejo, y el punto −i está dentro de ese dominio, por lo tanto, según la
expresión (3.84), tenemos
z2
I
dz = 2πif (z) = 2iπ(−i)2 = −2πi. (3.86)

z+i z→−i

En este ejemplo podemos observar que el valor de la integral no depende del


contorno que se analice, ya que la función es analı́tica en todo el plano complejo.
Sin embargo, si deseamos graficar un contorno debemos asegurarnos que éste
encierre al punto z0 .
3.5 Forma Integral de Cauchy. 83
Ejemplo 3.5.2. Evaluar la integral
2
ez
I
dz (3.87)
Γ z−i

para cualquier trayectoria cerrada que no pase por i.


2
Solución: Tenemos que la función f (z) = ez es diferenciable ∀z ∈ D. De
las condiciones del problema, tenemos dos casos:
Primer caso: Supongamos que la trayectoria cerrada Γ no encierra al punto
i. En este caso 2
ez
I
dz = 0 (3.88)
Γ z−i
z2
e
debido al teorema de Cauchy, ya que z−i es diferenciable en Γ y dentro de ella.
Segundo caso: La trayectoria cerrada Γ encierra al punto i, entonces por la
fórmula integral de Cauchy, tenemos
2
ez
I
2
dz = 2iπei = 2iπe−1 (3.89)
Γ z−i z=i

Ejemplo 3.5.3. Evaluar la integral


I 2z
e sen(z 2 )
dz (3.90)
z−2

Solución: La función f (z) = e2z sen(z 2 ) es diferenciable ∀z ∈ D. Aquı́ tam-


bién tenemos dos casos de interés:
Primer caso: La trayectoria Γ no encierra al punto 2. Entonces, por el teorema
de Cauchy
I 2z
e sen(z 2 )
dz = 0 (3.91)
z−2
la integral es cero.

Segundo caso: Si Γ encierra al punto 2, entonces, por la primera fórmula


integral de Cuachy, tenemos
I
1 f (z)
f (z0 ) = dz = 2iπf (2) = 2iπe4 sen(4) (3.92)
2πi z − z0

Ejemplo 3.5.4. Evaluar la integral


I
cosh(iz) dz
(3.93)
|z|=2 z 2 + 4z + 3

Solución: En este ejercicio nos estan dando el dominio, el cual es un cı́rculo


de radio 2 centrado en el orı́gen |z| = 2. Dentro del cı́rculo |z| = 2 el denominador
84 Integrales Complejas
se anula en el punto z = −1. Para aplicar la primera fórmula integral de Cauchy,
reescribimos (3.93) de la siguiente manera
I I I cosh(iz)
cosh(iz) dz cosh(iz) dz z+3
= = dz (3.94)
|z|=2 z 2 + 4z + 3 |z|=2 (z + 1)(z + 3) |z|=2 z − (−1)

Aqui, el punto z0 = −1, y la función f (z) = cosh(iz)


z+3 es analı́tica en el cı́rculo
|z| ≤ 2. Por consiguiente
I  
cosh(iz) dz cosh(iz)
2
= 2πif (z0 ) = 2πi
|z|=2 z + 4z + 3 z+3

z0 =−1

= πi cosh(−i) = πi cos(1) (3.95)

donde hemos aplicado la relación cosh(iz) = cos z y la propiedad de paridad.


Ejemplo 3.5.5. Evaluar la integral
I
sen z
dz (3.96)
|z|=2 2z − π

Solución: |z| = 2 indica que la integración debe hacerse a lo largo de un


cı́rculo de radio 2, con centro en el origen. Escribiendo la integral (3.96) de la
siguiente manera I
sen z dz
(3.97)
2(z − π2 )
La función f (z) es analı́tica en todos los puntos, excepto en el punto z0 = π/2,
el cual pertenece al dominio dado. Por consiguente, podemos aplicar la primera
forma integral de Cauchy. Escribiendo la función f (z) como
1
f (z) = sen z (3.98)
2
por la forma integral, tenemos
I    
sen z dz 1 1
π = 2πi sen z = 2πi = πi (3.99)

2(z − 2 ) 2 π
z0 = 2 2

Ejemplo 3.5.6. Evaluar la integral


2
ez
I
2
dz (3.100)
z − 6z
en los siguientes dominios: A) |z − 2| = 1; B) |z − 2| = 3; C) |z − 2| = 5
Solución:
A) En el dominio limitado por el cı́rculo |z − 2| = 1 la función dentro de
la integral es una función analı́tica. Esto es debido a que los puntos singulares
(polos) son z = 0 y z = 6 y estos puntos estan fuera del cı́rculo |z − 2| = 1, ya
3.5 Forma Integral de Cauchy. 85
que −1 + 2 ≤ z − 2 ≤ 1 se convierte en 1 ≤ z ≤ 3, donde podemos ver claramente
que los puntos z0 = 0 y z0 = 6 no estan en el dominio, por lo tanto la función es
analı́tica dentro del cı́rculo dado por |z − 2| = 1. Por consiguiente, el teorema de
Cauchy afirma que la integral debe ser cero
2
ez
I
dz = 0 (3.101)
z 2 − 6z
B) En el dominio limitado por el cı́rculo |z − 2| = 3, existe solo un punto
singular z0 = 0 en el cı́rculo. Entonces, reescribiendo la integral
2
2 ez
ez
I I
z−6
dz = dz (3.102)
z 2 − 6z |z−2|=3 z
z2
e
La función f (z) definida por f (z) = z−6 es analı́tica en el dominio dado.
Aplicando la primer fórmula integral de Cauchy, resulta
2 2
!
ez ez
I  
1 πi
dz = 2πi = 2πi − =− (3.103)

2
z − 6z z − 6 z=0

6 3

C) En el dominio limitado por |z − 2| = 5 exsiten dos puntos singulares


(polos) z0 = 0 y z0 = 6, estos dos puntos estan dentro del cı́rculo, ya que el
dominio es −3 ≤ z ≤ 7. Por consiguiente, no podemos aplicar la primer forma
integral de Cauchy directamente, sino debemos proceder de la siguiente manera.
Desarrollar en fracciones parciales el denominador
1 1 A B
= = + , Az + Bz − 2A = 1 (3.104)
z 2 − 6z z(z − 6) z z−6
de la última expresión en (3.104) obtenemos los valores A = −1/6 y B = 1/6.
Sustituyendo estos valores en (3.104), la integral (6.2) se escribe como
2 2 2
ez ez ez
I I I
1 1
dz = − dz + dz (3.105)
|z−2|=5 z(z − 6) 6 |z−2|=5 z 6 |z−2|=5 z − 6
Aplicando la primer forma integral de Cauchy a cada una de las integrales en
(3.105), por separado tenemos
2
ez
I
2
dz = 2πiez = 2πie36 (3.106)
|z−2|=5 z z=6

2 2
!
ez ez
I πi
dz = 2πi =− (3.107)

z − 6 z − 6 3

|z−2|=5 z=0

Sustituyendo estos valores en (3.105), tenemos el resultado final


2
ez
I    
1 36
 1 πi πi 36 1
dz = − 2πie + − =− e + (3.108)
|z−2|=5 z(z − 6) 6 6 3 3 6
86 Integrales Complejas
Ejemplo 3.5.7. Evaluar la integral
I
dz
(3.109)
|z−i|=2 z2 + 4

Solución: La integral (3.109) la podemos escribir de la siguiente manera


I
dz
(3.110)
|z−i|=2 (z − 2i)(z + 2i)

los puntos singulares son z0 = 2i y z0 = −2i. Estos puntos estan fuera del
dominio del cı́rculo de radio 2 y centro en i, por consiguiente, aplicando el
teorema de Cauchy, la integral (3.109) será cero.

Ejemplo 3.5.8. Evaluar la siguiente integral


I  
1 2
+ dz (3.111)
|z|=4 z+1 z−3

Solución: Escribiendo la integral de la siguiente manera

3z − 1
I
dz (3.112)
|z|=4 (z + 1)(z − 3)

podemos identificar dos puntos songulares; z = −1 y z = 3, los cuales pertenecen


al dominio |z| = 4 y definiendo la función f (z) = 3z−1
z−3 . Aplicando la forma
integral de Cauchy para el primer punto singular, z = −1
 
3z − 1
I
f (z) dz
= 2πif (z) = 2πi = 2πi (3.113)

|z|=4 z + 1 z−3

z=−1 z=−1

Para el segundo caso, es decir, cuando z = 3 es el punto singular, definimos la


función f (z) = 3z−1
z+1 . Aplicando la forma integral de Cauchy, resulta
 
3z − 1
I
f (z) dz
= 2πif (z) = 2πi = 4πi (3.114)

z−3 z+1

|z|=4 z=3 z=3

La integración total se obtiene sumando ambos resultados. Finalmente, se tiene

3z − 1
I
dz = 4πi + 2πi = 6πi (3.115)
|z|=4 (z + 1)(z − 3)

Ejemplo 3.5.9. Hallar el valor de la integral


I
cos (πz) dz
(3.116)
|z|=5 (z − 1)(z − 2)
3.5 Forma Integral de Cauchy. 87
Solución: Desarrollando el denominador en fracciones parciales
1 A B
= + (3.117)
(z − 1)(z − 2) z−1 z−2
de donde Az + 2A + Bz − B = 1, de aqui resultan dos ecuaciones A + B = 0
y −2A − B = 1. Resolviendo estas ecuaciones, tenemos B = 1 y A = −1.
Sustituyendo estos valores en (3.117)
1 1 1
=− + (3.118)
(z − 1)(z − 2) z−1 z−2

Por consiguiente, la integral (3.116) se escribe como una suma de dos integrales
I I I
cos (πz) dz cos (πz) dz cos (πz) dz
=− + (3.119)
|z|=5 (z − 1)(z − 2) |z|=5 z − 1 |z|=5 z−2

Debido a que la función f (z) = cos πz es analı́tica en z = 2 y z = 1 y estos


puntos estan dentro del dominio, podemos aplicar la fórmula integral de Cauchy.
El valor de cada una de las integrales de la derecha de (3.119) es
I
cos(πz) dz
= 2πi cos(πz) = 2πi cos (2π) = 2πi (3.120)

|z|=5 z − 2 z=2

y la segunda integral
I
cos (πz) dz
= 2πi cos(πz) = 2πi cos π = −2πi (3.121)

|z|=5 z−1 z=1

Sustituyendo los valores de (3.120) y (3.121) en (3.119), tenemos el valor de la


integral I
cos (πz) dz
= −(−2πi) + 2πi = 4πi (3.122)
|z|=5 (z − 1)(z − 2)
Ejemplo 3.5.10. Evaluar la integral

sen(πz 2 ) + cos(πz 2 )
I
dz (3.123)
|z|=3 (z − 1)(z − 2)

Solución: Como primer paso encontramos los polos: z = 1 y z = 2. Haciendo


la expansión en fracciones parciales
1 A B
= + (3.124)
(z − 1)(z − 2) z−1 z−2
De aqui obtenemos las ecuaciones A + B = 0 y −2A − B = 1, resolviendo,
obtenemos los valores de A = −1 y B = 1. Sustitutendo en (3.124)
1 1 1
=− + (3.125)
(z − 1)(z − 2) z−1 z−2
88 Integrales Complejas
La integral (3.123) se transforma en dos integrales

sen(πz 2 ) + cos(πz 2 ) sen(πz 2 ) + cos(πz 2 )


I I  
dz = − dz
|z|=3 (z − 1)(z − 2) |z|=3 (z − 1)
sen(πz 2 ) + cos(πz 2 )
I  
+ dz (3.126)
|z|=3 (z + 2)

Aplicando la forma integral de Cauchy a la primer integral, se tiene

sen(πz 2 ) + cos(πz 2 )
I  

dz = 2πi sen(πz 2 ) + cos(πz 2 )
|z|=3 (z − 1) z=1

= 2πi(sen π + cos π) = −2πi (3.127)

Para la segunda integral

sen(πz 2 ) + cos(πz 2 )
I  

dz = 2πi sen(πz 2 ) + cos(πz 2 )
|z|=3 (z + 2) z=−2

= 2πi(sen(4π) + cos(4π) = 2πi (3.128)

Sustituyendo los valores obtenidos en (3.126 obtenemos el valor de la integral

sen(πz 2 ) + cos(πz 2 )
I
dz = 2πi + 2πi = 4πi (3.129)
|z|=3 (z − 1)(z − 2)

Ejemplo 3.5.11. Evaluar la integral


I
sen z
dz (3.130)
|z|=5 (z − 2)(z − 3)(z − 4)

Solución: Los puntos singulares son z = 2 z = 3 y z = 4. Haciendo la


expansión en fracciones parciales

1 A B C
= + + (3.131)
(z − 2)(z − 3)(z − 4) z−2 z−3 z−4

De donde obtenemos el siguiente sistema de ecuaciones

A+B+C = 0
−7A − 6B − 5C = 0
12A + 8B + 6C = 1 (3.132)

Al resolver este sistema se encuentra A = 12 , B = −1 y C = 12 . Sustituyendo los


resultados en (3.131)

1 1 1 1
= − + (3.133)
(z − 2)(z − 3)(z − 4) 2(z − 2) z − 3 2(z − 4)
3.5 Forma Integral de Cauchy. 89
Entonces, la integral (3.130) se escribe como
I
sen z
dz (3.134)
|z|=5 (z − 2)(z − 3)(z − 4)
I I I
sen z sen z sen z
= dz − dz + dz
|z|=5 2(z − 2) |z|=5 z − 3 |z|=5 2(z − 4)

Debido a que los puntos singulares z = 2, z = 3 y z = 4 estan encerrados en el


dominio |z| = 5 y las funciones f (z) = 12 sen z, f (z) = sen z son fanalı́ticas en
los correspondientes puntos. Por consiguiente, podemos aplicar la forma integral
de Cauchy a cada una de las integrales en (3.134)
I
sen z sen z
dz = 2πi = iπ sen(2) (3.135)
|z|=5 2(z − 2) 2 z=2

I
sen z
dz = 2πi sen z = 2πi sen(3) (3.136)

|z|=5 z − 3 z=3
I
sen z sen z
dz = 2πi = iπ sen(4) (3.137)
|z|=5 2(z − 4) 2 z=4

Sumando los valores obtenidos y factorizando se tiene el valor de la integral


(3.134)
I
sen z
dz = iπ [sen(2) − 2 sen(3) + sen(4)] (3.138)
|z|=5 (z − 2)(z − 3)(z − 4)

Ejemplo 3.5.12. Evaluar la siguiente integral

eiz dz
I
2
(3.139)
|z−i|=1 z + 1

Solución: Los puntos singulares (polos) son en z = ±i. La integral (3.139)


la podemos escribir como

eiz dz
I
(3.140)
|z−i|=1 (z + i)(z − i)

Tenemos dos puntos singulares z = ±i. Sin embargo solo el punto z0 = i esta
dentro del cı́rculo |z − i| = 1. Por lo tanto, aplicando la primer forma integral de
Cauchy, se tiene el siguiente resultado
eiz
eiz dz eiz
I I  
z+i
= dz = 2πi = πe−1 (3.141)

(z + i)(z − i) z−i z+i

|z−i|=1 |z−i|=1 z0 =i

Teorema 3.5.2. (Segunda Fórmula integral de Cauchy). Sea f (z) una función
analı́tica en un cierto dominio D. Entonces, f (z) tiene derivadas de todos los
órdenes en cada punto del dominio D. Más aún, si Γ es una trayectoria cerrada
90 Integrales Complejas
en D, la cual encierra únicamente a puntos de z0 ∈ D, y z0 es cualquier punto
encerrado por Γ, entonces

I
(n) n! f (z)
f (z0 ) = dz. (3.142)
2iπ Γ (z − z0 )n+1

La fórmula (3.84) a menudo se escribe como

I
f (z) 2πi (n)
n+1
dz = f (z0 ), (3.143)
Γ (z − z0 ) n!

donde n! representa n factorial, esto es n! = 1 · 2 · 3 . . . n, y f (n) (z0 ) representa


la n derivada respecto a z, evaluada en el punto z0 .
Ejemplo 3.5.13. Calcular la integral
3
ez
I
3
dz (3.144)
Γ (z − i)

suponiendo que la trayectoria es cerrada y que no pasa por el punto i.


Solución: Si Γ no encierra al punto i, entonces la integral es cero por el
teorema de Cauchy. Ahora supongamos que Γ encierra al punto i. Debido a que
el factor z − i aparece a la tercera potencia en el denominador, usamos n = 2 en
3
el teorema, con f (z) = ez , y obtenemos
3
ez
I
2iπ (2)
3
dz = f (i) = πif 00 (i) (3.145)
Γ (z − i) 2!
3
Luego, derivando la función f (z) = ez dos veces respecto a z, tenemos
3 3 3
f 0 (z) = 3z 2 ez , f 00 (z) = 6zez + 9z 4 ez (3.146)

Entonces, tenemos el resultado de la integral


3
ez
I
dz = iπ 6ie−i + 9e−i = (−6 + 9i)πe−i

3
(3.147)
Γ (z − i)

Ejemplo 3.5.14. Calcular la integral


e2z
I
dz (3.148)
|z|=3 (z + 1)4
3.5 Forma Integral de Cauchy. 91
Solución: La función dada es diferenciable excepto en el punto z0 = −1.
Además el punto z0 = −1 está dentro del cı́rculo |z| = 3. Por consiguiente
podemos aplicar directamente la segunda fórmula integral de Cauchy, para n = 3
e2z
I
2πi 000
4
dz = · f (z0 ) (3.149)
|z|=3 (z + 1) 3!

donde f (z) = e2z y sus derivadas f 0 = 2e2z , f 00 = 4e2z


f 000 = 8e2z |z0 =−1 = 8e−2 (3.150)
Sustituyendo (3.150) en (3.149), el resultado es
e2z
I
2πi −2 8
4
dz = 8e = πie−2 (3.151)
|z|=3 (z + 1) 3! 3
Ejemplo 3.5.15. Evaluar la integral
I
cosh z
dz (3.152)
|z|=2 (z + 1)3 (z − 1)
Solución: El denominador se anula en los puntos z = −1 y z = 1, es decir,
estos son puntos singulares. Estos puntos estan dentro del cı́rculo |z| = 2, por
consiguiente no podemos aplicar directamente la segunda fórmula integral de
Cauchy. Sin embargo, podemos desarrollar el denominador en fracciones parciales
1 A B C D
= + + + (3.153)
(z − 1)(z + 1)3 z − 1 z + 1 (z + 1)2 (z + 1)3
Ejemplo 3.5.16. Evaluar la siguiente integral
ez
I
2
dz (3.154)
|z|=2 (z − 3)(z − 1)

Solución: La función dentro del integrando tiene dos polos z0 = 3 y z0 = 1.


Sin embargo, solo el polo z0 = 1 está dentro del dominio del cı́rculo |z| = 2.
Escribiendo (3.154) de la siguiente manera
I ez dz
(z−3)
(3.155)
|z|=2 (z − 1)2
y comparando ésta integral con la segunda fórmula integral de Cauchy, podemos
identificar n = 1, z0 = 1 y f (z) = ez /(z − 3). Esta función f (z) es analı́tica
en el dominio dado, ya que el punto z0 = 3 esta fuera de |z| = 2. Aplicando
directamente la segunda fórmula integral de Cuachy, resulta
ez dz
I  z 
d e
= 2πi (3.156)

(z − 3)(z − 1) 2 dz z − 3

z=1
|z|=2
 z
ez

e 3πie
= 2πi − =−
z − 3 (z − 3)2 |z|=2 2
92 Integrales Complejas
Ejemplo 3.5.17. Evaluar la integral

cos2 z
I
dz (3.157)
|z|=1 z3

Solución: la función es analı́tica en todos los puntos, excepto en z0 = 0, el


cual está en el cı́rculo |z| = 1. Podemos ver de (3.157) n = 2, entonces usando la
segunda fórmula integral de Cauchy, se tiene

cos2 z
I
2πi 00
dz = f (z0 ) (3.158)
|z|=1 z3 2!

donde la función es f (z) = cos2 z, derivando dos veces respecto a a, tenemos



f 0 = −2 cos z sen z, f 00 = 2(sen2 z − cos2 z) = −2 (3.159)

z=0

Sustituyendo (3.159) en (3.158, finalmente el resultado es

cos2 z
I
2πi 00 2πi
3
dz = f (z0 ) = (−2) = −2πi (3.160)
|z|=1 z 2! 2!
Capı́tulo 4

Series en el Dominio Complejo

Contents
4.1. Series de Potencias . . . . . . . . . . . . . . . . . . . 93
4.2. Series de Taylor . . . . . . . . . . . . . . . . . . . . . 96
4.3. Series de Laurent . . . . . . . . . . . . . . . . . . . . 101
4.4. Ceros y Puntos Singulares Aislados de Funciones
Complejas . . . . . . . . . . . . . . . . . . . . . . . . . 103

Las series de potencias son las series más importantes en el análisis complejo.

4.1. Series de Potencias


Una serie de potencias en potencias de z − z0 es una serie de la forma

X
c0 + c1 (z − z0 ) + c2 (z − z0 )2 + . . . + . . . = cn (z − z0 )n (4.1)
n=0

donde z es la variable, c0 , c1 , . . . cn son constantes complejas dadas, llamadas


coeficientes de la serie, y z0 es una constante, llamada centro de la serie. La serie
(4.1) se llama serie de potencias en el dominio complejo. Si z0 = 0, entonces,
obtenemos un caso particular de la serie (4.1)

X
c0 + c1 z + c2 z 2 + . . . + . . . = cn z n , (4.2)
n=0
94 Series en el Dominio Complejo
esta es una serie de potencias en potencias de z. Una de las cuestiones más
importantes de las series de potencias es su convergencia.

Teorema 4.1.1. (Teorema de Abel) Si la serie de potencias (4.1) converge para


algunos valores de z igual a z0 , entonces es absolutamente convergente para todos
los valores de z tales que |z| < |z0 |. Si la serie (4.1) diverge en z = z1 , entonces
ésta es divergente para cualquier valor de z tal que |z| > |z1 |. El dominio de
convergencia de (4.1) es un cı́rculo centrado en el orı́gen de coordenadas, el cual
se llama cı́rculo de convergencia de (4.1).

Propiedades de de las Series de Potencias


Supongamos que la serie (4.1) es divergente en cierto punto z1 . Entonces,
la serie es divergente en cada punto z que satisface

|z − z0 | > |z1 − z0 | (4.3)

Para cualquier serie de potencias (4.1) existe un número R tal que en el


cı́rculo
|z − z0 | < R (4.4)
la serie (4.1) converge, y fuera del cı́rculo

|z − z0 | > R (4.5)

la serie de potencias diverge.

Si R > 0, la región mayor de convergencia para la serie es el cı́rculo |z − z0 | < R.


En la frontera |z − z0 | = R la serie (4.1) puede converger o diverger. El dominio

|z − z0 | < R, R>0 (4.6)

es llamado cı́rculo de convergencia de la serie de potencias (4.1), el número R


en (4.6) se llama radio de convergencia de la serie. Los radios de convergencia
pueden ser calculados por las fórmulas

|cn | 1
R = lı́m , cn 6= 0, R = lı́m (4.7)
n→∞ |cn+1 | n→∞ |cn |1/n

siempre y cuando los lı́mites existan, finitos o infinitos.

Ejemplo 4.1.1. Hallar el radio de convergencia de la serie de potencias



X zn
, α>0 (4.8)
n=1

4.1 Series de Potencias 95
Solución: Los coeficientes de la serie son cn = 1/nα , entonces
1 1
cn = , cn+1 = . (4.9)
nα (n + 1)α
Por la primer fórmula de (4.7) se tiene

1 α
(n + 1)α

nα 1
R= lı́m 1 = lı́m = lı́m 1 + = 1. (4.10)
n→∞
(n+1)α
n→∞ nα n→∞ n
Tenemos, entonces que el radio de convergencia es |z| < 1.
Ejemplo 4.1.2. Hallar el radio de convergencia de la serie

X
ein z n (4.11)
n=1

Solución: Los coeficientes son


cn = ein , cn+1 = ei(n+1) (4.12)
Por la fórmula (4.7), se tiene
|ein |
R = lı́m = lı́m |ei | = ei · e−i = 1 (4.13)
n→∞ |ei(n+1) | n→∞

El radio de convergencia es |z| < 1.


Ejemplo 4.1.3. Hallar el radio de convergencia de la serie

X
in z n (4.14)
n=1

Solución: Los coeficientes de la serie son


cn = in , cn+1 = in+1 (4.15)
Entonces, por la fórmula (4.7), resulta
|in |
R = lı́m = lı́m | − i| = 1 (4.16)
n→∞ in+1 | n→∞

El radio de convergencia es |z| < 1.


Ejemplo 4.1.4. Hallar el radio de convergencia de la siguiente serie

X
(1 + i)n z n (4.17)
n=0

Solución: El módulo de los coeficientes cn = (1 + i)n , esto es


√ n
|cn | = |(1 + i)n | = |1 + i|n = 2 = 2n/2 (4.18)

Usando la segunda fórmula de (4.7), obtenemos el radio de convergencia


1 1
R = lı́m =√ (4.19)
n→∞ (2n/2 )1/n 2
96 Series en el Dominio Complejo
4.2. Series de Taylor
Teorema 4.2.1. Sea f (z) una función analı́tica en un disco abierto |z −z0 | < R0
centrado en z0 y de radio R0 figura. Entonces, para todo punto z de ese disco,
f (z) admite la representación en serie de potencias

X
f (z) = cn (z − z0 )n , |z − z0 | < R0 (4.20)
n=0

donde los coeficientes cn de la serie están dados por la expresión

f (n) (z0 )
I
1 f (z) dz
cn = = , (n = 0, 1, 2, 3 . . .) (4.21)
n! 2πi Γ (z − z0 )n+1
donde Γ representa un cı́rculo centrado en z = z0 y está enteramente en la
vecindad del punto z0 donde la función f (z) es analı́tica. La serie de potencias
(4.21) converge a f (z) cuando |z − z0 | < R0 . La serie de potencias (4.20) se
conoce como serie de Taylor .

La serie de Taylor (4.20) a menudo se escribe como


f 0 (z0 ) f 00 (z0 ) f 000 (z0 )
f (z) = f (z0 ) + (z − z0 ) + (z − z0 )2 + (z − z0 )3 + . . . (4.22)
1! 2! 3!
Para el caso particular en que, z0 = 0 se tiene la serie
f 0 (0) f 00 (0) 2 f 000 (0) 3
f (z) = f (0) + z+ z + z + ... (4.23)
1! 2! 3!
y se conoce como serie de Maclariun.
Ejemplo 4.2.1. Desarrollar en serie de Taylor la función f (z) = ez alrededor
del punto z0 = 0
Solución: El desarrollo alrededor de z0 = 0 viene siendo el desarrollo en
serie de Maclairun (4.23), para esto calculamos algunas derivadas de la función
dada y la evaluamos en el punto z0 = 0.

f (0) = e0 = 1, f 0 (0) = e0 = 1, f 00 (0) = e0 = 1, f 000 (0) = e0 = 1


IV
f (0) = e0 = 1, f V (0) = e0 = 1, f V I (0) = e0 = 1 (4.24)

sustituyendo en la serie (4.23) tenemos el desarrollo deseado



1 1 1 1 1 1 X 1 n
ez = 1 + z + z2 + z3 + z4 + z5 + z6 + . . . = z (4.25)
1! 2! 3! 4! 5! 6! n=0
n!

Este desarrollo lo podemos escribir en una forma mas conveniente



z
X 1 n
e = z (4.26)
n=0
n!
4.2 Series de Taylor 97
Ejemplo 4.2.2. Hallar el desarrollo en serie de Taylor de la función f (z) = ez
alrededor del punto z0 = i.
Solución: Del ejemplo anterior, sabemos que para todo z se cumple el
desarrollo (4.26). Para desarrollar alrededor del punto z0 = i, la serie de potencias
debe estar en términos de potencias de z − i. Para esto escribimos
∞ ∞
X ei X 1
ez = ez−i+i = ei ez−i = (z − i)n = ei (z − i)n (4.27)
n=0
n! n=0
n!

Este es el desarrollo de la función f (z) = ez alrededor del punto z0 = i. La serie


obtenida (4.27) converge para todo z.
Ejemplo 4.2.3. Hallar el desarrollo en serie de Maclairun de la función f (z) =
cos z.
Solución: Para esto calculamos algunas derivadas y las evaluamos en el
punto z0 = 0. Esto es

f (0) = cos 0 = 1, f 0 (0) = − sen 0 = 0, f 00 (0) = − cos 0 = −1


000 IV
f (0) = sen 0 = 0, f (0) = cos 0 = 1, f V (0) = − sen 0 = 0
f V I (0) = − cos 0 = −1 (4.28)

Sustituyendo estos resultados en la fórmula (4.23), tenemos


1 2 1 1
cos z = 1 − z + z4 − z6 + . . . (4.29)
2! 4! 6!
Esta expresión la podemos escribir de la siguiente manera

X (−1)n 2n
cos z = z (4.30)
n=0
(2n)!

Ejemplo 4.2.4. Desarrollar la función f (z) = z 4 en serie de Taylor en potencias


de z − i
Solución: Tenemos la función y sus correspondientes derivadas

f (z) = z4, f 0 (z) = 4z 3 , f 00 (z) = 12z 2 , f 000 (z) = 24z, f IV (z) = 24


f V (z) = f V I (z) = . . . = 0. (4.31)

El siguiente paso es evaluar la función y sus derivadas en el punto z0 = i

f (i) = i4 = 1, f 0 (i) = 4i3 = −4i, f 00 (i) = 12i2 = −12


000
f (i) = 24i, f IV (z) = 24 f V (z) = f V I (z) = . . . = 0. (4.32)

Sustituyendo en la serie de Taylor (4.22), obtenemos el desarrollo


4i 12 24i 24
f (z) = 1 − (z − i) − (z − i)2 + (z − i)3 + (z − i)2 . (4.33)
1! 2! 3! 4!
98 Series en el Dominio Complejo
1
Ejemplo 4.2.5. Desarrollar en serie de Maclairun la función f (z) = 1−z

Solución: La serie está dada por la expresión



1 X
= 1 + z + z2 + z3 + z4 + . . . + = zn, |z| < 1 (4.34)
1−z 0

Para el caso

1 X
= 1 − z + z2 − z3 + z4 − . . . + = (−1)n z n , |z| < 1 (4.35)
1+z 0

Ejemplo 4.2.6. Desarrollar en serie de Maclairun la función f (z) = sen z

Solución: Tenemos que las derivadas evaluadas en z0 = 0, tienen la forma


general
f (2n) (0) = 0, f (2n+1) (0) = (−1)n , n = 0, 1, 2, . . . (4.36)
Entonces, el desarrollo tiene la forma

z3 z5 X (−1)n 2n+1
sen z = z − + − ... = z (4.37)
3! 5! n=0
(2n + 1)!

Sabemos que la expresión senh z = −i sen (iz), es suficiente cambiar z por iz


en la expresión (4.37) y multiplicando por −i, tenemos el desarrollo en serie de
Maclairun de la función seno hiperbólico

X z 2n+1
senh z = , |z| < ∞ (4.38)
n=0
(2n + 1)!

Ejemplo 4.2.7. Hallar la serie de Maclairun de la función


z
f (z) = (4.39)
z4 +9

Solución: La función (4.39), la podemos escribir de la siguiente manera



" #  4 n
z z 1 zX n z
f (z) = 4
= z 4 = (−1) =
z +9 9 1+( 9 ) 9 n=0 9

X (−1)n 4n+1 √
= z , |z| < 3 (4.40)
n=0
3n+2

Ejemplo 4.2.8. Hallar el desarrollo en serie de Maclairun de la función

f (z) = z 2 e3z (4.41)


4.2 Series de Taylor 99
Solución: Usando el desarrollo en serie de Maclairun de la función exponen-
cial ez , donde debemos cambiar a z por 3z, resulta

3z
X z 3n
e = (4.42)
n=0
n!

El siguiente paso es multiplicar este resultado por z 2 , finalmente, tenemos



X 3n n+2
z 2 e3z = z (4.43)
n=0
n!

Ejemplo 4.2.9. Hallar la expansión de Maclairun y el radio de convergencia


de la función
1
f (z) = (4.44)
(z + 1)(z + 2)
Solución: Usando fracciones parciales, la función (4.44) la podemos escribir
de la siguiente manera
1 1 1 1 1 1
= − = − (4.45)
(z + 1)(z + 2) z+1 z+2 1 − (−z) 2 1 − (−z/2)

Usando la fórmula de la serie geométrica, tenemos


∞ ∞ ∞ 
(−1)n

X 1 X  z n X
f (z) = (−z)n − − = (−1)n − n+1 z n =
n=0
2 n=0 2 n=0
2
∞  
X 1
= (−1)n 1 − n+1 z n (4.46)
n=0
2

1

El coeficiente de la serie (−1)n 1 − 2n+1 . Para hallar el radio de convergencia
hacemos uso de la fórmula
  1/n
1 (−1)n 1 − 1

R = lı́m = lı́m n+1
=1 (4.47)
n→∞ |cn |1/n n→∞ 2

Entonces, el radio de convergencia de la serie dada es R = 1.

Ejemplo 4.2.10. Hallar los primeros tres términos en la serie de potencias


alrededor del punto z = 1 de la función

z2
f (z) = (4.48)
z−2
Solución: Para hallar la expansión alrededor de z0 = 1 debemos primero
modificar la función (4.48), de la siguiente manera
2
z 2 = [(z − 1) + 1] = (z − 1)2 + 2(z − 1) + 1 (4.49)
100 Series en el Dominio Complejo
y
z − 2 = − [1 − (z − 1)] (4.50)
De esta manera podemos escribir la función (4.48) como

" #
X
2 k
 
f (z) = (z − 1) + 2(z − 1) + 1 1 − (z − 1) (4.51)
n=0

Por consiguiente, los primeros tres términos son


f (z) = −1 − (1 + 2)(z − 1) − (1 + 2 + 1)(z − 1)2 − (1 + 2 + 1)(z − 1)3 (4.52)
Ejemplo 4.2.11. Hallar los primeros tres términos en la serie de potencias en
el punto z0 = 1 de la función
z−2
f (z) = (4.53)
(z + 3)(z + 2)
Solución: Haciendo la descomposición parcial tenemos
5 4
f (z) = − (4.54)
z+3 z+2
Luego, escribiendo
 
5 5 1 5 1 1 2 1 3
= = 1 − (z − 1) + (z − 1) − (z − 1) (4.55)
z+3 4 1 − 14 (z − 1) 4 4 42 43
Similarmente, tenemos para el segundo término en (4.54)
 
4 4 1 1 1
= 1 − (z − 1) + 2 (z − 1)2 − 3 (z − 1)3 (4.56)
z+2 3 3 3 3
Por consiguiente, hallamos que los primeros tres términos de la expansión en el
punto z0 = 1 de la función (4.53) esta dada por
   
z−2 5 4 5 4
f (z) = = − − − (z − 1) +
(z + 3)(z + 2) 4 3 42 32
   
5 4 2 5 4
+ − (z − 1) − − (z − 1)3 (4.57)
43 33 44 34
Ejemplo 4.2.12. Desarrollar las funciones a) cos z y b) sen z en serie de po-
tencias alrededor de z0 = π/2.
Solución: a) Tomando en cuenta que cos z = − sen(z − π/2) para todo
z ∈ C, entonces

X (−1)n+1  π 2n+1
cos z = z− (4.58)
n=0
(2n + 1)! 2
0
rmb) De manera similar, se tiene sen z = cos(z − π/2)

X (−1)n  π 2n
sen z = z− (4.59)
n=0
(2n)! 2
4.3 Series de Laurent 101
4.3. Series de Laurent
Supongamos ahora que la función f (z) no es analı́tica en todo el disco sino
solamente en un anillo A limitado por dos cı́rculos concéntricos γ1 : |z − z0 | = R1
y γ2 : |z − z0 | = R2 , 0 < R1 < R2 , para cierto punto z0 figura. Entonces, en
general podemos desarrollar la función f (z) en series de potencia, algunas de
las cuales o todas pueden ser negativas. Una serie de potencias que contenga
potencias positivas y negativas se llama series de potencias de Laurent.
Para un punto z0 , la serie de potencias

X
cn (z − z0 )n , n∈Z (4.60)
−∞

está definida por


∞ ∞
X X bn
an (z − z0 )n + . (4.61)
n=0 n=1
(z − z0 )n
Esta serie es la serie de Laurent si las dos series en (4.61) convergen.
Teorema 4.3.1. (Teorema de Laurent) Si f (z) es una función de una variable
compleja z y analı́tica en el anillo R1 < |z − z0 | < R2 para algún punto z0
entonces, f (z) es igual a la serie de Laurent (4.61)
∞ ∞
X X bn
f (z) = an (z − z0 )n + , (4.62)
n=0 n=1
(z − z0 )n

donde
f (n) (z0 )
Z
1 f (ζ) dζ
an = = , n = 0, 1, 2, 3 . . . (4.63)
n! 2πi γ2 (ζ − z0 )n+1
Z
1 f (ζ) dζ
bn = , n = 1, 2, 3 . . . (4.64)
2πi γ1 (ζ − z0 )−n+1
La integración sobre γ1 y γ2 es en la dirección positiva, es decir, en contra de
las manecillas del reloj.
La serie de Laurent (4.62) es la generalización de la serie de Taylor (5.13).
Si f (z) es analı́tica dentro y sobre γ2 , entonces todas las bn serán cero por el
teorema de Cauchy ya que los integrandos son funciones analı́ticas dentro y sobre
γ1 .
Ejemplo 4.3.1. Sea la función
1
f (z) = (4.65)
(z − 1)(z − 2)

desarrollar en serie de Laurent para f (z)


1. en el disco |z| < 1
102 Series en el Dominio Complejo
2. en el anillo 1 < |z| < 2

3. en la región 2 < |z|

Solución: Desarrollando en fracciones parciales la función (4.65), podemos


escribir
1 1 1
=− + (4.66)
(z − 1)(z − 2) z−1 z−2
1. Para el caso |z| < 1 tenemos

1 1 X
− = = zn (4.67)
z−1 1 − z n=0

y

1 1 1 1 X  z n
=− z =− (4.68)
z−2 2 1− 2 2 n=0 2

De esta manera, tenemos el desarrollo en serie de Laurent de (4.65)


∞ ∞
1 X 1 X  z n
f (z) = = zn − (4.69)
(z − 1)(z − 2) n=0 2 n=0 2

cuando |z| < 1.


2. Si 1 < |z| < 2, se tiene

1 1 1 1X 1
− =− 1 =− (4.70)
z−1 z 1− z
z n=0 z n

y

1 1 X  z n
=− (4.71)
z−2 2 n=0 2

De esta manera, el resultado es


∞ ∞
1 X 1 1 X  z n
f (z) = =− − (4.72)
(z − 1)(z − 2) n=0
z n+1 2 n=0 2

3. Si 2 < |z|, entonces


∞  n
1 1 1 1X 2
= = (4.73)
z−2 2 1 − z2 z n=0 z

De esta manera
∞ ∞  n
1 X 1 1X 2
f (z) = =− + (4.74)
(z − 1)(z − 2) n=0
z n+1 z n=0 z
4.4 Ceros y Puntos Singulares Aislados de Funciones Complejas 103
Ejemplo 4.3.2. Desarrollar en serie de Laurent la función

z+1
f (z) = (4.75)
z−1

en las regiones a). |z| < 1 y b). |z| > 1

Solución: Para a)., tenemos

z+1
= −(z + 1) 1 + z + z 2 + . . . ,

f (z) = − |z| < 1 (4.76)
1−z

Para b)., resulta


  
1 z+1 1 1 1
f (z) = = 1 + 1 + + + . . . , |z| > 1 (4.77)
z 1 − ( z1 ) z z z2

Ejemplo 4.3.3. Hallar la serie de Laurent para la función

1
f (z) = (4.78)
z 2 (1 − z)

para las regiones a) 0 < |z| < 1, b) |z| > 1

Solución: Para el caso a), podemos escribir

1 1 1 1 1
= 2 1 + z + z 2 + z 3 + . . . = 2 + + 1+z +z 3 +. . . (4.79)

f (z) = 2
z 1−z z z z

Para b) |z| > 1, se tiene


 
1 1 1 1 1 1 1 1
f (z) = − =− 3 1 + + 2 + . . . = − 3 − 4 − 5 − . . . (4.80)
z 3 1 − ( z1 ) z z z z z z

4.4. Ceros y Puntos Singulares Aislados


de Funciones Complejas
Sea f (z) una función analı́tica en el punto z0 . Entonces, se dice que el punto
z0 es un cero de orden n de f (z), si

f (z0 ) = 0, f 0 (z0 ) = 0, f 00 (z0 ) = 0, . . . , f (n−1) (z0 ) = 0,


f (n) (z0 ) 6= 0
(4.81)
En particular, si (n = 1) al punto z0 se le llama cero simple de f (z).

Ejemplo 4.4.1. Hallar los ceros de la función f (z) = z 4 + 4z 2 y determinar su


orden.
104 Series en el Dominio Complejo
Solución: Debemos hallar los puntos que satisfacen la relación f (z) = 0,
esto es

f (z) = z 4 + 4z 2 = 0 → z 2 (z 2 + 4) = 0 → z = 0, z = ±2i (4.82)

Calculando la derivada de f (z) y evaluando en el punto z = 0 de (4.82), resulta

f 0 (0) = 4z 3 + 8z z=0 = 0

(4.83)

Debido a que la derivada evaluada en z = 0 es cero, entonces, hacemos la segunda


derivada de f (z) y la evaluamos en z = 0, tenemos
 
f 00 (0) = 12z 2 + 8 = 8 6= 0 (4.84)
z=0

Concluimos, entonces que z = 0 es un cero de orden 2, ya que la segunda derivada


es la que no se anula en z = 0.
Tomamos los otros ceros de la función. Estos son z = ±2i. Primero evaluamos
la derivada en z = 2i, tenemos

f 0 (2i) = 4z 3 + 8z z=2i = 4(2i)3 + 8(2i) = −16i 6= 0



(4.85)

Para el punto z = −2i, tenemos

f 0 (−2i) = 4z 3 + 8z z=2i = 4(−2i)3 + 8(−2i) = 16i 6= 0



(4.86)

Conluimos, que los puntos z = ±2i son ceros simples, ya que la primer derivada
de la función evaluada en estos ceros es diferente de cero.
Ejemplo 4.4.2. Hallar los ceros de la función f (z) = z 2 sen z y determinar el
orden.
Solución: Debemos hallar los puntos que satisfacen la relación f (z) = 0.
Tenemos

f (z) = z 2 sen z = 0 → z = 0, sen z = 0, zn = nπ, n = ±1, ±2 . . . (4.87)

Los ceros de la función dada, estan en z = 0 y zn = nπ. Calculando la derivada


de la función y evaluando en el cero z = 0, se tiene

f 0 (0) = 2z sen z + z 2 cos z z=0 = 0



(4.88)

Como el resultado es cero, entonces debemos calcular la segunda derivada y


evaluarla en el mismo cero z = 0, tenemos

f 00 (0) = 2 sen z + 2z cos z + 2z cos z − z 2 sen z z=0 = 0



(4.89)

Obtenemos, nuevamente cero. Entonces, calculamos la tercer derivada evaluada


en z = 0, resulta

f 000 (0) = 2 cos z + 4 cos z − 4z sen z − 2z sen z − z 2 cos z z=0 = 6 6= 0 (4.90)



4.4 Ceros y Puntos Singulares Aislados de Funciones Complejas 105
Debido a que la tercer derivada es la única que no se anula en el cero z = 0,
concluimos que el cero z = 0 de la función f (z) es de orden tres.
Sustituyendo el cero zn = nπ en (4.88), resulta
 
f 0 (nπ) = 2z sen z + z 2 cos z = 2nπ sen (nπ) + n2 π 2 cos(nπ)
z=nπ
2 2 n
= n π (−1) 6= 0, n = ±1, ±2 . . . (4.91)

Entonces, podemos decir, que el cero zn = nπ es un cero simple o de orden uno.


El punto z0 es un cero de orden n de una función analı́tica f (z) en z0 , si y
sólo si en la vecindad (cercanı́a) del punto z0 se cumple

f (z) = (z − z0 )n φ(z) (4.92)

donde φ(z) es una función analı́tica en z0 y φ(z0 ) 6= 0.

Ejemplo 4.4.3. Hallar el orden del cero z0 = 0 de la función.

z8
f (z) = (4.93)
z − sen z
Solución: Para esto desarrollamos en serie de Taylor la función sen z alrede-
dor del punto z0 = 0

z8 z8 z8
f (z) = = 3 z5
= z3 z5
z − sen z z − z − z3! + 5! − ... 3! − 5! + ...
5
z 1
= 1 z2
= z5 1 z2
= z 5 φ(z) (4.94)
3! − 5! + ... 3! − 5! + ...

donde
1
φ(z) = 1 z2
(4.95)
3! − 5! + ...
es una función analı́tica en z0 = 0 y φ(0) = 6 6= 0. Comparando con la relación
(4.92), podemos concluir que el punto z0 = 0 de la función f (z) (4.93) es de
quinto orden.

Ejemplo 4.4.4. Hallar los ceros de la función f (z) = (z 2 + 1)3 senh z y deter-
minar el orden.

Solución: Para determinar los ceros de la función hacemos f (z) = 0 y


obtenemos
(z 2 + 1)3 senh z = 0 → z 2 + 1 = 0, senh z = 0 (4.96)
Resolviendo las dos últimas ecuaciones en (4.96), tenemos que los ceros de la
función f (z) son:

z = ±i, z = nπi (n = 0, ±1, ±2, . . .) (4.97)


106 Series en el Dominio Complejo
Tomamos el segundo cero de (4.97), z = −i. De la expresión (4.92), podemos
escribir
f (z) = (z + i)3 φ(z) (4.98)
donde la función φ(z) = (z − i)3 senh z es una función analı́tica en z = −i y

φ(−i) = 8i senh i = −8 sen 1 6= 0 (4.99)

Esto implica que el punto z = −i es un cero de tercer orden de f (z). De manera


análoga se puede verificar que el punto z = i es un cero de tercer orden de f (z).
Ahora analizaremos los ceros z = nπi. Para esto derivamos la función y la
evaluamos en z = nπi, resulta

f 0 (nπi) = 6z(z 2 + 1)2 senh z + (z 2 + 1)3 cosh z z=nπi 6= 0


 
(4.100)

Esto implica que los ceros dados por z = nπi son ceros simples o de primer
orden.
Se dice que el punto z0 es un punto singular aislado de una función f (z) si
existe una vecindad de este punto en la cual la función es analı́tica en todo lugar
excepto en z = z0 . Un punto singular aislado de una función f (z) es removible si
en el punto z = z0 la función f (z) tiene un lı́mite finito. Se dice que el punto z0
de una función f (z) es un polo de f (z) si la función tiende al infinito conforme
z → z0 . El punto z0 es un polo de orden n (n ≥ 1) de f (z) si éste es un cero de
1
orden n de la función φ(z) = f (z) . Si n = 1, decimos que el polo es simple. En
otras palabras, z0 es un polo de orden n de f (z) si y sólo si, podemos escribir
φ(z)
f (z) = (4.101)
(z − z0 )n
donde φ(z) es una función analı́tica en z0 , y φ(z0 ) 6= 0. El punto z0 se llama
singularidad esencial de la función f (z) si no existe el lı́mite de f (z) conforme
z → z0 .
Ejemplo 4.4.5. Analizar los puntos de la función
ez − 1
f (z) = (4.102)
z
Solución: La función (4.102) tiene un solo punto singular en z = 0. Veamos
si es o no removible, para esto calculamos el lı́mite de la función f (z)
d z
ez − 1 dz (e − 1)
lı́m f (z) = lı́m = lı́m d
= lı́m ez = 1 (4.103)
z→0 z→0 z z→0
dz (z)
z→0

El lı́mite de la función es finito y por consiguiente el punto singular z = 0 de la


función (4.102) es removible.
Ejemplo 4.4.6. Analizar los puntos singulares de la siguiente función
sen z
f (z) = (4.104)
z3 + z2 − z − 1
4.4 Ceros y Puntos Singulares Aislados de Funciones Complejas 107
Solución: El denominador de la función lo podemos escribir como

z 3 + z 2 − z − 1 = (z − 1)(z + 1)2 = 0 (4.105)

entonces los puntos singulares de la función son dos; z1 = 1 y z2 = −1. Tomando


el punto z1 = 1 y escribiendo
sen z
sen z (z+1)2
f (z) = 2
= (4.106)
(z − 1)(z + 1) z−1

Comparando con (4.101), identificamos la función


sen z
φ(z) = (4.107)
(z + 1)2

la cual es analı́tica en el punto z1 = 1, y φ(1) 6= 0. Concluimos de esta manera


que el punto z1 = 1 es un polo de orden uno ó simple de la función (4.104). De
igual manera, para el punto z2 = −1, escribimos la función (4.104) como
sen z
z−1
f (z) = (4.108)
(z + 1)2
de donde identificamos la función
sen z
φ(z) = (4.109)
z−1
la cual es analı́tica en el punto z2 = −1, y φ(−1) 6= 0. Concluimos que el punto
z2 = −1 es un polo de segundo orden para la función (4.104).
Ejemplo 4.4.7. Determinar el tipo de punto singular z = 0 de la función
1
f (z) = (4.110)
2 + z 2 − 2 cosh z
Solución: El punto singular z = 0 es un polo de (4.110) ya que la función
f (z) tiende a infinto conforme z → 0, ( 10 → ∞). Sea

1
φ(z) = = 2 + z 2 − 2 cosh z (4.111)
f (z)

El punto z = 0 es un cero para la función (4.111), ya que φ(0) = 0. Nos queda


hallar el orden del cero de la función φ(z), para esto calculamos las derivadas de
φ evaluadas en z = 0

φ0 (0) = (2z − 2 senh z)z=0 = 0, φ00 (0) = (2 − 2 cosh z)z=0 = 0(4.112)


0000 IV
φ (0) = (−2 senh z)z=0 = 0, φ (0) = (−2 cosh z)z=0 = −2 6= 0

Debido a que la derivada de cuarto orden es la única diferente de cero, implica


que el cero de φ(z) (4.111) es de orden cuatro. Por consiguiente, el punto singular
de la función dada (4.110) es un polo de orden cuatro.
108 Series en el Dominio Complejo
Ejemplo 4.4.8. Determinar el tipo de punto singular z = 1 de la función
sen πz
f (z) = (4.113)
2ez−1 − 2z
Solución: Sea
1 2ez−1 − 2z
φ(z) = = (4.114)
f (z) sen πz
Ahora, escribiendo el numerador de (4.114) como

ψ1 = 2ez−1 − 2z (4.115)

Desde luego que el punto z = 1 es una cero de (4.115). Para determinar el orden
derivamos y evaluamos en el punto z = 1

ψ10 = 2ez−1 − 2 z=1 = 0



(4.116)

Derivamos una vez más y evaluamos en z = 1



ψ100 = 2ez−1 = 2 6= 0 (4.117)

z=1

Entonces, decimos que el cero z = 1 de la función (4.115) es de segundo orden.


Luego, representadno el denominador de (4.114) como

ψ2 = sen nπ (4.118)

El punto z = 1 es tambien un cero para (4.118). Para saber de que orden


derivamos y evaluamo en z = 1. Tenemos

ψ10 (1) = − cos z = − cos(1) 6= 0 (4.119)

z=1

Lo que implica que z = 1 es un cero simple o de primer orden para la función ψ2 .


Finalmente, sustituyendo estos resultados en la función dada por (4.114),

1 2ez−1 − 2z
φ(z) = = (4.120)
f (z) sen πz

tenemos que el orden del cero z = 1 para φ(z) es (2 − 1 = 1), es decir, un cero
simple (lo anterior resulta del hecho que el numerador de (4.120) es de orden
2 y el denominador de orden 1). Por consiguiente el punto z = 1 es un polo de
primer orden o polo simple de la función f (z) en (1.45).
Capı́tulo 5

Series de Taylor y de Laurent:


Singularidades

Contents
5.1. Series de Potencias . . . . . . . . . . . . . . . . . . . 109

5.1. Series de Potencias


La suma de cualquier serie de potencias, con radio de convergencia positivo,
es una función analı́tica. Recı́procamente, toda función analı́tica f (z) se puede
representar mediante series de potencias, llamadas series de Taylor de f (z), y
tienen la misma forma que en el cálculo real, con x sustituida por el número
complejo z.
Sea f (z) una función analı́tica en una vecindad de un punto z = z0 , primero
obtendremos la fórmula de Taylor y a partir de esta obtenedremos la serie de
Taylor de f (z) con centro en z0 .
La herramienta crucial para obtener la fórmula de Taylor es la fórmula
integral de Cauchy, escribiendo ζ y z en lugar de z y z0 , tenemos
I
1 f (ζ)
f (z) = dζ (5.1)
2πi Γ ζ − z
z esta dentro de Γ, por lo cual en tal vecindad se considera un cı́rculo de radio r
110 Series de Taylor y de Laurent: Singularidades
con centro en z0 . ζ es la variable compleja de integración. La idea es desarrollar
1
ζ−z en potencias de z − z0 . Esto es

1 1 1
= =   (5.2)
ζ −z ζ − z0 − (z − z0 ) (ζ − z0 ) 1 − z−z0
ζ−z0

Podemos observar que como ζ está sobre Γ, mientras z está en Γ, se tiene



z − z0
ζ − z0 < 1 (5.3)

Usando la fórmula de la serie geométrica


1 q n+1
= 1 + q + . . . + qn + (5.4)
1−q 1−q
z−z0
Si ponemos que q = ζ−z0 y aplicando la fórmula, tenemos

1
=
ζ −z
"  2  3  n #
1 z − z0 z − z0 z − z0 z − z0
= 1+ + + + ... + +
ζ − z0 ζ − z0 ζ − z0 ζ − z0 ζ − z0
 n+1
1 z − z0
+ (5.5)
ζ − z ζ − z0
Sustituyendo en la fórmula (5.1), se tiene
z − z0
I I
1 f (ζ) f (ζ) dz
f (z) = dζ + + ... +
2πi Γ ζ − z0 2πi Γ (ζ − z0 )2
(z − z0 )n
I
f (ζ) dz
+ + Rn (z) (5.6)
2πi Γ (ζ − z0 )n+1
donde el último término está dado según la fórmula
(z − z0 )n+1
I
f (ζ)
Rn (z) = n+1 (ζ − z)
dζ (5.7)
2πi Γ (ζ − z0 )
Usando la fórmula integral de Cauchy
I
n! f (z)
f (n) (z0 ) = dz (5.8)
2πi Γ (z − z0 )n+1
De esta fórmula tenemos las siguientes expresiones, para n = 0, 1, 2
I
1 f (z)
f (z0 ) = dz (5.9)
2πi Γ (ζ − z0 )
I
0 1! f (z)
f (z0 ) = dz (5.10)
2πi Γ (ζ − z0 )2
5.1 Series de Potencias 111

I
2! f (z)
f 00 (z0 ) = dz (5.11)
2πi Γ (ζ − z0 )3
Sustituyendo estas derivadas en (5.6), tenemos
z − z0 0 (z − z0 )2 00 (z − z0 )n (n)
f (z) = f (z0 ) + f (z0 ) + f (z0 ) + . . . + f (z0 ) + Rn (z)
1! 2! n!
(5.12)
Esta representación se llama Fórmula de Taylor y Rn (z) se llama el residuo. La
función analı́tica f (z) tiene derivadas de todos los ordenes, entonces n se puede
tomar tan grande como se desee. Si dejamos que n se pueda considerar como
infinito a partir de la fórmula anterior se obtiene la serie de potencias

X f (m) (z0 )
f (z) = (z − z0 )m (5.13)
n=0
m!

Esta serie se llama serie de Taylor de f (z) con centro en z0 . El caso particular,
cuando z0 = 0 la serie se llama de Maclaurin de f (z).
Es claro que la serie de Taylor, converge y representa a f (z) si sólo si
lı́m Rn (z) = 0 (5.14)
n→∞

Teorema 5.1.1. Si f (z) es una función de una variable compleja z la cual es


analı́tica en un disco de radio R centrado en z0 , entonces la serie de Taylor para
f (z) alrededor de z0 converge a f (z)
f 00 (z0 )
f (z) = f (z0 ) + f 0 (z0 )(z − z0 ) + (z − z0 )2 + . . . + . . . =
2!

X
= an (z − z0 )n (5.15)
n=0

donde
1 (n)
an = f (z0 ) (5.16)
n!
O por la forma integral de Cauchy
I
1 f (z) dz
an = (5.17)
2πi Γ (z − z0 )n+1
Provided |z − z0 | < R.
En (5.17) la integración se hace en sentido contrario a las manecillas del reloj
alrededor de una trayectoria simple cerrada que contenga en su interior a z0 .
1
Ejemplo 5.1.1. Serie geométrica. Sea f (z) = 1−z . Entonces se tiene f (n) =
n! (n) 1
(1−z)n+1 , f (0) = n!, por tanto, el desarrollo de Maclaurin de 1−z es la serie
geométrica
n
∞z
1 X
= = 1 + z + z2 + . . . (5.18)
1−z n=0
f (z) es singular en z = 1; este punto se encuentra sobre el cı́rculo de convergencia.
112 Series de Taylor y de Laurent: Singularidades
Ejemplo 5.1.2. La función exponencial. Se sabe que la función exponencial
ez es analı́tica para toda z y (ez )0 = ez . Ası́, a partir de (5.13) con z0 = 0 se
obtiene la serie de Maclaurin

X zn z2 z3 zn
ez = =1+z+ + + ... + , |z| < ∞ (5.19)
n=0
n! 2! 3! n!

Esta serie también se obtiene si x se sustituye por z en la conocida serie de


Maclaurin de ex .

Ejemplo 5.1.3. Funciones trigonométricas e hiperbólicas. Al sustituir la


serie exponencial (5.19) en la serie geométrica (5.18) se obtienen las series de
las funciones trigonométricas mas importantes, éstas son:

X z 2n z2 z4
cos z = (−1)n =1− + − ... + ..., |z| < ∞ (5.20)
n=0
(2n)! 2! 4!

X z 2n+1 z3
sen z = (−1)n =z− + ... − ..., |z| < ∞ (5.21)
n=0
(2n + 1)! 3!

X z 2n−1 z3 z5
tan−1 (z) = (−1)n−1 =z− + − ... + ..., |z| <(5.22)
1
n=0
2n − 1 3 5

Cuando z = x, las expresiones anteriores son las conocidas series de Maclaurin de


las funciones reales cos(x), sen(x) y tan−1 (x). De manera semejante al sustituir
la serie exponencial en la serie geométrica se obtiene:

X z 2n z2 z4
cosh z = =1+ + + ... (5.23)
n=0
(2n)! 2! 4!

X z 2n+1 z3 z5
senh z = =z+ + + ... (5.24)
n=0
(2n + 1)! 3! 5!

Ejemplo 5.1.4. Logaritmo. Con base en la ecuación 5.12 se concluye que:

zn z2 z3
ln(1 + z) = (−1)n−1 =z− + − ... + ..., |z| < 1 (5.25)
n 2 3

Ejemplo 5.1.5. Fórmula binomial. Si (1 + z)p es multı́voca el resultado es


válido para la rama de la función que toma el valor de 1 cuando z = 0.

p(p − 1) 2 p(p − 1) · · · (p − n + 1) n
(1 + z)p = 1 + pz + z +...+ z +... |z| < 1
2! n!
(5.26)

Ejemplo 5.1.6. Determinar la serie de Taylor para la siguiente función, f (z) =


ez , donde z0 = 0.
5.1 Series de Potencias 113
Solución:
f 0 (z0 ) f 00 (z0 ) f 000 (z0 )
f (z) = f (z0 ) + (z − z0 ) + (z − z0 )2 + (z − z0 )3 + . . . (5.27)
1! 2! 3!
Como al derivar la función exponencial se obtiene la misma función. Sustituyendo
z0 = 0 se obtiene
e(0) e(0) e(0)
f (z) = e(0) + (z − 0) + (z − 0)2 + (z − 0)3 + . . . (5.28)
1! 2! 3!
z2 z3
f (z) = 1 + z + + + ... (5.29)
2 6
Ejemplo 5.1.7. Desarrollar en serie de Maclaurin las funciones:
A) log(z + 1):
Solución:
z2 z3 zn
log(z + 1) = z − + − . . . + (−1)n−1 + ... (R = 1). (5.30)
2 3 n
B) (1 + z)α :
Solución:
α(α − 1) 2 α(α − 1)(α − 2) 3
(1 + z)α = 1 + αz + z + z
2! 3!
α(α − 1) . . . (α + n − 1) n
+ ... + z + .... (R = 1).
(5.31)
n!
Un caso particular de esta serie es cuando α = −1, entonces la expansión es
1
= 1 − z + z 2 − . . . + (−1)n z n + . . . (R = 1). (5.32)
1+z
La fórmula (5.30) nos da la expansión en series de Taylor alrededor del punto
z0 = 0 del valor principal del logaritmo. Para obtener la serie de Taylor para los
otros valores de una función multivaluada log(1 + z) debemos sumar en la serie
(5.30) el valor 2πn (n = ±1, ±2, . . .).
Entonces, tenemos
z2 z3 zn
log(z + 1) = z − + − . . . + (−1)n−1 + . . . + 2nπi (R = 1) (5.33)
2 3 n
Ejemplo 5.1.8. Desarrollar en serie de Maclaurin la función cos (z 3 )
Solución: Tenemos el desarrollo de la función coseno

X (−1)n 2n
cos (z) = z (5.34)
n=0
(2n)!

Entonces, es suficiente reemplazar a z por z 3 , el resultado es:


∞ ∞
3
X (−1)n 3 2n X (−1)n 6n
cos z = (z ) = z (5.35)
n=0
(2n)! n=0
(2n)!
114 Series de Taylor y de Laurent: Singularidades
Ejemplo 5.1.9. Desarrollar en serie de Taylor la función f (z) = ez alrededor
del punto i
Solución: Sabemos el desarrollo en serie de la función ez es:

X 1 n
ez = z (5.36)
n=0
n!
Entonces, el resultado es:

X 1
ez = e(z−i+i) = ei (z − i)n (5.37)
n=0
n!

Singularidades: Una singularidad (o punto singular) z0 de una función f (z)


es un punto en el cual, la función f (z) no es analı́tica. Se dice que el punto z0 es
una singularidad aislada de f (z) si ésta no es analı́tica en z0 , pero existe una
vecindad dada por 0 < |z − z0 | < R, donde la función f (z) es analı́tica.
Existen tres diferentes tipos de singulariades. Las singularidades aisladas
entran en estos tres tipos de singulariadades que existen. Las series de potencias
de Laurent proveen un método para identificar las singularidades de una función
f (z).
Si z0 es una singularidad aislada de la función f (z), entonces tenemos la serie
de Laurent (4.62)
∞ ∞
X X bn
f (z) = an (z − z0 )n + , (5.38)
n=0 n=1
(z − z0 )n
válida en el dominio 0 < |z − z0 | < R.
Si bn = 0 para todo n, entonces para z 6= z0 la serie (5.38) se reduce a

X
f (z) = an (z − z0 )n . (5.39)
n=0

poniendo f (z0 ) = a0 hace (makes) a la función f (z) analı́tica en z0 , y z0 is


termed a removable singularity.
Por ejemplo, la función

sen z X (−1)n z 2n
f (z) = = , (z 6= 0) (5.40)
z n=0
(2n + 1)!

tiene una singularidad removible en z = 0 si ponemos f (0) = 1.


If all but finitely many bn are zero, say bn = 0 para todo n > m ≥ 1 y
bm 6= 0, entonces

b1 b2 bm X
f (z) = + 2
+ ... + m
+ an (z − z0 )n . (5.41)
z − z0 (z − z0 ) (z − z0 ) n=0

en este caso, decimos que z0 es un polo de orden m de f (z). Si m = 1,


entonces z0 es un polo simple de f (z).
5.1 Series de Potencias 115
Si un número infinito de bn no son cero en (5.38), entonces z0 es una
singularidad esencial de f (z).

Residuos Para una función f (z) con una singularidad isolated en z0 y la


serie de Laurent
∞ ∞
X bn X
f (z) = n
+ an (z − z0 )n . (5.42)
n=1
(z − z0 ) n=0

en 0 < |z − z0 | < R, el coeficiente b1 , de acuerdo con (4.64) está dado por


Z
1
b1 = f (ζ) dζ, (5.43)
2πi C

para C : |z − z0 | < r < R. Este coeficiente es muy especial debido a su


representación integral y se conoce como el residuo de f (z) en z0 , y se representa
por Res(z0 ).
In the event f (z) tiene un polo de orden m en z0 , el algoritmo

1 dm−1
Res(z0 ) = b1 = lı́m [(z − z0 )m f (z)] (5.44)
(m − 1)! z→z0 dz m−1

permite calcular el residuo de la función f (z). Cuando z0 es un polo simple (es


decir, m = 1), tenemos

Res(z0 ) = lı́m (z − z0 )f (z). (5.45)


z→z0

Este último caso, puede también ser tratado de la siguiente manera. Supongamos
que
p(z)
f (z) = , (5.46)
q(z)
donde p(z) y q(z) son funciones analı́ticas en z0 , p(z0 ) 6= 0, y q(z) tiene un
polo simple en z0 , whence f (z) tiene un polo simple en z0 . Entonces, q(z) =
(z − z0 )Q(z), Q(z0 ) 6= 0, y q 0 (z0 ) = Q(z0 ), aplicando

p(z) p(z) p(z0 )


Res(z0 ) = lı́m (z − z0 ) = lı́m = . (5.47)
z→z0 q(z) z→z0 q(z)−q(z0 ) q 0 (z0 )
z−z0

Por otro lado, si q(z0 ) = 0 y q 0 (z0 ) 6= 0, entonces

q 00 (z0 )
q(z) = q 0 (z0 )(z − z0 ) + (z − z0 )2 + . . . = (z − z0 )Q(z), (5.48)
2!
donde Q(z0 ) = q 0 (z0 ) 6= 0. Es decir, hemos mostrado que z0 es un cero simple de
q(z), hence un simple polo de f (z).
La razón de calcular el residuo es la siguiente:
116 Series de Taylor y de Laurent: Singularidades
Teorema de Cauchy para el Residuo. Sea f (z) una función analı́tica
dentro y sobre un contorno simple y cerrado C excepto en finitely many points
z1 , z2 , . . . zn lying en el interior de C figura. Entonces
Z n
X
f (z) dz = 2πi Res(zi ), (5.49)
C i=1

donde la integral se toma en dirección positiva.


Ejemplo 5.1.10. Encontrar la serie de Laurent de z −5 sen(z) con centro en 0.
Solución:

X (−1)n 2n−4 1 1 1 1 2
z −5 sen z = z = 4− 2+ − z + −... (|z| > 0)
n=0
(2n + 1)! z 6z 120 5040
(5.50)
Aquı́ la corona de convergencia es todo el plano complejo sin el origen.
cos(z)
Ejemplo 5.1.11. Encontrar la serie de Laurent de z5 alrededor de 0.
Solución: La serie para el coseno es

X (−1)n 2n
cos z = z (5.51)
n=0
(2n)!

Para z 6= 0

cos z X (−1)n 2n−5 1 1 1 1 1 1 1
= z = 5− · 3+ · − ·z + z 3 − . . . (5.52)
z5 n=0
(2n)! z 2 z 24 z 720 40320

Este es el desarrollo de Laurent de cos z


z 5 alrededor de 0. Este desarrollo tiene
exactamente tres términos que contienen potencias negativas de z, y el resto de
los términos contienen sólo potencias positivas. Podemos pensar que cos(z)
z5 =
h(z) + g(z), donde
1 1
g(z) = − z+ z3 − . . . (5.53)
720 40320
es una función diferenciable (es una serie de potencias alrededor del origen), y
1 1 1 1 1
h(z) = − · 3+ · (5.54)
z5 2 z 24 z
cos (z)
Es h(z) quien determina el comportamiento de z5 cerca del origen.
1
Ejemplo 5.1.12. Encontrar la serie de Laurent de (1+z 2 ) alrededor de -i.

Solución: Queremos una serie de potencias de z+i. La función es diferenciable


en el anillo 0 < |z + i| < 2 con centro en −i. Este anillo tiene radio 2 ya que esta
es la distancia entre −i, el centro del desarrollo propuesto, e i, el otro punto en
5.1 Series de Potencias 117
1
donde (1+z 2 ) no está definida. Para obtener potencias de z + i, primero usamos

fracciones parciales
1 1 i 1 i 1
2
= = · − · (5.55)
(1 + z ) (z + i)(z − i) 2 z+i 2 z−1

El penúltimo término ya es una potencia de z + i, de manera que lo dejamos


como está y rearreglamos el último término de la derecha:
1 1 1 1
= =− ·
z−i −2i + (z + i) 2i 1 − (z+i)
2i
∞  n ∞
1 X z+i X 1
= − =− n+1
(z + i)n (5.56)
2i n=0 2i n=0
(2i)

Este desarrollo en serie geométrica es válido para | (z+i)


2i | < 1, o |z + i| < 2.
Ahora se tiene:

    !
1 i 1 i X 1 n
= − − (z + 1)
1 + z2 2 z+i 2 n=0
(2i)n+1

i 1 i X 1
= · + · (z + 1)n (5.57)
2 z + i 2 n=0 (2i)n+1

1
Este es el desarrollo de Laurent de (1+z 2 ) en el anillo 0 < |z + i| < 2. Expresa la

función como parte de una serie de potencias y una parte que contiene potencias
negativas de z + i.
118 Series de Taylor y de Laurent: Singularidades
Capı́tulo 6

Residuos

Contents
6.1. Residuos . . . . . . . . . . . . . . . . . . . . . . . . . . 119
6.1.1. Residuo en el Infinito. . . . . . . . . . . . . . . . . . 120
6.1.2. Residuos Logarı́tmicos - Principio del Argumento. . 122
6.2. Integrales de Funciones Reales. . . . . . . . . . . . . 124
6.2.1. Integrales de Funciones Racionales de cos(θ) y sen(θ).124
6.2.2. Integrales Impropias de Funciones Racionales. . . . . 126
6.3. Ejercicios Resueltos. . . . . . . . . . . . . . . . . . . . 128

En este capitulo analizaremos los polos de funciones, el teorema del residuo


y su aplicación al cálculo de integrales de funciones reales.

6.1. Residuos
Primero se explicará qué es un residuo y cómo puede usarse para evaluar
integrales I
f (z) dz (6.1)
C
Estas serán integrales de contorno tomadas alrededor de una trayectoria simple
cerrada C.
Si f (z) es analı́tica en todas partes sobre C y dentro de C, entonces tal integral
es cero debido al teorema de Cauchy-Goursat. Si f (z) tiene una singularidad en
120 Residuos
un punto z = z0 en el interior de C, pero de otra forma es analı́tica sobre C y
dentro de C, entonces f (z) tiene una serie de Laurent

X b1 b2
f (z) = an (z − z0 )n + + + ··· (6.2)
n=0
z − z0 (z − z0 )2

que converge para todos los puntos próximos a z = z0 (excepto en z = z0 mismo),


en algún dominio de la forma 0 < |z − z0 | < R. Ahora sigue la idea crucial.
1
El cociente b1 de la primer potencia negativa z−z 0
de esta serie de Laurent
está dado por la siguiente fórmula integral utilizando n = 1
I
1
b1 = f (z) dz (6.3)
2πi c

Esto lo podemos escribir como

I
f (z) dz = 2πib1 . (6.4)
C

Aquı́ se integra en sentido contrario al movimiento de las manecillas del reloj


alrededor de la trayectoria simple cerrada C que contiene a z = z0 en su interior.
El coeficiente b1 (6.3) se denomina residuo en z = z0 y se denota por

b1 = Res f (z), z = z0 . (6.5)

6.1.1. Residuo en el Infinito.


Sea f (z) analı́tica para |z| > R. El residuo de f (z) en el ∞ se define como
sigue:

I
1
Res[f (z), ∞] = f (z) dz, (6.6)
2πi C

donde la integral se toma en la dirección negativa de una trayectoria simple


cerrada C, (a favor de las manecillas del reloj), en el dominio de analı́ticidad de
6.1 Residuos 121
la función, y fuera del cual f (z) no tiene singularidad, salvo en ∞. Por tanto se
puede decir que el residuo de f (z) en ∞ está dado por la ecuación:

Res[f (z), ∞] = −a−1 , (6.7)

donde a−1 es el coeficiente de z −1 en el desarrollo de Laurent de f (z) en ∞:


a−n a−1
f (z) = · · · + + ··· + + a0 + a1 z + · · · (6.8)
zn z
El teorema del residuo de Cauchy tiene también una extensión para incluir:
Sea f (z) analı́tica en un dominio D que incluye una vecindad diluida de ∞.
Sea C una trayectoria cerrada simple en D fuera de la cual f (z) es analı́tica
excepto para singularidades aisladas en z1 , . . . , zk . Entonces
I
f (z) dz = 2πi{Res[f (z), z1 ] + · · · + Res[f (z), zk ] + Res[f (z), ∞]} (6.9)
C

Tomando la integral sobre C en la dirección negativa y que el residuo en ∞ debe


ser incluido a la derecha.
Para una integral en particular
I
f (z) dz (6.10)
C

sobre una trayectoria simple cerrada C se tienen ahora dos formas de evaluación:
la integral es igual a 2πi veces la suma de los residuos dentro de la
trayectoria (siempre que haya solamente un número finito de singularidades
allı́), y es también igual a menos 2πi veces la suma de los residuos fuera de
la trayectoria más el de ∞ (siempre que haya un número finito de singularidades
en el dominio exterior). El principio involucrado se resume en el teorema:
Si f (z) es analı́tica en el plano z extendido excepto para un número finito de
singularidades, entonces la suma de todos los residuos de f (z) (incluyendo ∞)
es cero.
Para evaluar los residuos en ∞, se puede formular un grupo de reglas.
A) Si f(z) tiene un cero de primer orden en ∞, entonces

Res[f (z), ∞] = − lı́m zf (z) (6.11)


z→∞

Si f (z) tiene un cero de segundo orden u orden más alto en ∞, el residuo


en ∞ es 0.
B)    
1 1
Res[f (z), ∞] = −Res 2 f ,0 (6.12)
z z
122 Residuos
donde
a−1 a−2
f (z) = · · · + an z n + · · · + a1 z + a0 + + 2 + ··· , |z| > R (6.13)
z z
Entonces
 
1
f =
z
an a1 1
= ··· + n + ··· + + a0 + a−1 z + a−2 z + · · · , 0 < |z| <
z  z R
1 1 a0 a−1
f = ··· + 2 + + a−2 + · · · (6.14)
z2 z z z

Por lo que    
1 1
Res 2 f , 0 = a−1 (6.15)
z z
Este resultado reduce el problema a la evaluación de un residuo en 0.

6.1.2. Residuos Logarı́tmicos - Principio del Argumento.


Sea f (z) analı́tica en un dominio D. Entonces

f 0 (z)
, (6.16)
f (z)

es analı́tica en D excepto en los ceros de f (z). Si se elige una rama analı́tica de


log f (z) en parte de D (necesariamente exluyendo los ceros de f (z)), entonces

d f 0 (z)
log f (z) = (6.17)
dz f (z)

Por esta razón la expresión 6.16 se conoce como la derivada logarı́tmica de


f (z) y conduce al siguiente teorema.
Sea f (z) analı́tica en un dominio D. Sea C una trayectoria cerrada simple D
dentro de la cual f (z) es analı́tica excepto para un número finito de polos y sea
f (z) 6= 0 sobre C. Entonces

f 0 (z)
I
1
dz = N0 − Np (6.18)
2πi C f (z)

Donde N0 es el número total de ceros en f dentro de C y Np es el número total


de polos de f dentro de C, contándose los ceros y los polos de acuerdo a sus
multiplicidades.
6.1 Residuos 123
Conforme C recorre la trayectoria C, el punto w = f (z) traza una trayectoria
Cw en el plano w y cambiando variables se obtiene
f 0 (z)
I Z
1 1 dw
dz = (6.19)
2πi C f (z) 2πi cw w
La trayectoria Cw será una trayectoria cerrada, pero se puede cruzar varias veces
a sı́ misma. Por suposición, f (z) 6= 0 sobre C, de manera que Cw no pasa por el
origen del plano w, obteniendose la integral
Z w2
dw
= log(w2 ) − log(w1 ) (6.20)
w1 w

mide el cambio total en log(w), como log(w) varı́a continuamente sobre la


trayectoria. Si w1 = w2 , entonces

log(w2 )−log(w1 ) = log |w2 |+i arg(w2 )−[log |w1 |+i arg(w1 )] = i[arg(w2 )−arg(w1)]
(6.21)
Luego, sobre una trayectoria cerrada Cw , la integral
Z
dw
(6.22)
Cw w

es puramente imaginaria y mide i veces el cambio total en arg(w), como arg(w)


varı́a continuamente sobre la trayectoria. Este cambio total en arg(w) debe ser un
múltiplo de 2π, puesto que w1 = w2 , y puede considerarse como una medida del
número de veces que la trayectoria Cw encierra al origen en el plano w. También

−v du + u dv
Z Z
dw
=i . (6.23)
Cw w Cw u2 + v 2

La ecuación 6.23 es llamada integral de Kronecker Si


f 0 (z)
I
1 1
dz = [aumento en arg[f (z)] sobre la trayectoria] (6.24)
2πi C f (z) 2π
−v du + u dv
Z Z
1 dw 1
= N0 − Np = =
2πi Cw w 2π Cw u2 + v 2

La afirmación:
1
[aumento en arg[f (z)] sobre la trayectoria] = N0 − Np (6.25)

se conoce como el principio del argumento. El cual es de gran valor para
encontrar las raı́ces de funciones analı́ticas, el siguiente teorema explica este
principio.
124 Residuos
Teorema. Sea f (z) analı́tica en un dominio D. Sea Cz una trayectoria cerrada
simple en D, dentro de la cual f (z) es analı́tica. Si la función f (z) representa
a la curva Cz en una transformación uno a uno sobre una trayectoria cerrada
simple Cw en el plano w y f 0 (z) 6= 0 dentro de Cz , entonces f (z) representa el
interior de Cz en una transformación uno a uno en todo el interior de Cw .
H
Ejemplo 6.1.1. Evalue C cot(z) dz, con C la trayectoria dada en la Figura
6.1.

y
C
πi

x
−π π 2π 3π
−πi

−2πi

Figura 6.1: Prueba del Principio del Argumento.

Solución: Reescribiendo la integral se tiene

cos z f 0 (z)
cot z = = (6.26)
sen z f (z)

donde f (z) = sen z. Como f tiene 5 ceros simples y ningún polo encerrado por
C, el principio del argumento da
I
cot z dz = 2πi(5 − 0) = 10πi (6.27)
C

6.2. Integrales de Funciones Reales.


El teorema del residuo también constituye un método muy elegante y simple
para evaluar ciertas clases de integrales reales complicadas.

6.2.1. Integrales de Funciones Racionales de cos(θ) y sen(θ).


Primero se considerarán integrales del tipo
Z 2π
I= F [cos θ, sen θ] dθ (6.28)
0
6.2 Integrales de Funciones Reales. 125
en donde F [cos θ, sen θ] es una función racional de cos θ y sen θ. Por ejemplo,
sen2 θ iθ
5−4 cos θ , y es finita sobre el intervalo de integración. Al hacer e = z se obtiene

   
1 1 1 1 1 1
cos θ = (eiθ +e−iθ ) = z+ , sen θ = (eiθ −e−iθ ) = z− ,
2 2 z 2i 2i z
(6.29)

y se observa que el integrando se vuelve una función racional de z, por ejemplo


f (z). A medida que θ varı́a desde 0 hasta 2π, la variable z varı́a una vez alrededor
de la circunferencia unitaria |z| = 1 en sentido contrario al movimiento de las
manecillas del reloj. Como dz iθ dz
dθ = ie , se tiene que dθ = iz , y la integral dada
asume la forma I
dz
I= f (z) (6.30)
C iz
la integración se efectúa en sentido contrario al movimiento de las manecillas del
reloj alrededor de la circunferencia unitaria.

Ejemplo 6.2.1. Verificar que


Z 2π

√ = 2π (6.31)
0 2 − cos θ

Solución: Se usa cos θ = 12 (z + z1 ) y dθ = dz


iz . Ası́, la integral se vuelve

I dz I
iz dz
√ 1 1
 = i

2− − 2 − 2 2z + 1)
C 2 z + z C 2 (z
I
2 dz
= − √ √ (6.32)
i C (z − 2 − 1)(z − 2 + 1)

Se observa que el integrando tiene dos polos simples, uno en z1 = 2 + 1, que
está fuera de la circunferencia
√ unitaria C: |z| = 1 y por tanto carece de interés;
y el otro en z2 = 2 − 1 dentro de C, en donde el residuo es
 
1 1 1
Resz=z2 √ √ = √ =− (6.33)


(z − 2 − 1)(z − 2 + 1) (z − 2 − 1) z= 2−1 2

Junto con el factor − 2i antes de la integral, sustituyendo se tiene el resultado


deseado   
2 1
2πi − − = 2π (6.34)
i 2
126 Residuos
6.2.2. Integrales Impropias de Funciones Racionales.
Son integrales de este tipo
Z ∞
I= f (x) dx (6.35)
−∞

Tal integral, para la que el intervalo de integración no es finito, se denomina


integral impropia, y significa que
Z ∞ Z 0 Z b
f (x) dx = lı́m f (x) dx + lı́m f (x) dx (6.36)
−∞ a→−∞ a b→∞ 0

Si ambos lı́mites existen, entonces es posible acoplar los dos pasajes indepen-
dientes a −∞ y ∞, y escribir

Z ∞ Z R
f (x) dx = lı́m f (x) dx. (6.37)
−∞ R→∞ −R

Se supone que la función f (x) es una función racional real cuyo denominador es
diferente de cero para todo x real, y que su grado es por lo menos dos unidades
mayor que el grado del denominador. Ası́, el lı́mite existe.
Considerando la siguiente integral de contorno correspondiente
I
f (z) dz (6.38)
C

alrededor de una trayectoria C.


Como f (x) es racional, entonces f (z) tiene una infinidad de polos en el
semiplano superior, y si se elige R suficientemente grande, entonces C abarca a
todos estos polos. Entonces, por el teorema del residuo se obtiene
I I Z R X
f (z) dz = f (z) dz + f (x) dx = 2πi Res f (z) (6.39)
C S −R

en donde la suma consta de todos estos residuos de f (z) en los puntos en el


semiplano superior en donde f (z) tiene un polo. A partir de lo anterior se obtiene
Z R X Z
f (x) dx = 2πi Res f (z) − f (z) dz (6.40)
−R S

Si R → ∞, entonces el valor de la integral sobre el semicı́rculo S tiende a cero. Si


se hace z = Reiθ , entonces S está representado por R = constante, y a medida
que z varı́a a lo largo de S, la variable θ varı́a desde 0 hasta π. Como, por
6.2 Integrales de Funciones Reales. 127
hipótesis, el grado del denominador de f (z) es por lo menos dos unidades mayor
que el grado del numerador, entonces se tiene
k
|f (z)| < (|z| = R > R0 ) (6.41)
|z|2
para constantes k y R0 suficientemente grandes. Por la desigualdad M L se tiene
Z
f (z) dz < k πR = kπ

R2 (R > R0 ) (6.42)

S R
Por tanto, cuando R tiende a infinito, el valor de la integral sobre S tiende a
cero y se produce el resultado

Z ∞ X
f (x) dx = 2πi Res f (z), (6.43)
−∞

en donde se suma sobre todos los residuos f (z) correspondientes a los polos de
f (z) en el semiplano superior.
Ejemplo 6.2.2. Verificar
Z ∞
dx π
4
= √ (6.44)
0 1+x 2 2
1
Solución: f (z) = 1+z 4 tiene cuatro polos simples en los puntos
πi 3πi −3πi −πi
z1 = e 4 , z2 = e 4 , z3 = e 4 , z4 = e 4 (6.45)
Los dos primeros estan en el semiplano superior, tal y como y lo muestra en la
Figura 6.2. Entonces se tiene:
   
1 1 1 −3πi 1 πi
Resz=z1 f (z) = = = e 4 =− e4

(1 + z 4 )0 z=z1 4z 3 z=z1 4 4

   
1 1 1 −9πi 1 −πi
Resz=z2 f (z) = = = e 4 = e 4

(1 + z 4 )0 z=z2 4z 3 z=z2 4 4

(6.46)
Entonces se obtiene
Z ∞
dx 2πi πi −πi π π
4
= (−e 4 + e 4 ) = π sen = √ (6.47)
−∞ 1 + x 4 4 2 2
1
Como 1+x4 es una función par, entonces se obtiene, como se afirmó
Z ∞
1 ∞ dx
Z
dx π
= = √ (6.48)
0 1 + x4 2 −∞ 1 + x4 2 2
128 Residuos
Plano complejo z
y

z1 z0

x
0.5 1

z2 z3

Figura 6.2: Ubicación de los polos de f (z).

Ejemplo 6.2.3. Verificar



x2 − 1
Z
π
4 2
dx = (6.49)
−∞ x + 5x + 4 6
Solución: El grado del denominador es dos unidades mayor que el del
numerador, por tanto, se tiene que
p(z) z2 − 1 z2 − 1
f (z) = = 4 2
= 2 (6.50)
q(z) z + 5z + 4 (z + 4)(z 2 + 1)
que tiene polo simples en 2i e i en el semiplano superior (y en −2i y −i en
el semiplano inferior, que carecen de interés aqui). Por tanto, se calculan los
residuos observando que q 0 (z) = 4z 3 + 10z
 2 
z −1 5
Resz=2i f (z) = =

4z 3 + 10z z=2i 12i

 2 
z −1 2
Resz=i f (z) = =− (6.51)

4z 3 + 10z z=i 6i

Aplicando el teorema del residuo se tiene


5 2 π
2πi( − )= (6.52)
12i 6i 6
Como se afirmó.

6.3. Ejercicios Resueltos.


Ejemplo 2: Demostrar que la serie z(1 − z) + z 2 (1 − z) + z 3 (1 − z) + . . .
converge para |z| < 1 y hallar su suma.
6.3 Ejercicios Resueltos. 129
Solución: La suma de los primeros n términos de la serie es
sn (z) = z(1 − z) + z 2 (1 − z) + . . . + z n (1 − z)
= z − z 2 + z 2 − z 3 + . . . + z n − z n+1
= z − z n+1 .
Ahora |sn (z) − z| = | − z n+1 | = |z|n+1 < .
log  log 
Para (n + 1) log(z) < log(), o sea, n + 1 > log |z| ó n > log |z| − 1 si z 6= 0.
Si z = 0, sn (0) = 0 y |sn (0) − 0| <  para todo n. Por esto lı́mn→∞ sn (z) = z,
la suma buscada para todo z tal que |z| < 1.

Ejemplo 3: Demostrar que la serie z(1 − z) + z 2 (1 − z) + z 3 (1 − z) + . . . es


absolutamente convergente para |z| < 1.
Solución:
fn (z) = z(1 − z)| + |z 2 (1 − z)| + . . . + |z n (1 − z)|
= |1 − z|[|z| + |z|2 + |z|3 + . . . + |z|n ]
1 − |z|n
 
= |1 − z| · |z|
1 − |z|
n
Si |z| < 1, entonces lı́m |z| = 0 y lı́mn→∞ fn (z) existe de modo que la serie
es absolutamente convergente.

Ejemplo 4: Demostrar para que valores de z convergen las siguientes series:


P∞ n
A) n=0 nz2 2n .
zn
Solución: Si un = n2 2n

z n+1 n2 2n |z|

un+1
lı́m = lı́m
· n =
n→∞ un n→∞ (n + 1)2 2n+1 z 2

Aplicando la prueba de la razón, la serie converge si |z| < 2 y diverge si


|z| > 2. Si |z| = 2, la prueba de la razón falla.
P∞ zn P∞ |z|n
Sin embargo, los valores absolutos
P∞ 1 de la serie n=0 n2 2n = n=0 n2 2n
converge si |z| = 2, desde n=1 n2 converge. Ası́, la serie converge (ab-
solutamente) para |z| ≤ 2, si todos los puntos estan dentro del cı́rculo
|z| = 2.

P∞ (−1)n−1 z 2n−1 z3 z5
B) n=0 (2n−1)! =z− 3! + 5! .
Solución:
n 2n+1
−z 2

un+1
= lı́m (−1) z
(2n − 1)!
lı́m · n−1 2n−1
= lı́m =0
n→∞ un n→∞ (2n + 1)! (−1) z n→∞ 2n(2n + 1)

Entonces la serie, que representa al sen(z), converge para todos los valores
de z.
130 Residuos
P∞ n
z
Ejemplo 5: Probar que n=0 n(n+1) converge absolutamente para |z| ≤ 1.
Solución:
n
z |z|n 1 zn
Si |z| ≤ 1, entonces n(n+1) = n(n+1) ≤ n(n+1) ≤ n12 , tomando un = n(n+1) ,

1
P∞ 1 1 1
vn = n2 en el criterio de comparación y si n=0 n2 = 1p = 1p−1 , donde,
1 1 1 1 1 1 1 1 1 1 1 1 1 1
2p + 3 p ≤ 2p + 2p = 2p−1 , 4p + 5 p + 6 p + 7p ≤ 4p + 4p + 4p + p = 4p−1 .
Con lo cual se deduce que la suma de cualquier número finito de términos de
la serie dada es menor que la serie geométrica.

1 1 1 1 1
+ p−1 + p−1 + p−1 + · · · = 1
1p−1 2 4 8 1 − 2p−1
P P
Que converge para p > 1, entonces vemos que |un | converge, o sea, un
converge absolutamente.

Ejemplo 6: Hallar la región de convergencia de las siguientes series:


P∞ (−1)n−1 z 2n−1
A) n=0 (2n−1)! .
n−1 2n−1 n 2n+1
Solución: Si un = (−1)(2n−1)!z
, entonces un+1 = (−1) z
(2n+1)! . Por esto,
excluyendo z = 0 para el cual la serie dada converge, tenemos
2
(2n − 1)!|z|2

un+1 z (2n − 1)!
lı́m
= lı́m = lı́m
n→∞ un n→∞ 2n + 1 n→∞ (2n + 1)(2n)(2n − 1)!

|z|2
= lı́m =0
n→∞ (2n + 1)(2n)

Para todo z finito, de tal, modo que la serie converge (absolutamente)


para todo z, y decimos que la serie converge para |z| < ∞. Se puede decir,
equivalentemente, que el cı́rculo de convergencia es ∞ o que el radio de
convergencia es ∞.

P∞
B) n=0 n!z n .
Solución: Si un = n!z n , entonces un+1 = (n+1)!z n+1 . Entonces excluyendo
z = 0 para el cual la serie dada converge, tenemos:
n+1

un+1
= lı́m (n + 1)!z

lı́m = lı́m (n + 1)|z| = ∞
n→∞ un n→∞ n!z n n→∞

De tal modo que la convergencia es sólo en z = 0.

Ejemplo 7: Encontrar las siguientes series de Maclaurin:


6.3 Ejercicios Resueltos. 131
1
A) f (z) = 1+z 2 .

Solución: Al sustituir −z 2 por z se obtiene


∞ ∞
1 1 X
2 n
X
= = (−z ) = (−1)n z 2n
1 + z2 1 − (−z)2 n=0 n=0

= 1 − z2 + z4 − z6 + . . .

B) f (z) = tan−1 (z).


1
Solución: Se tiene f (z) = 1+z 2 . Al integrar término a término y usando
f (0) = 0, se obtiene

X (−1)n 2n+1 z3 z5
tan−1 = z =z− + − +... (|z| < 1)
n=0
2n + 1 2 5

Esta serie representa el valor principal de w = u + iv = tan−1 (z), definido


como el valor para el que |u| < π2 .

Ejemplo 8: Resolver las siguientes ecuaciones diferenciales:

A) f (z) = sen(z) z0 = 0
Solución: Derivando sucesivamente la función se obtiene:

f (z) = sen(z)

f 0 (z) = cos(z)
f 00 (z) = − sen(z)
f 000 (z) = − cos(z)
f (4) (z) = sen(z)

Sustituyendo en la serie de Taylor, tomando z0 = 0

f 0 (z0 ) f 00 (z0 ) f 000 (z0 )


f (z) = f (z0 ) + (z − z0 ) + (z − z0 )2 + (z − z0 )3 + . . .
1! 2! 3!
cos(0) sen(0) cos(0) sen(0)
f (z) = sen(0)+ (z−0)− (z−0)2 − (z−0)3 + (z−0)4 +. . .
1! 2! 3! 4!
1 1 5
f (z) = z − z 3 + z − ···
6 120
132 Residuos
B) f (z) = sen(z) z0 = i
Solución: Derivando sucesivamente la función se obtiene:

f (z) = sen(z)

f 0 (z) = cos(z)
f 00 (z) = − sen(z)
f 000 (z) = − cos(z)
f (4) (z) = sen(z)

Sustituyendo en la serie de Taylor, tomando z0 = i


f 0 (z0 ) f 00 (z0 ) f 000 (z0 )
f (z) = f (z0 ) + (z − z0 ) + (z − z0 )2 + (z − z0 )3 + . . .
1! 2! 3!
cos(i) sen(i) cos(i) sen(i)
f (z) = sen(i)+ (z−i)− (z−i)2 − (z−i)3 + (z−i)4 +. . .
1! 2! 3! 4!

Ejemplo 9: Encontrar las siguientes series de Taylor:


A) f (z) = ez , alrededor de i.
Solución:
f 0 (z0 ) f 00 (z0 ) f 000 (z0 )
f (z) = f (z0 ) + (z − z0 ) + (z − z0 )2 + (z − z0 )3 + . . .
1! 2! 3!
ei ei
f (z) = ei + ei (z − i) + (z − i)2 + (z − i)3 + . . .
2 6

B) f (z) = cos(z), alrededor de 3i.


Solución:
f 0 (z0 ) f 00 (z0 ) f 000 (z0 )
f (z) = f (z0 ) + (z − z0 ) + (z − z0 )2 + (z − z0 )3 + . . .
1! 2! 3!
cos(3i) sen(3i)
f (z) = cos(3i) − sen(3i)(z − 3i) − (z − 3i)2 + (z − 3i)3 + . . .
2 6

Ejemplo 10: Encontrar el radio de convergencia de la serie de Taylor csc(z)


alrededor de 3 − 4i.
Solución:
1
Como csc(z) = sen(z) , esta función es diferenciable excepto en z = nπ, con n
cualquier entero. Observando que π es el punto más cercano a 3 − 4i en el cual
csc(z) no es diferenciable. la distancia entre π y 3 − 4i es
p
(π − 3)2 + 16,
6.3 Ejercicios Resueltos. 133
Plano complejo z
y
π
x

p
R= (π − 3)2 + 42

(3, −4)

Figura 6.3: Figura para el Problema 10.

y éste es el radio de convergencia del desarrollo en serie de csc(z) alrededor de


3 − 4i.
Ejemplo 11: Hallar la serie de Laurent alrededor de las singularidades
indicadas para cada una de las siguientes funciones. Clasificar la singularidad en
cada caso e indique la región de convergencia de cada serie.
e2z
A) (z−1)3 ; z = 1.
Solución: Sea z − 1 = u. Entonces z = 1 + u y

e2z e2+2u e2 2u e2 (2u)2 (2u)3 (2u)4


 
= = 3 ·e = 3 1 + 2u + + + + ···
(z − 1)3 u3 u u 2! 3! 4!

e2 2e2 2e2 4e3 2e2


= 3
+ 2
+ + + (z − 1) + · · ·
(z − 1) (z − 1) z−1 3 3

z = 1 es un polo de tercer orden, o polo triple.

1
B) (z − 3) sen( z+2 ); z = −2.
Solución: Sea z + 2 = u. Entonces z = u − 2 y

1 1 h1 1 1 i
(z − 3) sen( ) = (u + 5) sen( ) = (u − 5) − 3
+ 5
− ···
z+2 z u 3!u 5!u

5 1 5 1
=1− − + + − ···
u 3!u2 3!u3 5!u4
5 1 5 1
=1− − 2
+ 3
+ − ···
z + 2 6(z + 2) 6(z + 2) 120(z + 2)4

z = −2 es una singularidad esencial. La serie converge para todos los


valores de z 6= −2.
134 Residuos
z−sen(z)
C) z3 ; z = 0.
Solución:
z3 z5 z7
  
z − sen(z) 1
= z − z − + − + · · ·
z3 z3 3! 5! 7!

1 h z3 z5 z7 i 1 z2 z4
= − + − · · · = − + − ···
z 3 3! 5! 7! 3! 5! 7!
z = 0 es una singularidad evitable. La serie converge para todos los valores
de z.

z
D) (z+1)(z+2) ; z = −2.
Solución: Sea z = −2. Entonces z + 2 = u y
z u−2 2−u 1 2−u
= = · = (1 + u + u2 + u3 + · · · )
(z + 1)(z + 2) (u − 1)u u 1−u u
u 2
+ 1 + u + u2 + · · · = + 1 + (z + 2) + (z + 2)2 + · · ·
2 z+2
z = −2 es un polo de primer orden o polo simple. La serie converge para
todos los valores de z tales que 0 < |z + 2| < 1.

1
Ejemplo 12: Desarrollar f (z) = (z+1)(z+3) en una serie de Laurent válida
para:
A) 1 < |z| < 3.
1
Solución: Resolviendo en fracciones parciales, se tiene (z+1)(z+3) =
1 1 1 1
2 ( z+1 ) − 2 ( z+3 )
Si z > 1
 
1 1 1 1 1 1 1 1 1 1
= 1 = 1 − + 2 − 3 + ··· = − 2 + 3 − 4 +· · ·
2(z + 1) 2z(1 + z ) 2z z z z 2z 2z 2z 2z

Si |z| < 3

z2 z3 1 z z2 z3
 
1 1 1 z
= = 1− + − + ··· = − + − +· · ·
2(z + 3) 6(1 + z3 ) 6 3 9 27 6 18 54 162

Entonces la serie de Laurent buscada, válida para |z| > 1 y |z| < 3, o sea
1 < |z| < 3, es

1 1 1 1 1 z z2 z3
··· − + − + − + − + − ···
2z 4 2z 3 2z 2 2z 6 18 54 162
6.3 Ejercicios Resueltos. 135
B) |z| > 3.
Solución: Si z > 1, tenemos
Si z > 1
 
1 1 1 1 1 1 1 1 1 1
= 1 = 2z 1 − + 2 − 3 + ··· = − + − +· · ·
2(z + 1) 2z(1 + z ) z z z 2z 2z 2 2z 3 2z 4

Si |z| > 3
 
1 1 1 3 9 27 1 3 9 27
= = 1− + 2 − 3 + ··· = − + − +· · ·
2(z + 3) 2z(1 + z3 ) 2z z z z 2z 2z 2 2z 3 2z 4

Entonces la serie de Laurent buscada, válida para |z| > 1 y |z| > 3, o sea,
|z| > 3, es, restando
1 4 13 40
2
− 3 + 4 − 5 + ···
z z z z

C) 0 < |z + 1| < 2.

Solución: Sea z + 1 = u, tenemos


u u2 u3
 
1 1 1 1
= = = 1− + − + ···
(z + 1)(z + 3) u(u + 2) 2u(1 + u2 ) 2u 2 4 8
1 1 1 1
= − + (z + 1) − (z + 1)2 + · · ·
2(z + 1) 4 8 16
válida para |u| < 2, u 6= 0 ó 0 < |z + 1| < 2.

D) |z| < 1.
Solución:
1 1 1 1 1 1 1
= = (1 − z + z 2 − z 3 + · · · ) = − z + z 2 − z 3 + · · ·
2(z + 1) 2(1 + z) 2 2 2 2 2

Si |z| < 3
z2 z3 1 z z2 z3
 
1 1 1 z
= z = 1− + − + ··· = − + − +· · ·
2(z + 3) 6(1 + 3 ) 6 3 9 27 6 18 54 162

Entonces la serie de Laurent buscada, válida para |z| < 1 y |z| < 3, o sea
|z| < 1, es, restando
1 4 13 40
− z − z2 − z3 + · · ·
3 9 27 81
Esta es una serie de Taylor.
136 Residuos
Ejemplo 13: Localizar en el plano finito z todas las singularidades de las
siguientes funciones:
z2
A) f (z) = (z+1)3 .
Solución: z = −1 es un polo de tercer orden.

2z 3 −z+1
B) f (z) = (z−4)2 (z−i)(z−1+2i)

Solución: z = 4 es un polo de segundo orden (polo doble); z = i y z = 1 − 2i


son polos de primer orden (polos simples).

1−cos(z)
C) f (z) = z
Solución: z = 0 aparece como una singularidad. Sin embargo, si
lı́mx→0 1−cos(z)
z = 0, esta es una singularidad removible.
h  i
z2 z4 z6
Otro método: Teniendo 1−cos(z)z = 1
z 1 − 1 − 2! + 4! − 6! + · · · =
z z3
2! − 4! + · · · , observando que si z = 0 es una singularidad removible.

sen (mz)
D) f (z) = z 2 +z+2 m 6= 0
Solución: Teniendo z 2 + 2z + 2 = 0, se obtiene:
√ p √
−b ± b2 − 4ac −2 ± (2)2 − 4(1)(2) −2 ± 4 − 8 −2 ± 2i
z1, 2 = = = = −1±i
2a 2(1) 2 2

con lo cual se puede escribir z 2 + 2z + 2 = [z − (−1 + i)][z − (−1 − i)] =


(z + 1 − i)(z + 1 + i). Obteniendo dos polos simples en: z = −1 + i y
z = −1 − i.

1
− (x−1)2
E) f (z) = e
Solución: Al desarrollar la función en series de Laurent se obtiene
1 1
1− 2
+ − ···
(z − 1) 2!(z − 1)4

La cual es una serie de Laurent donde la parte principal tiene numero


infino de términos que no son cero. Por tanto z = 1 es una singularidad
esencial.
Índice alfabético

complejo algebraica del número complejo, 14


número, 2 trigonométrica del número comple-
jo, 14
forma Residuos, 119
exponencial del número complejo, Integral de Kronecker, 123
17 Integrales de Funciones Reales, 124
Funciones Analı́ticas, 37 Integrales de Funciones Raciona-
Derivada, 54 les de cos(θ) y sen(θ), 124
Ecuaciones de Cauchy-Riemann en Integrales Impropias de Funcio-
Forma Polar, 59 nes Racionales, 126
Ecuaciones de Laplace, 60 Residuo en el Infinito, 120
Funciones Armónicas Conjugadas, Residuos Logarı́tmicos - Principio
59 del Argumento, 122

Integrales, 67 series
Forma Integral de Cauchy, 82 de potencias, 93
Primera Forma, 82
Segunda Forma, 89 valor principal, 11
Integral Compleja en Términos de
Integrales Reales, 71
Integral de una Función Compleja,
67
Integrales de Lı́nea, 69
Teorema de Cauchy-Goursat, 77

Números Complejos, 2
Cı́rculos y Discos, 30
Regiones en el Plano, 35

representación

También podría gustarte